Você está na página 1de 134

Tópicos de

Tópicos de Matemática Elementar: teoria dos números


Copyright © 2014, 2013 Antonio Caminha Muniz Neto
Direitos reservados pela Sociedade Brasileira de Matemática

Sociedade Brasileira de Matemática


Presidente: Marcelo Viana Mateniática Elenientar
Volume 5
Vice-Presidente: Vanderlei Horita
Primeiro Secretário: Ali Tahzibi
Segundo Secretário: Luiz Manoel de Figueiredo
Terceiro Secretário: Marcela Souza
Tesoureiro:Carmen Mathias Teoria dos Números
Antonio Caminha Muniz Neto
Editor Executivo
Hilário Alencar
Assessor Editorial
TiagoCosta Rocha
Coleção Professor de Matemática

Comitê Editorial
Bernardo Lima y l (p-l)q
Djairo de Figueiredo -1 (p-
2 I 2p )
Ronaldo Garcia (Editor-Chefe) 2
José Espinar
JoséCuminato
Sílvia Lopes

Capa
Pablo Diego Regino
Distribuição e vendas
Sociedade Brasileira de Matemática
Estrada DonàCastorina, 110 Sala 109 - Jardim Botânico
22460-320 Rio de Janeiro RJ !1
Telefones: (21) 2529-5073 / 2529-5095 p
http://www.sbm.org.br / email:lojavirtual@sbm.org.br X
1 J p-l
ISBN 978-85-85818-87-6 2
2ª edição
MUNIZ NETO, Antonio Caminha. 2ª impressão
Tópicos de Matemática Elementar: teoria dos números / Caminha Muniz 2014
Rio de Janeiro
-2.ed. -- Rio de Janeiro: SBM, 2013.
v.5 ; 263p. (Coleção Professor de Matemática; 28)
ISBN 978-85-85818-87-6

l.Divisibilidade. 2. Equações Diofontinas.


3. Cálculo e Teoria dos Números. 1. Título .!SBM
COLEÇÃO DO PROFESSOR DE MATEMÁTICA
•!lseM
COLEÇÃO DO PROFESSOR DE MATEMÁTICA

Logaritmos - E. L. Lima
Análise Combinatória e Probabilidade com as soluções dos exercícios -
A. C. Morgado, J. B. Pitombeira, P. C. P. Carvalho e P. Fernandez
Medida e Forma em Geometria (Comprimento, Área, Volume e Semelhança) -
E. L. Lima
Meu Professor de Matemática e outras Histórias - E. L. Lima
Coordenadas no Plano com as soluções dos exercícios - E. L. Lima com a
colaboração de P. C. P. Carvalho
Trigonometria, Números Complexos - M. P. do Carmo, A. C. Morgado e E. Wagner,
Notas Históricas de J. B. Pitombeira
Coordenadas no Espaço - E. L. Lima
Progressões e Matemática Financeira - A. C. Morgado, E. Wagner e S. C. Zani
Construções Geométricas - E. Wagner com a colaboração de J. P. Q. Carneiro
Introdução à Geometria Espacial - P. C. P. Carvalho
Geometria Euclidiana Plana - J. L. M. Barbosa
Isometrias - E. L. Lima
A Matemática do Ensino Médio Vol. 1 - E. L. Lima, P. C. P. Carvalho, E. Wagner e
A. C. Morgado
A Matemática do Ensino Médio Vol. 2 - E. L. Lima, P. C. P. Carvalho, E. Wagner e
A. C. Morgado
A Matemática do Ensino Médio Vol. 3 - E. L. Lima, P. C. P. Carvalho, E. Wagner e
A. C. Morgado
Matemática e Ensino - E. L. Lima
Temas e Problemas - E. L. Lima, P. C. P. Carvalho, E. Wagner e A. C. Morgado
Episódios da História Antiga da Matemática - A. Aaboe
Exame de Textos: Análise de livros de Matemática - E. L. Lima
A Matemática do Ensino Media Vol. 4 - Exercicios e Soluções - E. L. Lima, P. C. P.
Carvalho, E. Wagner e A. C. Morgado
Construções Geométricas: Exercícios e Soluções - S. Lima Netto
Um Convite à Matemática - D.C de Morais Filho
Tópicos de Matemática Elementar - Volume 1 - Números Reais - A. Caminha
Tópicos de Matemática Elementar - Volume 2 - Geometria Euclidiana Plana - A.
Caminha
Tópicos de Matemática Elementar - Volume 3 - Introdução à Análise - A. Caminha
A meus filhos Gabriel e Isabela,
Tópicos de Matemática Elementar - Volume 4 - Combinatória - A. Caminha na esperança de que um dia leiam este livro.
Tópicos de Matemática Elementar - Volume 5 - Teoria dos Números - A. Caminha
rr
11

Sumário

Prefácio à primeira edição


1 1

Prefácio à segunda edição

1 Divisibilidade 1
1.1 O algoritmo da divisão 2
1.2 MDC e MMC .. 13
1.3 Números primos .. 35

2 Equações Diofantinas 51
2.1 Ternos Pitagóricos 51
2.2 A equação de Pell . 62

3 Funções Aritméticas Multiplicativas 73

4 Cálculo e Teoria dos Números 91


4.1 Sobre a distribuição dos primos 91
4.2 O teorema de Chebyshev 99
4.3 O teorema de Cêsaro . . . . . . 105
SUMÁRIO

5 A Relação de Congruência 117


5.1 Definições e propriedades básicas 118
5.2 Os teoremas de Euler e Fermat 133
5.3 Congruências lineares e o teorema chinês dos restos 145

6 Classes de Congruência 153


6.1 Sistemas de restos . . . 153
6.2 O conjunto quociente Zn 161
Prefácio à primeira edição
7 Raízes Primitivas e Resíduos Quadráticos 169
7.1 Ordem módulo n .. 170
7.2 Raízes primitivas .. 175
7.3 Resíduos quadráticos 187
7.4 Somas de quadrados 202

8 Sugestões e Soluções 209 Esta coleção evoluiu a partir de sessões de treinamento para olim-
píadas de Matemática, por mim ministradas para alunos e professores
Referências 241 do Ensino Médio, várias vezes ao longo dos anos de 1992 a 2003 e,
mais recentemente, como orientador do Programa de Iniciação Cien-
A Glossário 243 tífica para os premiados na Olimpíada Brasileira de Matemática das
Escolas Públicas (OBMEP) e do Projeto Amílcar Cabral de coopera-
ção educacional entre Brasil e Cabo Verde.
Idealmente, planejei o texto como uma mistura entre uma iniciação
suave e essencialmente autocontida ao fascinante mundo das competi-
ções de Matemática, além de uma bibliografia auxiliar aos estudantes
e professores do secundário interessados em aprofundar seus conhe-
cimentos matemáticos. Resumidamente, seu propósito primordial é
apresentar ao leitor uma abordagem de quase todos os conteúdos ge-
ralmente constantes dos currículos do secundário, e que seja ao mesmo
tempo concisa, não excessivamente tersa, logicamente estruturada e
mais aprofundada que a usual.
Na estruturação dos livros, me ative à máxima do eminente mate-
mático húngaro-americano George Pólya, que dizia não se poder fazer
IT"'
,,

Prefácio à primeira edição Prefácio à primeira edição

Matemática sem sujar as mãos. Assim sendo, em vários pontos dei- tes dos livros-texto do secundário, fazem sua aparição. Numa terceira
xei a cargo do leitor a tarefa de verificar aspectos não centrais aos etapa, o texto apresenta outros métodos elementares usuais no estudo
desenvolvimentos principais, quer na forma de detalhes omitidos em da geometria, quais sejam, o método analítico de R. Descartes, a tri-
demonstrações, quer na de extensões secundárias da teoria. Nestes ca- gonometria e o uso de vetores; por sua vez, tais métodos são utilizados
sos, frequentemente referi o leitor a problemas específicos, os quais se tanto para reobter resultados anteriores de outra(s) maneira(s) quanto
encontram marcados com* e cuja análise e solução considero parte in- para deduzir novos resultados.
tegrante e essencial do texto. Colecionei ainda, em cada seção, outros De posse do traquejo algébrico construído no volume inicial e do
tantos problemas, cuidadosamente escolhidos na direção de exercitar aparato geométrico do volume dois, discorremos no volume três sobre
os resultados principais elencados ao longo da discussão, bem como aspectos elementares de funções e certos excertos de cálculo diferencial
estendê-los. Uns poucos destes problemas são quase imediatos, ao e integral e análise matemática, os quais se fazem necessários em cer-
passo que a maioria, para os quais via de regra oferto sugestões preci- tos pontos dos três volumes restantes. Prescindindo, inicialmente, das
sas, é razoavelmente difícil; no entanto, insto veementemente o leitor a noções básicas do Cálculo, elaboramos, dentre outros, as noções de
debruçar-se sobre o maior número possível deles por tempo suficiente gráfico, monotonicidade e extremos de funções, bem como examina-
para, ainda que não os resolva todos, passar a apreciá-los como corpo mos o problema da determinação de funções definidas implicitamente
de conhecimento adquirido. por relações algébricas. Na continuação, o conceito de função contínua
O primeiro volume discorre sobre vários aspectos relevantes do con- é apresentado, primeiramente de forma intuitiva e, em seguida, axio-
junto dos números reais e de álgebra elementar, no intuito de munir o mática, sendo demonstrados os principais resultados pertinentes. Em
leitor dos requisitos necessários ao estudo dos tópicos constantes dos especial, utilizamos este conceito para estudar a convexidade de gráfi-
volumes subsequentes. Após começar com uma discussão não axiomá- cos - culminando com a demonstração da desigualdade de J. Jensen -
tica das propriedades mais elementares dos números reais, são aborda- e o problema da definição rigorosa da área sob o gráfico de uma fun-
dos, em seguida, produtos notáveis, equações e sistemas de equações, ção contínua e positiva - que, por sua vez, possibilita a apresentação
sequências elementares, indução matemática e números binomiais; o de uma construção adequada das funções logaritmo natural e expo-
texto finda com a discussão de várias desigualdades algébricas impor- nencial. O volume três termina com uma discussão das propriedades
tantes, notadamente aquela entre as médias aritmética e geométrica, mais elementares de derivadas e do teorema fundamental do cálculo,
bem como as desigualdades de Cauchy, de Chebychev e de Abel. os quais são mais uma vez aplicados ao estudo de desigualdades, em
Dedicamos o segundo volume a uma iniciação do leitor à geometria especial da desigualdade entre as médias de potências.
Euclidiana plana, inicialmente de forma não axiomática e enfatizando O volume quatro é devotado à análise combinatória. Começamos
construções geométricas elementares. Entretanto, à medida em que o revisando as técnicas mais elementares de contagem, enfatizando as
texto evolui, o método sintético de Euclides - e, consequentemente, construções de bijeções e argumentos recursivos como estratégias bá-
demonstrações - ganha importância, principalmente com a discussão sicas. Na continuação, apresentamos um apanhado de métodos de
dos conceitos de congruência e semelhança de triângulos; a partir desse contagem um tanto mais sofisticados, como o princípio da inclusão
ponto, vários belos teoremas clássicos da geometria, usualmente ausen- exclusão e os métodos de contagem dupla, do número de classes de
Prefácio à primeira edição Prefácio à primeira edição

equivalência e mediante o emprego de métricas em conjuntos finitos. exemplos discutidos e dos problemas propostos ao longo do texto, boa
A cena é então ocupada por funções geradoras, onde a teoria elementar parte dos quais oriundos de variadas competições ao redor do mundo.
de séries de potências nos permite discutir de outra maneira problemas Finalmente, números complexos e polinômios são os objetos de
antigos e introduzir problemas novos, antes inacessíveis. Terminada estudo do sexto e último volume da coleção. Para além da teoria
nossa excursão pelo mundo da contagem, enveredamos pelo estudo do correspondente usualmente estudada no secundário - como a noção
problema da existência de uma configuração especial no universo das de grau, o algoritmo da divisão e o conceito de raízes de polinômios
configurações possíveis, utilizando para tanto o princípio das gavetas -, vários são os tópicos não padrão abordados aqui. Dentre outros,
de G. L. Dirichlet - vulgo "princípio das casas dos pombos" -, um destacamos inicialmente a utilização de números complexos e polinô-
célebre teorema de R. Dilworth e a procura e análise de invariantes mios como ferramentas de contagem e a apresentação quase completa
associados a problemas algorítmicos. A última estrutura combinatória de uma das mais simples demonstrações do teorema fundamental da
que discutimos é a de um grafo, quando apresentamos os conceitos bá- álgebra. A seguir, estudamos o famoso teorema de I. Newton sobre
sicos usuais da teoria com vistas à discussão de três teoremas clássicos polinômios simétricos e as igualmente famosas desigualdades de New-
importantes: a caracterização da existência de caminhos Eulerianos, ton, as quais estendem a desigualdade entre as médias aritmética e
o teorema de A. Cayley sobre o número de árvores rotuladas e o teo- geométrica. O próximo tema concerne os aspectos básicos da teoria
rema extremal de P. Turán sobre a existência de subgrafos completos de interpolação de polinômios, quando dispensamos especial atenção
em um grafo. aos polinômios interpoladores de J. L. Lagrange. Estes, por sua vez,
Passamos em seguida, no quinto volume, à discussão dos conceitos são utilizados para resolver sistemas lineares de Vandermonde sem o
e resultados mais elementares de teoria dos números, ressaltando-se recurso à álgebra linear, os quais, a seu turno, possibilitam o estudo
inicialmente a teoria básica do máximo divisor comum e o teorema de uma classe particular de sequências recorrentes lineares. O livro
fundamental da aritmética. Discutimos também o método da descida termina com o estudo das propriedades de fatoração de polinômios
de P. de Fermat como ferramenta para provar a inexistência de solu- com coeficientes inteiros, racionais ou pertencentes ao conjunto das
ções inteiras para certas equações diofantinas, e resolvemos também classes de congruência relativas a algum módulo primo, seguido do
a famosa equação de J. Pell. Em seguida, preparamos o terreno para estudo do conceito de número algébrico. Há, aqui, dois pontos cul-
a discussão do famoso teorema de Euler sobre congruências, constru- minantes: por um lado, uma prova mais simples do fechamento do
indo a igualmente famosa função de Euler com o auxílio da teoria mais conjunto dos números algébricos em relação às operações aritméticas
geral de funções aritméticas multiplicativas. A partir daí, o livro apre- básicas; por outro, o emprego de polinômios ciclotômicos para provar
senta formalmente o conceito de congruência de números em relação a um caso particular do teorema de Dirichlet sobre primos em progres-
um certo módulo, discutindo extensivamente os resultados usualmente sões aritméticas.
constantes dos cursos introdutórios sobre o assunto, incluindo raízes Várias pessoas contribuíram ao longo dos anos, direta ou indire-
primitivas, resíduos quadráticos e o teorema de Fermat de caracteriza- tamente, para que um punhado de anotações em cadernos pudesse
ção dos inteiros que podem ser escritos como soma de dois quadrados. transformar-se nesta coleção de livros. Os ex-professores do Departa-
O grande diferencial aqui, do nosso ponto de vista, é o calibre dos mento de Matemática da Universidade Federal do Ceará, Marcondes
Prefácio à primeira edição Prefácio à primeira edição

Cavalcante França, João Marques Pereira, Guilherme Lincoln Aguiar ram várias sug~stões. Os pareceristas indicados pela SBM opinaram
Ellery e Raimundo Thompson Gonçalves, ao criarem a Olimpíada Cea- decisivamente para que os livros certamente resultassem melhores que
rense de Matemática na década de 1980, motivaram centenas de jovens a versão inicial por mim submetida. O presidente da SBM, professor
cearenses, dentre os quais eu me encontrava, a estudarem mais Ma- Hilário Alencar da Silva, o antigo editor-chefe da SBM, professor Ro-
temática. Meu ex-professor do Colégio Militar de Fortaleza, Antônio berto Imbuzeiro de Oliveira, bem como o novo editor-chefe, professor
Valdenísio Bezerra, ao convidar-me, inicialmente para assistir a suas Abramo Refez, foram sempre extremamente solícitos e atenciosos co-
aulas de treinamento para a Olimpíada Cearense de Matemática e pos- migo ao longo de todo o processo de edição. Por fim, quaisquer erros
teriormente para dar aulas consigo, iniciou-me no maravilhoso mundo ou incongruências que ainda se façam presentes, ou omissões na lista
das competições de Matemática e influenciou definitivamente minha acima, são de minha inteira responsabilidade.
escolha profissional. Os comentários de muitos de vários de ex-alunos Por fim e principalmente, gostaria de agradecer a meus pais, Anto-
contribuíram muito para o formato final de boa parte do material nio Caminha Muniz Filho e Rosemary Carvalho Caminha Muniz, e à
aqui colecionado; nesse sentido, agradeço especialmente à João Luiz minha esposa Mônica Valesca Mota Caminha Muniz. Meus pais me fi-
de Alencar Araripe Falcão, Roney Rodger Sales de Castro, Marcelo zeram compreender a importância do conhecimento desde a mais tenra
Mendes de Oliveira, Marcondes Cavalcante França Jr., Marcelo Cruz idade, sem nunca terem medido esforços para que eu e meus irmãos
de Souza, Eduardo Cabral Balreira, Breno de Alencar Araripe Falcão, desfrutássemos o melhor ensino disponível; minha esposa brindou-me
Fabrício Siqueira Benevides, Rui Facundo Vigelis, Daniel Pinheiro So- com a harmonia e o incentivo necessários à manutenção de meu ânimo
breira, Antônia Taline de Souza Mendonça, Carlos Augusto David Ri- e humor, em longos meses de trabalho solitário nas madrugadas. Esta
beiro, Samuel Barbosa Feitosa, Davi Máximo Alexandrino Nogueira coleção de livros também é dedicada a eles.
e Yuri Gomes Lima. Vários de meus colegas professores teceram co-
mentários pertinentes, os quais foram incorporados ao texto de uma
ou outra maneira; agradeço, em especial, a Fláudio José Gonçalves,
Francisco José da Silva Jr., Onofre Campos da Silva Farias, Emanuel FORTALEZA, JANEIRO de 2012
Augusto de Souza Carneiro, Marcelo Mendes de Oliveira, Samuel Bar-
bosa Feitosa e Francisco Bruno de Lima Holanda. Os professores João
Antonio Caminha M. Neto
Lucas Barbosa e Hélio Barros deram-me a conclusão de parte destas
notas como alvo a perseguir ao me convidarem a participar do Pro-
jeto Amílcar Cabral de treinamento dos professores de Matemática da
República do Cabo Verde. Meus colegas do Departamento de Mate-
mática da Universidade Federal do Ceará, Abdênago Alves de Barros,
José Othon Dantas Lopes, José Robério Rogério e Fernanda Esther
Camillo Camargo, bem como meu orientando de iniciação científica
Itamar Sales de Oliveira Filho, leram partes do texto final e oferece-
Prefácio à primeira edição

Prefácio à segunda edição

Para a segunda edição fiz uma extensa revisão do texto e dos pro-
blemas propostos, corrigindo várias imprecisões de língua portuguesa
e de Matemática. Adicionei também alguns problemas novos, no in-
tuito de melhor exercitar certos pontos da teoria, os quais não se en-
contravam adequadamente contemplados pelos problemas propostos
à primeira edição. Diferentemente da primeira edição, nesta segunda
edição as sugestões e soluções aos problemas propostos foram cole-
cionadas em um capítulo separado (o capítulo 8, para este volume);
adicionalmente, apresentei sugestões ou soluções a praticamente todos
os problemas do livro.
Por fim, gostaria de aproveitar o ensejo para agradecer à comu-
nidade matemática brasileira, em geral, e a todos os leitores que me
enviaram sugestões ou correções, em particular, o excelente acolhi-
mento desfrutado pela primeira edição desta obra.

FORTALEZA, MAIO de 2013

Antonio Caminha M. Neto


Prefácio à segunda edição

CAPÍTULO 1

Divisibilidade
1

Este capítulo é devotado ao estabelecimento das definições e pro-


priedades elementares concernentes à relação de divisibilidade no con-
junto dos números inteiros, enfatizando o algoritmo da divisão, a noção
de máximo divisor comum e o papel fundamental desempenhado pe-
los números primos. Em que pese o caráter elementar dos argumentos
utilizados para tal fim, encontraremos ao longo do caminho vários pro-
blemas e resultados interessantes, destacados, dentre esses últimos, a
caracterização dada por Bézout para o máximo divisor comum de dois
inteiros e o teorema de Euclides sobre a infinitude do conjunto dos
números primos. Ao leitor interessado em aprofundar seus conheci-
mentos de teoria dos números para além do material que discutimos
neste volume, sugerimos o clássico [5].

1
1.
2 Divisibilidade 1.1 O algoritmo da divisão 3

1.1 O algoritmo da divisão Como ak-l + ak- 2b+ · · · + abk- 2 + bk-l E Z, basta usar a definição 1.1.

Definição 1.1. Dados a, b E Z, com b -/=- O, dizemos que b divide a,


(b) Para k ímpar, recorrendo uma vez mais ao problema 2.1.18 do
e escrevemos b [ a, se existir e E Z tal que a= bc. Caso b não divida
volume 1, temos a fatoração
a, escrevemos b f a.

Seja b um inteiro não nulo. Se b dividir a, dizemos que b é um


divisor de a, que a é divisível por b ou ainda que a é um múltiplo com ak-l_ak- 2b+· · ·-abk- 2+bk-l também inteiro. É, pois, suficiente
de b. Se b [ a e b > O, então b é um divisor positivo de a. Note que utilizar novamente a definição 1.1. •
todo inteiro não nulo é um divisor de si mesmo e de O.
Exemplo 1.4. Prove que, para todo k EN, o número lük - 1 é um
Exemplo 1.2. Um inteiro n é par se for um múltiplo de 2; caso múltiplo de 9.
contrário, n é ímpar. De acordo com a definição 1.1, os inteiros pares
são precisamente os que podem ser escritos na forma 2k, para algum Prova. Basta aplicar o item (a) do exemplo anterior: lük -1 = lOk -
k E Z, i.e., são os inteiros 1k é divisível por 10 - 1 = 9. Outras possibilidades são observar que

... , -6, -4, -2, o, 2, 4, 6, ....


k algarismos k algarismos

Os inteiros restantes, i.e., claramente um múltiplo de 9, ou escrever

L (k). 9k-j - L (k). 9k-j,


... , -5, -3, -1, 1, 3, 5, ... ,
rnk - 1 = (9 + l)k - 1 = k
1= k-l

são os ímpares. Assim, todo ímpar é igual a um par mais 1, de modo j=O J j=O J
que podemos denotar um inteiro ímpar genérico escrevendo 2k + 1,
onde k E Z.
novamente um múltiplo de 9.

A proposição a seguir estabelece algumas propriedades básicas da
Exemplo 1.3. Dados a, b inteiros e k natural, temos que:
relação de divisibilidade, as quais o leitor deve guardar para uso futuro.
(a) Se a-/=- b, então (a - b) 1 (ak - bk). Proposição 1.5. Sejam a, b, e inteiros não nulos e x, y inteiros quais-
quer.
(b) Se a-/=- -b e k for ímpar, então (a+ b) 1 (ak + bk).
Solução. (a) Se b I a e a I b, então a = ±b.
(a) Sabemos, do problema 2.1.18 do volume 1 que (b) Se e I b e b I a, então e I a.
ak - bk = (a - b)(ak-1 + ak-2b + ... + abk-2 + bk-1). (c) Se e I a e e I b, então e 1 ( ax + by).
4 Divisibilidade 1.1 O algoritmo da divisão 5

(d) Se b Ia, então lbl : : ; lal. Observações 1.6.

(e) Se e I b, então e I ab. i. Como caso particular do item (c) da proposição anterior, se e I a
e e I b, então e 1 (a± b). Utilizaremos essa observação várias
(f) Se b I a, então bc I ac. vezes no que segue.
Prova. 11. O item (c) da proposição acima pode ser facilmente generalizado
(a) Se a' e b' são inteiros tais que a= ba' e b = ab', então a= (ab')a' =
para provar que, Se C I a1, ... , an, então C 1 (a1X1 + · · · + anXn),
a(a'b') e, daí, a'b' = 1. Logo, a'= ±1, donde segue que a= ±b. para todos X1, ... , Xn E Z.

(b) Se a = ba' e b = cb', com a', b' E Z, então a = (cb')a' = c(a'b'), 111. Segue do item (d) acima que todo inteiro não nulo tem somente
com a'b' E Z. Logo, e I a. um número finito de divisores.

Continuando, recorde (cf. problema 1.1.6, volume 3) que, dado


(c) Sejam a = ca' e b = cb', com a', b' E Z. Então ax + lYy
x E JR, sua parte inteira l x J é definida por
ca'x + cb'y = c(a'x + b'y), com a'x + b'y E Z. Logo, e 1 (ax + by).
lxJ = max{n E Z; n::::; x}. (1.1)
(d) Se a = ba', com a' E Z, então a -=1- O ~ a' -=1- O e, daí, la'I ~ 1.
Logo, lal = lba'I = lblla'I ~ lbl. De outro modo, para n E Z, temos

(e) Se b = cb', com b' E Z, então ab = e( ab'), com ab' E Z. Logo, e I ab. lxJ =n{=:}n:::;x<n+l. (1.2)

Por exemplo, como 1 < v2 < 2, temos l v2J = 1; como -3 < -2, 3 <
(f) Se a= ba', com a' E Z, então ac = (bc)a' e, daí, bc I ac.
• -2, temos l-2, 3J = -3.
A próxima proposição é conhecida como o algoritmo da divisão
para números inteiros e é um dos pilares da teoria básica de divisibi-
lidade.

Proposição 1. 7. Dados a, b E Z, com b i- O, existem únicos q, r E Z


tais que a= bq + r, com O::::; r < lbl. Ademais, se b > O, então

q= liJ e r =a- liJ b. (1.3)

Tais inteiros q e r são, respectivamente, o quociente e o resto da


divisão de a por b.
6 Divisibilidade 1.1 O algoritmo da divisão 7

Prova. Suponha primeiro b > O, e seja q o maior inteiro tal que • Se n = 3q, então n 2 = 3. 3q 2 .
bq :::; a. Então bq :::; a < b( q + 1), de modo que O :::; a - bq < b
e basta definir r = a - bq (observe que, caso b > O, tal inteiro q é • Se n = 3q + 1, então n 2 = 3 (3q 2 + 2q) + 1.
precisamente llJ ). Se b < O, então -b > O, donde existem q, r E Z
tais que a = (-b)q + r, com O :::; r < -b. Daí, a = b(-q) + r, com • Se n = 3q + 2, então n 2 = 3 (3q 2 + 4q + 1) + 1.
O:::; r < -b = lbl.
No primeiro caso acima, n 2 deixa resto O quando dividido por 3; nos
Suponha, agora, que a = bq + r = bq' + r', onde q, q', r, r' E Z e
outros dois casos, n 2 deixa resto 1 quando dividido por 3.
O:::; r, r' < lbl. Então

Ir' - ri < lbl e b(q - q') = r' - r. (b) Novamente pelo algoritmo da divisão, o resto da divisão de n por 4
é O, 1, 2 ou 3, de modo que n = 4q ,4q+ 1, 4q+2 ou 4q+3, para algum
Se q-::/=- q', então lq - q'I 2 1, de modo que q E Z, e podemos dar uma prova análoga à do item (a). Vejamos,
contudo, uma prova mais simples: invocando o algoritmo da divisão
lbl :::; lbl · lq - q'I = Ir' - ri < lbl, (ou o exemplo 1.2), temos n = 2q ou 2q + 1 para algum q E Z.

uma contradição. Portanto, q = q' e, daí, r = r'.


• • Se n = 2q, então n 2 = 4q 2 .
O corolário a seguir, além de importante em si, ilustra em sua
• Se n = 2q + 1, então n 2 = 4(q 2 + q) + 1.
prova a utilização típica que fazemos do algoritmo da divisão. Para o
enunciado do mesmo, lembre-se de que um inteiro positivo m é dito No primeiro caso, n 2 deixa resto O quando divididido por 4; no se-
um quadrado perfeito se m = n 2 , para algum inteiro n (o qual, gundo, n 2 deixa resto 1 quando dividido por 4.
sempre que conveniente, podemos supor não negativo).

Corolário 1.8. Todo quadrado perfeito: (c) Uma vez mais, poderíamos apelar diretamente ao algoritmo da
divisão, escrevendo n = 8q + r para algum q E Z e algum r E
(a) deixa resto O ou 1 quando dividido por 3. {O, 1, 2, ... , 7}, dando uma prova análoga à do item (a). No entanto,
temos pelo item (b) que:
(b) deixa resto O ou 1 quando dividido por 4.
• Se n = 2q, então n 2 = 4q 2 . Há, agora, duas possibilidades:
(c) deixa resto O, 1 ou 4 quando dividido por 8.

Prova. Seja num natural. - Se q2 for par, digamos q2 = 2k para algum k E N, então
n 2 = 8k.

(a) Pelo algoritmo da divisão, o resto da divisão de n por 3 é O, 1 ou - Se q2 for ímpar, digamos q2 = 2k + 1 para algum k E N,
2, de modo que n = 3q, 3q + 1 ou 3q + 2, para algum q E Z. Agora: então n 2 = 8k + 4.
~
,. 'I
i
8 Divisibilidade 1.1 O algoritmo da divisão 9
i

• Se n = 2q + 1, então n 2 = 4q(q + 1) + 1. Mas, como ao menos Sejam, agora, n: EN e q, r E Z tais que n = lOq+r, com O~ r ~ 9.
um dos inteiros q, q + 1 é par, temos q(q + 1) = 2k para algum Então
k EN, de sorte que n 2 = 8k + 1.
n2 (lüq + r )2 = 100q2 + 20qr + r 2
Por fim, os casos acima garantem que o resto da divisão de n 2 por 10(10k2 + 2kr) + r 2,
8 só pode ser igual a O, 1 ou 4. •
de sorte que 10 1 (n 2 - r 2) e o corolário 1.9 garante que n 2 e r 2 deixam
Corolário 1.9. Dados inteiros a 1, a 2 e b, sendo b não nulo, temos que restos iguais na divisão por 10. Portanto, segue de nossa observação
b 1 (a 1 - a 2) se, e só se, a 1 e a 2 deixam restos iguais na divisão por b. inicial que o último algarismo de n 2 é igual ao último algarismo de r 2.
Para terminar, basta checar quais são os últimos algarismos dos
Prova. Suponha primeiro que a 1 = bq 1 + r e a2 = bq2 + r, com
números r 2 quando r varia de O a 9: 02 = O; 12 e 92 terminam em 1;
q1, q2, r E Z. Então a 1 - a 2 = b(q 1 - q2), i.e., b 1 (a1 - a2). Reciproca-
22 e 8 2 terminam em 4; 3 2 e 72 terminam em 9; 42 e 62 terminam em
mente, suponha que b 1 (a1 - a2), e sejam a1 = bq1 + r1, a2 = bq2 + r2,
6; 52 termina em 5. Assim, os possíveis últimos algarismos de n 2 são
com q1 , q2, r 1, r 2 E Z e O~ r1, r2 < lbl. Então
o, 1, 4, 5, 6 ou 9. •

Exemplo 1.11. Seja n > 1 inteiro. Mostre que qualquer sequência


de n inteiros consecutivos possui exatamente um múltiplo de n.
e, como b 1 (a 1 -a2) e b 1 (q 1 -q2)b, segue do item (c) da proposição 1.5
que b divide r 1 - r 2 = (a 1 - a 2) - (q1 - q2)b. Por outro lado, O ~ Prova. Seja a+ 1, a+ 2, ... , a+ numa sequência de n inteiros con-
r 1 , r 2 < lbl implica !ri - r 2 < lbl, de modo que a única possibilidade
1 secutivos, com a= nq + r, O~ r < n. Então O< n - r ~ n, de modo
é ser lr 1 - r 21 = O ou, ainda, r1 = r2: • que a + (n - r) é um dos números de nossa sequência. Mas

Como aplicação do corolário acima, calculamos a seguir os possíveis a + (n - r) = (nq + r) + (n - r) = n( q + 1),


valores do último algarismo de um quadrado perfeito.
um múltiplo de n.
Exemplo 1.10. O último algarismo de um quadrado perfeito só pode Para o que falta, usemos novamente o corolário 1.9: os números
ser O, 1, 4, 5, 6 ou 9. a+ 1, a+ 2, ... , a+ n deixam restos dois a dois distintos na divisão
por n, uma vez que o módulo da diferença de dois quaisquer deles é
Prova. Observe primeiro que o resto da divisão de um número natural
no mínimo 1 e no máximo n - 1. Logo, há no máximo um múltiplo
por 10 coincide com seu último algarismo (o algarismo mais à direita,
de n em nossa sequência. •
na representação decimal do número). De fato, sejam= lüm' + ao,
com m' E Z+ e O~ a0 ~ 9; como o inteiro lüm' termina por O, segue
que a representação decimal da soma lüm' + a0 ( que é igual a m)
termina à direita com o algarismo a0 .
10 Divisibilidade 1.1 O algoritmo da divisão 11

Problemas - Seção 1.1 Para o próximo problema, dado m E Z, denotemos por Zm o


conjunto
1. Prove o critério de divisibilidade por 9: o resto da divisão
de um número natural por 9 é igual ao resto da divisão por Zm = {mq; q E Z} = {O, ±1, ±2, ±3, ... }.
9 da soma dos algarismos de sua representação decimal. Em
particular, um natural é múltiplo de 9 se, e só se, a soma de seus 9. Seja S um conjunto de números inteiros, contendo O e satisfa-
algarismos o for. zendo a seguinte propriedade:

2. Encontre o resto da divisão do número lük por 11. x, y E s, X=/:- y =} X - y E S.

3. Prove o critério de divisibilidade por 11: se o número natural Se S =/:- {O}, prove que:
n tem representação decimal n = (akak-l ... a 1 a 0 ) 10 , então o
(a) SnN=/:-0.
resto da divisão de n por 11 é igual ao resto da divisão por 11
da soma alternada (b) Se x, y E S, então x + y E S.
(c) Sem= min(S n N), então S = Zm.

10. * Calcule o resto da divisão de 264 + 1 por 232 + 1. Mais geral-


Em particular, 11 n se, e só se, 11 dividir a soma alternada dos
1
mente, dados a, m e n inteiros maiores que 1 e tais que m > n,
algarismos de n. calcule o resto da divisão de a 2m + 1 por a 2 n + 1.

4. (IMO.) Encontre todos os naturais n de três algarismos, tais que 11. * Para x E IR, prove os seguintes itens:
11 n e {l é igual à soma dos quadrados dos algarismos de n.
1

(a) x -1 < lxJ ::::; x.


5. (IMO.) Ache todos os n E N tais que o produto dos algarismos (b) lX + 1j = lX j + 1.
da representação decimal de n seja igual a n 2 - lOn - 22.
se x E Z
(c) lxJ+l-xJ={ O,
6. * Dado um natural n, prove que o produto de n inteiros conse- -1, se x (/:. Z
cutivos é sempre divisível por n!. 12. * Sejam x, y E IR em, n inteiros quaisquer, sendo n > O. Prove
os seguintes itens:
7. (Hungria.) Para n EN, prove que o número gn - 3n - 5n + ln é
um múltiplo de 10. (a) X ::::; lX j ::::; lY J·
Y =}

8. (Hungria.) Prove que 5 divide 199 + 299 + 399 + 499 + 599 . (b) lnxJ ~ n lx J.
(c) l m:1 J :=::; l~ J + 1.
n.····.·1··
i'·
! '
1
1

12 Divisibilidade 1.2 MDC e MMC 13

(d) ll:JJ = l~U- 20. (OBM.) Prove que não existem inteiros x e y tais que 15x2
7y 2 = 9.
-

(e) lx J + lY J ::; lX + y J ::; lX J + lY J + 1.


(f) lX + y j + lX j + lY j ::; l2x j + l2y j . 21. (França.) Para k, m, n E N, com k > +1
2, sejam Fm = 22=
o m-ésimo número de Fermat e Gn = n(n2- 1 ) (k - 2) + n.
13. Para x E IR, prove que o inteiro mais próximo de x é o inteiro Fixado k, queremos encontrar (se existirem) os m, n E N tais
lX+ !J. que Fm = Gn· Para tanto, faça os seguintes itens:
14. (ORM.) Encontre todos os a, b EN tais que
(a) Prove que, se existir um tal par (m, n), então há um inteiro
não negativo r tal que n = 2r + 1.
l-ª2J
b
+ lb2j
- + 1 = lª2 + b2J +ab.
a ab (b) Conclua, a partir de (a), que k - 1 = 2rt 1 , para algum
t1 EN, de sorte que 2 2= + 1 = 2 2r(2rt1 + t1 - 1).
15. Prove que não existem inteiros ímpares x, y e z tais que
(c) Mostre, por indução sobre o natural l, que se tivermos 22= +
(x + y) 2 + (y + z) 2 = (x + z)2. 1 = 2U+1)r(2rt1 +t1 =f 1), para algum t1 EN, então 22 = + 1 =
2(1+ 2 )r(2rt1+1 + tl+l ± 1).
16. Sejam x, y e z inteiros tais que x 2+ y 2= z2. Prove que 6 1xy. (d) Conclua sobre a existência de tais pares (m, n).

Para o problema a seguir, lembre-se de que um cubo perfeito


é um inteiro da forma q3 , para algum q E Z. 1.2 MDCeMMC
17. (OBM - adaptado.) Se a 1 , ... , an são inteiros não nulos dados, dizemos que um inteiro
(a) Encontre os possíveis restos da divisão de um cubo perfeito d é um divisor comum de a1, ... , an quando d I a 1, ... , d 1 ªn· Note
que a 1 , ... , an sempre têm divisores comuns: 1, por exemplo. Ademais,
por 7.
desde que qualquer inteiro não nulo tem apenas um número finito de
(b) Se x, y, z são inteiros tais que x 3 + y 3 - z 3 é múltiplo de 7, divisores, a1 , ... , an têm apenas um número finito de divisores comuns.
prove que ao menos um dentre os números x, y, zé múltiplo Assim, a definição a seguir tem sentido.
de 7.
Definição 1.12. O máximo divisor comum dos inteiros não nulos
18. (OBM.) Ache todos os x, y, z, n naturais tais que nx + nY = nz.
a 1 , a 2 , ... , an, denotado mdc (a 1 , a 2 , ... , an), é o maior dentre os divi-
19. (União Soviética.) Para n > 3 inteiro, mostre que o número sores comuns de a 1 , a 2 , ... , ªn· Os inteiros a 1 , a 2 , ... , an são primos
1! + 2! + 3! + · · · + n! nunca é um quadrado perfeito. entre si, ou relativamente primos, se mdc (a1, a2, ... , an) = 1.
14 Divisibilidade 1.2 MDC e MMC 15

Para o teorema a seguir, devido ao matemático francês Étienne Prova. É imediato que todo múltiplo de um elemento de S pertence
Bézout, dado n E Z denote por nZ o conjunto dos múltiplos inteiros a S. Por outro lado, como d divide a1x1 + a2x2 + · · ·+ anXn para todos
de n, i.e., x 1 , x2, ... , Xn E Z, temos que Se dZ.
Para estabelecer a inclusão contrária, note primeiro que S contém
nZ = {nx; x E Z} = {O, ±n, ±2n, ±3n, .. .}.
inteiros positivos; de fato, escolhendo x 1 = a 1 e x 2 = · · · = Xn = O,
por exemplo, concluímos que

Como S contém inteiros positivos, existe um menor inteiro positivo d'


em S. Se mostrarmos que d'= d, seguirá que d E Se nossa observação
inicial garantirá que dZ e S.
Afirmamos, inicialmente, que d' 1 a 1 , ... , ªn· De fato, como d' E S,
Figura 1.1: Étienne Bézout, matemático francês do
existem u1, u2, ... , Un E Z tais que d' = a1u 1 + a2u 2 + · · · + anun.
século XVIII, um dos precursores da área da Ma-
Agora, seja a 1 = d'q + r, com q, r E Z e O :Sr< d'. Então
temática hoje conhecida como Geometria Algébrica.
Em que pese seu teorema sobre o mdc, o mais fa- r ª1 - d'q
moso teorema de Bézout é provavelmente aquele que
afirma que, sendo f e g polinômios reais em X e a1 - (a1u1 + a2u2 + · · · + anun)q
Y, sem fatores comuns e de graus respectivamente a1(l - u1q) + a2(-u2q) + · · · + an(-unq),
m e n, o número de pontos de interseção das cur-
de sorte que r E S. Se O < r < d', teríamos uma contradição ao
vas f (x, y) = O e g(x, y) = O no plano Cartesiano
é no máximo mn. Para uma exposição elementar, fato de ser d' o menor inteiro positivo pertencente a S. Logo, r = O e
sugerimos [11]. d' 1 a1. Analogamente, d' 1 a2, ... , ªn·
Para terminar, como d' é um divisor comum de a 1 , a2, ... , an, para
mostrarmos que d' = d basta que seja d' 2: d. Mas, se a 1 = dq 1,
Teorema 1.13 (Bézout). Sejam a 1 , a2, ... , an inteiros não nulos da- a2 = dq2, ... , an = dqn, com q1, q2, ... , qn E Z, então
dos. Se n d' a1U1 + a2U2 + · · · + anUn
S = {Laixi; xi E Z, \fl :Si :S n}, dq1 U1 + dq2U2 + · · · + dqn Un
i=l
d(q1U1 + q2U2 + · · · + qnun),
então S = dZ, onde d= mdc (a 1 , a2, ... , an)· Em particular, existem
números inteiros u 1 , ... , Un tais que

(1.4)
ou seja, O< d I d'. Logo, d s d'.
Colecionamos, a seguir, vários corolários úteis do teorema acima.

16 Divisibilidade 1.2 MDC e MMC 17

Corolário 1.14. Sejam a 1, a 2, ... , an inteiros não nulos e d seu mdc. Observação 1.17. Se a1, a2, ... , an são inteiros não nulos com mdc
Se d' EN, então d' 1 a 1, a2, ... , an se, e só se, d' 1 d. igual a d, o item (b) do corolário anterior garante que, fazendo ui = 5;f
para 1 ::; i ::; n, temos ui, u2, ... , Un primos entre si e a 1 = du 1 ,
Prova. Tome inteiros u1, u2, ... , Un tais que d = a1 u1 + a2u2 + · · · +
ª2 = du2, ... , an = dun.
anun. Uma vez que d' 1 a 1, a 2, ... , an, o item i. das observações 1.6
garante que d' 1 d. A recíproca é imediata. • A forma de escrever inteiros não nulos a 1, a 2, ... , an, como des-
crito na observação acima, é útil na análise de várias situações, como
Corolário 1.15. Sejam a 1, a 2, ... , an inteiros não nulos dados e d seu atestam os exemplos a seguir.
mdc.
Exemplo 1.18. Todo racional não nulo admite uma representação em
(a) d = 1 se, e só se, existirem inteiros u 1, u2, ... , Un tais que a1 u1 + fração irredutível, i.e., uma fração da forma i,
com a e b inteiros
a2U2 + · · · + anUn = 1. não nulos e primos entre si. De fato, seja r um racional não nulo e
r = ~ uma representação fracionária de r. Então, m e n são inteiros
não nulos e, sendo d = mdc (m, n), m = da e n = db, segue do item
(b) do corolário 1.15 que mdc (a, b) = 1. Por outro lado,
Prova.
(a) Se d= 1, a existência de inteiros u 1 , u 2, ... , Un como pede o enunci- m da a
r=-=-=-
ado segue do teorema de Bézout. Reciprocamente, sejam u 1 , u 2, ... , Un n db b'
inteiros como no enunciado. Como d I a 1 , a 2, ... , an, segue novamente o que nos dá a representação desejada de r em fração irredutível.
do item i. das observações 1.6 que d 1 (a1u1 + · · · + anun), i.e., d l 1.
Exemplo 1.19 (Rússia). Sejam a, b naturais tais que ª! 1 + b!l EN.
Logo, d= 1. Prove que
mdc(a,b)::; v'a+b.
(b) Sendo d= a1u1+a2u2+· · +anun, temos (1) u1+· · +(ª;) Un = 1,
e o resultado segue imediatamente do item (a). • Prova. Sendo d = mdc (a, b), existem inteiros u, v tais que a = du,
b = dv e mdc (u, v) = 1. Então
Exemplo 1.16 (China). Sejam a, b, e, d inteiros não nulos, tais que
a+1 b+ 1 du + 1 dv + 1
e + d # O e ad - be = 1. P rove que a f raçao + db e, 1rre
- ªe + · du t'1ve1. --+-- =
b a dv
+- du
-
u(du + 1) + v(dv + 1)
Prova. Queremos provar que mdc ( a + b, e + d) = 1. Para tanto, duv
procuremos, de acordo com o corolário acima, inteiros u, v tais que d( u 2 + v 2 ) + (u + v)
duv EN,
(a + b)u + (e + d)v = 1. de modo que

Ora, uma vez que ad - bc = 1, basta tomarmos u =de v = -b.


• UV·
d( u 2 + v 2) + (u
duv
+ v)
=
( 2
U +v
2) u+v
+--
d
18 Divisibilidade 1.2 MDC e MMC 19

também é natural. Portanto, d 1 ( u + v) e, daí, d ::; u + v. Mas isso é Proposição 1.21. Para a, b e e inteiros não nulos, temos que:
o mesmo que
(a) Se e I ab e mdc (b, e) = 1, então e I a.
d2 < du + dv = a + b.
(b) Se a+ bc # O, então mdc (a+ bc, b) = mdc (a, b) .
• (c) Se mdc (a, e)= 1, então mdc (a, bc) = mdc (a, b).
Corolário 1.20. Para a 1 , ... , an, k inteiros não nulos, temos:
(d) Se e I b e mdc (a, b) = 1, então mdc (a, e)= 1.
(a) mdc (ka1, ... , kan) = lkl mdc (a1, ... , an)·
(e) Se mdc (b, e)= 1, então mdc (a, bc) = mdc (a, b) mdc (a, e). Em
(b) mdc (a1, ... , an) = mdc ( mdc (a1, ... , ªn-1), an)·
particular, se b e e são primos entre si e dividem a, então bc
Prova. divide a.
(a) Denotemos d= mdc (a 1, ... , an) e d'= mdc (ka 1, ... , kan)· Como
Prova.
d I a1, ... , an, temos que lkld I ka1, ... , kan, i.e., lkld é um. divisor
comum positivo de ka 1, ... , kan. Mas, como d' é o maior dentre tais (a) Sejam u, v inteiros tais que bu + cv = 1. Multiplicando ambos os
divisores comuns, segue que lkld::; d'. Reciprocamente, como k divide membros dessa igualdade por a, obtemos (ab )u + e( av) = a. Por fim,
d' e d' divide ka 1, ... , kan, temos que é um inteiro positivo que di-
{
como e 1 ( ab), segue do item (c) da proposição 1.5 que e I a.
1 1

v1ºd e a1, ... , an, d e maneira


. que TkT
d' ::; d ou, o que e
, o mesmo, dI ::; 1k 1d.
(b) Sejam d= mdc (a+ bc, b) e d'= mdc (a, b). Como d' a, b, temos
Logo, d'= lkld. 1

que d' a, a+ bc. Portanto, pelo corolário 1.14 temos que d' d. Reci-
1 1

procamente, como d 1 (a+ bc) e d I b, temos que d 1 [ (a+ bc) - bel, i.e.,
(b) Exercício.
d I a e d I b. Novamente pelo corolário 1.14, temos que d I d' e, assim,
• d= d'.
Especializemos nossa discussão ao máximo divisor comum de dois
inteiros não nulos. Dados a, b inteiros não nulos, com d= mdc (a, b), (c) Sejam d = mdc (a, b) e d' = mdc (a, bc). De d I b, segue que
o teorema de Bézout garante a existência de inteiros u e v tais que d = d I bc. Assim, d I a e d I bc, de sorte que d I mdc (a, bc) = d'. Para
au + bv. É importante notar que tal maneira de escrever o mdc não é terminar, mostremos que d' 1 d: como mdc (a, e) = 1, segue do teo-
única; de fato, se t E Z, então temos também d = a( u - tb) + b( v +ta) rema de Bézout a existência de u, v E Z tais que au + cv = 1 e, daí,
(teremos mais a dizer sobre isso na proposição 1.25). a(bu) + (bc)v = b; mas, como d' a e d' bc, temos que d' b. Então,
1 1 1

Mais adiante, estabeleceremos um algoritmo muito útil e impor- d' a e d' b, de modo que d' mdc (a, b) = d.
1 1 1

tante para encontrar efetivamente o mdc de dois inteiros, o algoritmo


de Euclides. Comecemos estudando algumas propriedades do mdc de (d) Sejam d E Z tal que b = cd e u, v E Z tais que au + bv = 1.
dois inteiros não nulos. Então, au + c(dv) = 1 e segue, do item (a) do corolário 1.15, que
~
1' 1 :, 1!
1

' 1

1 :

mdc (a, e)= 1. e, por indução sobrem, obtemos mdc(am,bn) = mdc(a,bn) = 1.


Alternativamente, uma vez que mdc (a, b) = 1, o teorema de Bé-
(e) Sejam d = mdc (a, b) e a = du, b = dv, com u e v inteiros pri- zout garante a existência de x, y E Z tais que ax + by = 1. Portanto,
mos entre si. Aplicando sucessivamente o corolário 1.22 e o item (c), segue da fórmula de expansão binomial que
obtemos

mdc(a,bc) = mdc(du,dvc) = dmdc(u,vc) = dmdc(u,c).


1 = (ax + by)" = ~ (;) (ax)"-k(by)' + (by)" = aq + bnyn,

Mas, d I b e mdc (b, e) = 1 implicam (cf. item (d)) mdc (d, e) = 1. onde q = I:~:~ (~)an-k-lxn-k(by)k. Portanto, segue do item (a) do
Portanto, mais uma aplicação do item (c) nos dá corolário 1.15 que mdc (a, bn) = 1. Argumentando de maneira aná-
loga, concluímos que mdc (am, bn) = 1.
mdc (a, e)= mdc (du, e)= mdc (u, e).

Finalmente, juntando as duas relações acima, obtemos (b) Sejam u = mdc (a, k) e v = mdc (b, k). Como mdc (a, b) = 1, o
item (d) da proposição anterior garante que
mdc (a, bc) =d· mdc (u, e)= mdc (a, b) mdc (a, e).
k = mdc (kn, k) = mdc (ab, k) = mdc (a, k) · mdc (b, k) = uv,
O resto é imediato.
• de modo que
Corolário 1.22. Sejam a e b inteiros não nulos e k, m e n números
naturais.
Agora, u I a e mdc (a, b) = 1 nos dão (cf. item (d) da proposição
(a) Se mdc (a, b) = 1, então mdc (am, bn) = 1. anterior) mdc (u, b) = 1; analogamente, mdc (v, a) = 1. Segue, pois,
do item (a) que
(b) Se mdc (a, b) = 1 e ab = kn, então existem u, v E Z tais que
a=un,b=vn. mdc (un, b) = 1 e mdc (vn, a)= 1.
Prova. Por fim,
(a) Aplicando o item (c) da proposição anterior, com bn-l no lugar de
b e b no lugar de e, obtemos un I ab e mdc (un, b) = 1 =} un Ia=} un ~ a
{
vn I ab e mdc (vn, a) = 1 =} vn I b =} vn ~ b ·
mdc (a, bn) = mdc (a, bn-l · b) = mdc (a, bn- 1 ).
Mas, como ab = unvn, un ~ a e vn ~ b, a única possibilidade é que
Segue por indução sobre n que mdc (a, bn) = mdc (a, b) = 1. Analo- sejam a= une b = vn. •
gamente,
Exemplo 1.23. Dados números naturais n e k, com k > 1, ou existe
m E N tal que n = mk ou {ln é um número irracional.
22 Divisibilidade 1.2 MDC e MMC 23

Prova. Se ijn = 7;, com m e p naturais primos entre si (cf. exemplo não há dois cubos perfeitos cuja diferença seja igual a 4, chegamos a
1.18), então pkn = mk. Mas, como mdc (m,p) = 1, segue do corolário uma contradição. •
acima que mdc (mk,pk) = 1. Por outro lado, pkn = mk garante que
Em que pese o caráter ad hoc do exemplo acima, a teoria desen-
pk I mk. Assim, a única maneira de pk e mk serem relativamente
volvida até agora para o estudo do mdc de dois inteiros permite re-
primos é que seja pk = 1. Mas, como k > 1, devemos ter, então,
solver completamente a equação diofantina linear a duas variáveis
p= 1, i.e., n=mk. •
ax + by = e, onde x e y são incógnitas inteiras e a, b e e parâmetros
V ma equação em números inteiros e com mais de uma variável inteiros não nulos dados, conforme ensina o resultado a seguir.
é denominada uma equação Diofantina, em homenagem ao mate-
Proposição 1.25. Sejam a, b e e inteiros não nulos dados. A equação
mático grego Diofanto de Alexandria 1 , quem primeiro tentou estudar
ax + by = e admite soluções x, y E Z se, e só se, mdc (a, b) 1 e. Nesse
sistematicamente as soluções de algumas dessas equações. A análise
caso, se d = mdc (a, b) e x = x 0 , y = y0 for uma solução inteira
de equações Diofantinas gerais é uma tarefa das mais difíceis, exigindo
qualquer da equação, então as fórmulas
em geral argumentos bastante sofisticados. Vejamos um exemplo sim-
ples. b a
x = Xo + dt, Y = Yo - dt, (1.5)
Exemplo 1.24. Prove que não existem x, y E N tais que x 3 +3 =
t E Z, dão todas as soluções inteiras possíveis. Em particular, pode-
4y(y + 1).
mos supor que seja x >O> y ou, ainda, x <O< y.
Prova. Suponha o contrário, i.e., que x3 +3 = 4y(y + 1), para certos
Prova. Suponha, inicialmente, que existam inteiros x e y tais que
x, y E N. Então
ax + by = e. Como d Ia e d I b, segue que d 1(ax + by), ou seja, d I e.
x3 + 4 = 4y(y + 1) + 1 = (2y + 1) 2 Reciprocamente, seja e = de, com e E Z, e (pelo teorema de Bézout)
u e v inteiros tais que d = au + bv. Então e = a · eu + b · ev, quer dizer,
e, daí, a equação admite a solução inteira x 0 = eu, y0 = ev.
x 3 = (2y + 1) 2 - 22 = (2y - 1)(2y + 3). Para o que falta, suponha que d I e e seja x = x 0 , y = y0 uma
Agora, se d= mdc (2y-1, 2y + 3), então d é ímpar e divide (2y + 3) - solução inteira qualquer da equação. Se x = x 1 , y = y 1 for outra
(2y - 1) = 4, de maneira que d= 1. Portanto, o produto dos inteiros solução inteira da mesma, teremos a(x 1 - x 0 ) = b(y0 -y1 ); cancelando
primos entre si 2y -1 e 2y + 3 é um cubo perfeito, e o corolário 1.22 d em ambos os membros dessa igualdade, segue que
garante que ambos 2y-1 e 2y+3 devem ser cubos perfeitos. Mas, como
1 Matemáticogrego do século III d.C., considerado um dos pais da Álgebra e da
Teoria dos Números. Em seu livro Aritmética, Diofanto procurou, pela primeira
vez, estudar sistematicamente as soluções inteiras de certos tipos particulares de Assim, ~ 1 ~(x 1 -x 0 ) e, como mdc (~, ~) = 1, o item (a) da proposição
equações polinomiais - de fato, essa nomenclatura não estava disponível à sua anterior garante que ~ 1 (x 1 - x 0 ). Sendo x 1 - x 0 = ~t, obtemos
época-, as quais passaram, merecidamente, a ser conhecidas como Diofantinas. y0 - y 1 = ~t e as fórmulas do enunciado seguem. Reciprocamente, é
imediato verificar que tais fórmulas dão, de fato, soluções inteiras para Proposição 1.26. Nas notações da tabela acima para o algorimo de
a equaçao. Euclides, temos mdc (a, b) = rj.
Para o que falta, suponha que a, b > O (os demais casos podem ser
analisados de modo análogo). Como Prova. Pelo item (b) da proposição 1.21, temos sucessivamente
U V
u - tb > oB t < b e V + ta < o B t < --;;, ' mdc (a, b) mdc (a - bq 1, b) = mdc (r 1 , b)
mdc (r1, b - r 1q2) = mdc (r 1, r 2)
escolhendo um inteiro t tal que t < E", - ~ e fazendo u 1 = u - tb e
mdc (r1 - r2q3, r2) = mdc (r3, r2)
v1 = v + ta, teremos u 1 > O, v1 < O e d = au 1 + bv 1. Analogamente,
mostramos que podemos tomar uma solução x < O < y da equação
dada. •
Por enquanto, sabemos apenas que é possível escrever o mdc de
dois inteiros não nulos a e b da forma au + bv, para algum par de
onde utilizamos, na última igualdade, o fato de que rj I rj-l·

inteiros u, v, sem termos ainda uma maneira razoável para fazer isso Embutido no algoritmo de Euclides também está um método para
efetivamente. Para remediar essa situação, consideremos o seguinte encontrarmos os inteiros u, v cuja existência é garantida pelo teorema
algoritmo, usualmente atribuído a Euclides. de Bézout, i.e., tais que mdc (a, b) = au + bv. Vejamos como proceder
através de um exemplo numérico.
Algoritmo de Euclides
Exemplo 1.27. Utilize o algoritmo de Euclides para mostrar que
o mdc de 120 e 84 é igual a 12. Em seguida, resolva a equação
Passo 1 a bq1 + r1 O< r 1 < b 120x + 84y = 12 no conjunto dos inteiros.
Passo 2 b r1q2 + r2 O< r2 < r1
Solução. Comecemos executando o algoritmo de Euclides:
Passo 3 r1 r2q3 + r3 O< r3 < r2
120 84 · 1 + 36
Passo j rj-2 rj-lqj + rj O< rj < rj-1 84 36 · 2 + 12
Passo j +1 rj-1 rjqj+1 +O 36 12 · 3.
Note que a execução do algoritmo realmente para após um número
Como 12 é o último resto não nulo, segue da proposição 1.26 que
finito de passos, pois, desde que r 1 , r 2 , ... são inteiros para os quais
b > r 1 > r 2 > · · · ;:::: O, deve existir um menor natural j tal que rj é o mdc (120, 84) = 12.
último resto não nulo no processo de divisões acima.
A importância do algoritmo de Euclides é explicada pela proposi- Podemos, agora, encontrar inteiros u e v tais que 12 = 120u + 84v
ção a seguir. trabalhando de trás para frente com as divisões sucessivas que nos
"1 •..
1

"
26 Divisibilidade f 1.2 MDC e MMC 27
I
·. '· · .

levaram ao mdc : Sendo


12 84 · 1- 36 · 2
84 · 1 - (120 - 84 · 1) · 2
{~ d'
(bq-l + bq- 2 + · · · + b + l)ar
mdc (am - 1, an - 1)
mdc (an - 1, ar - 1)
84 · 1 - 120 · 2 + 84 · 2 temos
84 · 3 + 120(-2). am - 1 = (an - l)c + (ar - 1).
Portanto, segue do item (b) da proposição 1.21 que
Por fim, de posse da solução x 0 = -2 e y0 = 3 da equação 120x +
84y = 12, podemos obter todas as soluções dessa equação utilizando mdc (am - 1, an - 1) mdc ((an - l)c + (ar - 1), an - 1)
as fórmulas (1.5): mdc (ar - 1, an - 1).

X= -2 + (84/12)t = -2 + 7t Para o que falta suponhamos, sem perda de generalidade, que


{ m 2: n. Se m = n, nada há a fazer. Suponhamos, pois, m > n e
y= 3 - (120/12)t = 3 - lOt
consideremos o algoritmo de Euclides para m e n:
• m
n
nq1 + r1
r1q2 + r2
O< r1 < n;
O< r2 < r1;
O algoritmo de Euclides e sua prova também são úteis para muitos
propósitos teóricos, conforme atesta o próximo exemplo. r1 r2q3 + r3 O< r3 < r2;

Exemplo 1.28. Se a, m e n são naturais tais que a > 1 e m = nq + r, T'j-2 = T'j-lqj+ T'j
com O ::::; r < n, prove que rj-1 = ríqí+1 + O.

mdc (am - 1, an - 1) = amdc(m,n) - 1. Nossa discussão anterior garante que

mdc (m, n) = mdc (n, r 1) = mdc (r 1, r 2) = · · · = mdc (rj-l, rj) = rj.


Prova. Mostremos inicialmente que, ser= O, então (an-1) 1 (am-1).
Para tanto, basta observar que am - 1 = (an)q - 1 e lembrar que já Portanto, aplicando a discussão acima sucessivas vezes, concluímos
sabemos, pelo exemplo 1.3, que an - 1 divide (an)q - 1. que
Provemos agora que, se r > O, então mdc (am - 1, an - 1) = mdc(am -1,an -1) mdc (an - 1, ari - 1)
mdc (an - 1, ar - 1). De fato, fazendo an = b quando conveniente,
mdc (ar 1 - 1, ar 2 - 1)
temos

anq+r - 1 = (anq - l)ar + (ar - 1) mdc (ari- 1 - 1, arj - 1)


((an)q - l)ar + (ar - 1) arj _ 1 = a mdc (m,n)-1 _
l)(bq-l + · · · + b + l)ar + (ar
(an - - 1).

r
r

28 Divisibilidade 1.2 MDC e MMC 29

Finalizamos esta seção estudando o minimo múltiplo comum de Proposição 1.32. Se a e b são inteiros não nulos, então
um conjunto finito de inteiros não nulos. Dados inteiros não nulos
mmc (a, b) · mdc (a, b) = labl.
a1, a2, ... , an, o inteiro positivo la 1a2 ... anl é um múltiplo comum de -
a 1 , a 2, ... , an· Existe, portanto, um menor inteiro positivo que é múl-
Prova. Mostremos primeiro que mdc (a, b) = 1 ==} mmc (a, b) = labl.
tiplo comum de a 1 , a 2, ... , an, o que dá sentido à definição a seguir.
Sejam um múltiplo positivo comum de a e b. Como mdc (a, b) = 1, o
Definição 1.29. Dados inteiros não nulos a1, a2, ... , an, o mínimo item (e) da proposição 1.21 garante que ab I m. Portanto, m 2". labl,
múltiplo comum de a1, a 2, ... , an, denotado mmc (a1, a2, ... , an), é de forma que labl é o menor múltiplo positivo comum de a e b, ou seja,
o menor dentre todos os múltiplos positivos comuns de a1, a2, ... , ªn· labl = mmc (a, b).
Para o caso geral, note primeiro que mmc (a, -b) = mmc (a, b);
Os resultados a seguir estabelecem as propriedades básicas do mmc . como já temos mdc (a, -b) = mdc (a, b), podemos supor, sem perda
de generalidade, que a, b > O. Sejam d = mdc (a, b) e u, v intei-
Proposição 1.30. Sejam a1, a2, ... , an E Z* em= mmc (a1, a2, ... , ros primos entre si e tais que a = du, b = dv. Queremos mos-
an)· Para todo inteiro M, temos que M é um múltiplo comum de a 1 , trar que mmc (du, dv )d = d 2uv ou, ainda, (e pelo lema anterior) que
a 2, ... , an se, e só se, m I M. mmc (u, v) = uv. Mas isso é exatamente o conteúdo da primeira parte
aCTma. •
Prova. Seja M um múltiplo comum de a1, a2, ... , an, e escreva M =
mq + r, com q, r E Z e O ::::; r < m. Como a1 1 m e a1 1 M, segue Exemplo 1.33 (Japão). Encontre todos os pares (a, b) de inteiros
da proposição 1.5 que a1 1 (M - mq), i.e., a1 1 r; analogamente, positivos tais que a 2". b e
a 2, ... , an Ir, i.e., O::::; r < m é um múltiplo comum de a1, a2, ... , ªn·
Mas, como m é o menor múltiplo comum positivo de a1, a2, ... , an, a mmc(a,b)+ mdc(a,b)+a+b=ab.
única possibilidade é termos r = O, de maneira quem I M. •
Solução. Sejam d= mdc (a, b) eu, v E N tais que a = du, b = dv.
Lema 1.31. Sejam a 1 , a 2, ... , an inteiros não nulos. Se k EN, então Então u 2". v e
mmc (ka 1, ka 2, ... , kan) = k mmc (a1, a2, ... , an)·
d· mmc (a, b) = mdc (a, b) · mmc (a, b) = ab = d 2uv,
Prova. Sejam M = mmc (ka 1, ka 2, ... , kan) em= mmc (a1, a2, ... ,
an)· Então km é múltiplo de ka 1, ka2, ... , kan, de modo que km 2". M. de maneira que mmc (a, b) = duv. Substituindo as expressões acima
Por outro lado, como M é um múltiplo comum de ka1, ka2, ... , kan, na equação original, concluímos que a mesma equivale a
segue que M/k é um múltiplo comum de a 1 , a2, ... , an e, daí, M/k 2". m
ou, ainda, M 2". km. Logo, M = km. • u(v + 1) + (v + 1) = duv, (1.6)

A proposição a seguir relaciona o mdc e o mmc de dois inteiros de maneira que u 1 (v+l). Assim, u::::; v+l e, daí, u =vou u = v+l.
não nulos. Seu= v, segue de mdc(u,v) = 1 que u = v = 1, de sorte que d= 4
ml!rl
'1ffll!',,·11}i . .1
1,'

il ,11

li
11

+ 1, segue de
1

e a= b = 4. Seu= v (1.6) que Problemas - Seção 1.2

u(v+l)+(v+l) (v+l)2+(v+l) 1. (IMO.) Para n EN, prove que mdc (2ln + 4, 14n + 3) = 1.
d
UV (V+ l)v 2. (OIM.) Sejam me n inteiros positivos. Se 2m +1= n 2 , prove
v + 3v + 2 = 1 + ~
2
quem= n = 3.
v2 + v v'
3. Considere duas progressões aritméticas infinitas e não constan-
o que implica v = 1 ou v = 2. Examinando cada caso separadamente,
tes, cujos termos são inteiros positivos. Prove que existem in-
chegamos às demais soluções: a = 6, b = 3 ou a = 6, b = 4. •
finitos naturais que são termos de ambas as sequências se, e só
se, o mdc de suas razões dividir a diferença entre seus termos
iniciais.

4. * Dados a, m, n EN, com m-/=- n, prove que mdc (a 2n + 1, a 2= +


1) = 1 ou 2.

5. (Japão.) Prove que mdc (n! + 1, (n + l)! + 1) 1, para todo


n EN.

6. (Torneio das Cidades.) Se a e b são números naturais tais que


ab (a 2 + b2 ), mostre que a= b.
1

7. (OBM.) Dados a, b E N, particione um retângulo a x b em ab


quadradinhos 1 x 1. Prove que cada diagonal do retângulo passa
pelo interior de exatamente a+ b- mdc (a, b) quadradinhos 1 x 1.

8. Sejam n e k inteiros positivos, com n 2'.: k. Prove que o mdc


dos números
( n)
k '
(n +
k
1) ' ... ' (n +k k)
é igual a 1.

9. Seja n > 1 um inteiro tal que 2k é a maior potência de 2 que


divide n. Prove que o máximo divisor comum dos números bi-
no miais
r---
.
l!i... ·.i I
Pfl
1: 1
'
l 1 1

i'

é igual a 2H 1 . 17. (Bulgária.) Seja (an)n:ê".l a sequência de inteiros positivos defi-


nida por a1 = 2 e, para todo n EN, ªn+l = a~ - an + 1. Mostre
1

10. (Estados Unidos - adaptado.) Fixado k EN, mostre que que dois termos quaisquer dessa sequência são primos entre si.
max{ mdc (n 2 + k, (n + 1) 2 + k); n EN}= 4k + 1.
18. * Seja (Fn)n2:1 a sequência de Fibonacci, i.e., a sequência tal que
F1 = F2 = 1 e Fk+2 = Fk+l + Fk, para todo k E N. Nosso
11. Se a, b e e são inteiros positivos tais que + = i t i,
prove que
objetivo neste problema é provar que mdc (Fm, Fn) = Fmdc(m,n)·
existem inteiros positivos q, u e v tais que mdc (u, v) = 1 e
Para tanto, faça os seguintes itens:
a = qu( u + v), b = qv (u + v) e e = quv.

12. (Inglaterra.) Sejam x, y naturais tais que 2xy divide x 2 + y 2 - x. (a) Prove que, para todo n EN, tem-se mdc (Fn, Fn+l) = 1.
Prove que x é um quadrado perfeito. (b) Prove que, para todos n, k EN, com n > 1, tem-se Fn+k =
Fn-lFk + FnFk+l·
13. (Estados Unidos.) De qualquer conjunto de 10 naturais conse-
cutivos, prove que é sempre possível escolher ao menos um que (c) Use o item (b) para concluir que, para todos n, q, r E N,
seja relativamente primo com os nove restantes. (i) mdc (Fnq, Fn) = Fn.
mdc(y, z) = 1. (ii) mdc (Fnq-l, Fn) = 1.
(iii) m dc (Fnq+r, Fn) = mdc (Fr, Fn)·
14. Sejam a, bem naturais dados, com mdc (a, m) = 1. Mostre que

m-1
~
l .+ bj
ªJ m
1
= 2(a - l)(m - 1) + b.
(d) Conclua que mdc (Fn, Fm) = Fmdc(m,n) para todos m e n
naturais.

19. (Croácia.) Seja (an)n2:1 uma sequência de números inteiros tal


Para o problema a seguir, recorde (cf. problema 1.1.7 do volume que a1 = 1 e ªn+2 = ª2ªn+1 + an, para todo n EN. Prove que
3) que, dado x E IR, sua parte fracionária é o real {x}, tal que mdc (am, an) = ªmdc(m,n)·
{x} = x - lxJ. Assim, {x} E [O, 1) e {x} =O{:} x E Z.
20. Dados a, b EN primos entre si, prove os seguintes itens:
15. (Japão.) Sejam n, r E N tais que n > 1 e n f r. Se g
(a) Todo natural n > ab pode ser escrito da forma n = ax + by,
mdc (n, r), prove que
com x, y naturais.
(b) O inteiro ab não pode ser escrito como em (a).
(c) Todo inteiro n > ab - a - b pode ser escrito da forma
n = ax + by, com x, y inteiros não negativos.
16. (Putnam.) Dados m, n E N, com m 2:: n, prove que o número
mdclm,n) (7:) é um natural. (d) O inteiro ab - a - b não pode ser escrito como em (c).
34 Divisibilidade 1.3 Números primos 35

(e) Há exatamente Ha - l)(b - 1) inteiros não negativos que Informações sobre essa questão nos são dadas por um teorema
não podem ser escritos da forma ax+by, com x e y também devido ao matemático francês Gabriel Lamé, e o objetivo deste
inteiros não negativos. problema é prová-lo. Para tanto, faça os seguintes itens:

21. (IMO - adaptado.) Sejam a, b, e naturais dois a dois primos entre (a) Se Fk é o k-ésimo número de Fibonacci e n EN, prove que
Sl. Fn+5 > lOFn e deduza, a partir daí, que Fsn+ 2 > 10n, de
modo que Fsn+2 tem pelo menos n + 1 algarismos em sua
(a) Mostre que não existem x, y, z E Z+ tais que 2abc - ab - representação decimal.
bc - ca = xbc + yac + zab.
(b) Se n passos forem usados no algoritmo de Euclides para
(b) Se n E N é tal que n > abc - a - bc, use o resultado do calcular mdc (a, b), com a > b > O e usando b como o
problema anterior para concluir pela existência de x, t E Z+ primeiro divisor, prove que b ~ Fn- l ·
tais que n = xbc + ta, com O :::; x :::; a - 1.
(c) Prove o teorema de Lamé: o número de divisões necessá-
(c) Mostre que o inteiro t do item (b) é maior que bc-b-c, e use rias, no algoritmo de Euclides, para calcularmos o mdc de
novamente o resultado do problema ;mterior pará concluir dois inteiros positivos é, no máximo, cinco vezes o número
pela existência de y, z E Z+ tais que t = bz + cy. de algarismos da representação decimal do menor dos dois
(d) Se n E N é tal que n > 2abc - ab - bc - ca, use os itens números.
(b) e (c) para mostrar que existem x, y, z E Z+ tais que
24. (OBM.) Seja n > 1 inteiro. Prove que o número
n = xbc + yac + zab.
1 1 1
22. Generalize o resultado do problema anterior do seguinte modo: 1+-+-+···+-
2 3 n
! 1
se a1, a 2 , ... , an são naturais dois a dois primos entre si e nunca é inteiro.

1.3 Números primos


(âJ indica que ªJ não está presente no produto Xja1 · · · âj · · · an), Um inteiro p > 1 é primo se seus únicos divisores positivos forem 1
então o maior natural que não pertence a A é o número e p. Um inteiro a> 1 que não é primo é dito composto. Provaremos
logo mais (cf. teorema 1.37) que o conjunto dos números primos é in-
finito; por ora, para comodidade do leitor, listamos abaixo os números
primos menores que 50, cujas primalidades podem ser verificadas sem
dificuldade:
23. É interessante nos perguntarmos sobre a eficiência do algoritmo
de Euclides para o cálculo do mdc de dois inteiros não nulos. 2,3,5, 7,ll,13,17,19,23,29,31,37,41,43,47.
36 Divisibilidade 1.3 Números primos 37

Nosso objetivo nesta seção é estudar os números primos e sua relação Prova. Inicialmente, note que 25 < v641 < 26. Portanto, se 641 for
com os números compostos. Começamos com o seguinte resultado composto, segue do corolário acima que 641 deve possuir um divisor
auxiliar, conhecido como o lema de Euclides. primo p:::; 25, de modo que

Lema 1.34 (Euclides). Todo inteiro n > 1 pode ser expresso como o
p E {2,3,5, 7, 11,13, 17, 19,23}.
produto de um número finito de primos, não necessariamente distin-
tos2. No entanto, é imediato verificar que, dentre as divisões de 641 pelos
Prova. Façamos a prova por indução sobre n. Se n = 2, nada há a primos acima, nenhuma é exata. Logo, 641 é primo. •
fazer (uma vez que 2 é primo). Suponha, agora, que todo inteiro n
A utilização do corolário 1.35 para decidir se um certo natural é
tal que 2 :::; n < m pode ser escrito como o produto de um número
primo (como no exemplo acima) é conhecida como o crivo de Eratós-
finito de primos; provemos que este é também o caso para m: se m
tenes. De maneira geral, basta dividir o natural em questão por todos
for primo, nada há a fazer. Senão, existem inteiros a e b tais que
os primos menores ou iguais que sua raiz quadrada por falta; de acordo
m = ab, com 1 < a, b < m. Pela hipótese de indução, a e b podem
com o corolário, se nenhuma dessas divisões for exata, o número será
ser escritos como produtos de números finitos de primos, digamos a =
primo. Mais que interesse prático, contudo, a maior utilidade do crivo
p 1 ... Pk, b = q1 · · · qz, com k, l 2:: 1 e p 1, ... , Pk, q1, ... , qz primos. Logo,
de Eratóstenes é teórica, uma vez que, para aplicá-lo para decidir se
m = ab = p 1 ... pkq1 · · · qz, também o produto de um número finito de
primos.
• 999997 é primo, por exemplo, teríamos de dividi-lo por todos os primos
menores ou iguais a J999997 "'"' 1000, o que não é razoável 3 .
Como corolário do lema acima, temos o seguinte critério de pes- De volta ao lema 1.34, reunindo primos iguais concluímos que é
quisa de divisores primos de um número composto, devido ao mate- possível escrever todo inteiro n > 1 na forma n = pf 1 . . . p~k, onde
mático grego Eratóstenes de Cirene (cf. figura 1. 2). p 1 , ... , Pk são primos dois a dois distintos e a: 1 , ... , ak E N. Veremos
no teorema 1.42 que, a menos de uma reordenação de fatores, essa
Corolário 1.35 (Eratóstenes). Se um inteiro n > 1 for composto,
maneira de escrever n é única. Antes, contudo, provemos que o con-
então n possui um divisor primo p, tal que p :S: Jri.
junto dos números primos é infinito, resultado este também devido a
Prova. Seja n = ab, com 1 < a:::; b. Sendo pum divisor primo de a, Euclides.
segue que p I n e
Teorema 1.37 (Euclides). O conjunto dos números primos é infinito.
p 2 :::; a 2 :::; ab = n,

de modo que p :S: fo. • 3 Com um pouco mais de trabalho, é possível mostrar que o crivo de Eratóstenes
é um algoritmo que, genericamente, não termina em tempo polinomial. De outra
Exemplo 1.36. Use o corolário acima para provar que 641 é primo. forma, isto significa que a quantidade de operações que necessitamos executar
para encontrar o n-ésimo primo com o auxílio do crivo de Eratóstenes cresce
2 Conforme estabelecido na definição 4.21 do volume 1, identificamos o produto exponencialmente com n, de forma que mesmo com o auxílio de um computador
Ili=l ai com a1. esse não é um bom algoritmo de primalidade.

''
, ,
1,
.,,.
l!j ·; !1 •

i .. i1 1 .•

' : 1

-=-3=8_______________________.c:cD::..:ic..:.v. :::is""ib""1""'·1i~·d"'a~d~e r 3 Números primos 39



1 A utilização de pequenas variações do argumento usado na prova
I do teorema de Euclides nos permite mostrar que várias PA's contêm
r uma infinitude de números primos. Vejamos um exemplo nesse sen-
tido4.

Exemplo 1.39. Prove que há infinitos primos da forma 4k - 1.

Figura 1.2: Eratóstenes de Cirene, matemático Prova. Suponha que só houvesse uma quantidade finita de primos da
grego do século II a.C. Além de seu algoritmo de pri- forma 4k - 1, digamos PI = 3,p2 = 7,p3 = 11, ... ,Pt, e considere o
malidade, outro trabalho notável de Eratóstenes foi 1 número
f
conseguir medir indiretamente o diâmetro da Terra, t
m = 4PIP2 · · ·Pt - 1.
com precisão impressionante para a época.
1
(
Claramente, m > 1 e, sendo m' = PIP2 .. ·Pt, temos m = 4m' - 1.
Por outro lado, o lema 1.34 garante a existência de primos ímpares
l
Prova. Por indução sobre n, provemos que, se N contiver n pri- J qI, ... , qz tais quem= qI ... qz. Observe, agora, que todo primo ímpar
,,
mos distintos, então N conterá n + 1 primos distintos. Suponha que r é da forma 4q' - 1 ou 4q' + 1, para algum q' E Z. Se fosse qi = 4qf + 1 .
~:

~ para 1 :::; i :::; t, teríamos


PI, ... , Pn são primos distintos, e seja t

i;
m =PI·· ·Pn + 1. f m = (4q~ + 1) ... (4qf + 1) = 4q + 1,
~.
i;i;·

E
Pelo lema 1.34, existe um primo p tal que p I m. Se p = Pi para algum ~ 1ii· para algum q E N, contradizendo o fato de ser m = 4m' - 1. Portanto,
ti

1 :::; i :::; n, então p I PI ... Pn e segue da proposição 1.5 que p divide "é:' existe 1 :::; i :::; l tal que qi = 4qf - 1. Finalmente, como PI,P2, ... ,Pt
a diferença m - PI ... Pn, i.e., p 1 1, o que é um absurdo. Logo, p é são todos os primos dessa forma, deveríamos ter qi = Pi para algum
um primo diferente de todos os p/s, de maneira que temos pelo menos 1 :::; j :::; t. Mas, como qi I m, seguiria então que Pi seria um divisor
n + 1 primos distintos em N. • do número m = 4PIP2 .. ·Pt - 1, o que é uma contradição. •

Observação 1.38. Em que pese o resultado acima, há lacunas arbi- Para a prova do teorema 1.42, precisamos do lema a seguir, o qual
trariamente grandes na sequência dos primos. De fato, dado k 2 3 apresenta interesse próprio.
inteiro, para que os inteiros positivos e consecutivos a+2, a+3, ... , a+k
4É possível provar um resultado muito mais geral, devido ao matemático alemão
sejam todos compostos basta que a seja um múltiplo comum dos nú-
do século XIX Gustav Lejeune Dirichlet, o qual afirma que toda PA infinita e não
meros 2, 3, ... , k. constante de números naturais contém infinitos números primos, contanto que seu
Teremos mais a dizer sobre a distribuição dos números primos ao primeiro termo e sua razão sejam primos entre si. Veremos um caso particular
longo dos naturais no capítulo 4. relevante desse teorema na seção 8.2 do volume 6.
40 Divisibilidade 1.3 Números primos 41

Lema 1.40. Se a1, ... , an EN e pé um primo tal que p I a1a2 ... an, 1 Teorema 1.42. Todo inteiro n > 1 pode ser escrito como o produto de
então existe 1 :::; i :::; n tal que p I ai. Em particular, se a1, ... , an '.! potências de primos distintos. Ademais, tal decomposição de n é única
forem todos primos, então existe 1 :::; i :::; n tal que p = ªi· f: no segum. t e sent"d
1 o.. se n = Pi°'1 ... Pk°'k = q1f31 ... q1f31 , ond e P1 < · · · < Pk

Prova. Façamos a prova no caso n ~


2, sendo o caBo geral totalmente
análogo. Suponha que p 1 (ab) mas p f a, e seja d= mdc (a,p). Como
I
,i
e q1 < · · · < qz são números primos e ai, /3j ~ 1 são inteiros, então
k = l e, para 1 :::; i :::; k, Pi = qi e ai = /3i-

d I p, temos d = 1 ou d = p; mas p f a e d I a garantem que d =/= p, i.e., tf Prova. A parte de existência foi estabelecida no parágrafo anterior ao
d= 1. Assim, como p 1 (ab) e mdc (a,p) = 1, segue do item (a) da
proposição 1.21 que p I b. O resto é imediato a partir da definição de
tt teorema 1.37. Para a unicidade, suponha que o inteiro n > 1 admite
duas decomposições como no enunciado. Como p 1 1 n, temos que
número primo. • ~ p 1 1 qf1 ••• qf1 , e o lema anterior garante a existência de 1 :::; j :::; l tal
que p 1 = qj. Por outro lado, como q1 1 n, temos que q1 1 pf 1 .•• p~k e,
Exemplo 1.41. Se p é um primo ímpar, então p divide cada um dos novamente pelo lema anterior, existe 1 :::; i :::; k tal que q1 = Pi· Assim,
números P1 = qj ~ q1 = Pi ~ P1, de onde segue que P1 = q1 e, daí,

Prova. Seja 1 :::; k :::; p - 1. Uma vez que o número


i Provemos agora que a1 = /31. Se a 1 < /31, então p~2 ... p~k
Pi!31 -a1 q2!32 ... q1f31 , de maneira
· que P1 1 p a2 °'k
2 •.. Pk .
M as ai,, argumen-
(p)k = p! = (k+l) ... (p-l)p tando como acima, existiria 2 :::; i :::; k tal que p 1 = Pi, o que é um
k!(p - k)! (p - k)! 1 absurdo. Analogamente, não pode ser a 1 > /31. Logo, a 1 = /31 e segue
é um inteiro, temos que (p- k) ! divide (k + 1) ... (p-1 )p. Agora, como 'ft que
°'2 °'k _ (32 f31
pé primo e p f 1, 2, ... ,p-k, o lema anterior garante que p f (p-k)!, de J P2 · · · Pk - q2 · · · qz ·
sorte que mdc (p, (p- k)!) = 1. Portanto, pelo item (a) da proposição rr Repetindo o argumento acima várias vezes (há uma indução embu-
1.21 concluímos que (p - k)! divide (k + 1) ... (p - 1) e, daí, tida aqui, a qual não formalizaremos a bem da clareza do argumento),
concluímos sucessivamente que P2 = q2 e a2 = /32, p3 = q3 e a3 = /33,
(p)
k
= (k + 1) ... (p - 1) .
(p- k)! P,
etc. Ao final, se k < l, obteremos 1 = qi~j_ 1 . • . qf1 , o que é claramente
um absurdo; se k > l, obtemos um absurdo análogo. Logo, k = l e
EN nada mais há a fazer. •
um múltiplo de p.
• Exemplo 1.43. Os naturais x e y são tais que 3x2
Prove que x - y é um quadrado perfeito.
+x = 4y 2 + y.
O teorema a seguir é a pedra fundamental da teoria elementar dos
números, sendo conhecido como o teorema fundamental da arit- Prova. Sejam p um primo e pª, pb e pc as maiores potências de p que
mética. dividem x, y ex - y, respectivamente. Suponha, por um momento,
43

que a S b. Então p 2ª 1 x 2 , y 2 , e segue do item (c) da proposição 1.5 i •


que p 2ª 1 ( 4y 2 - 3x 2 ). Mas, como x - y = 4y 2 - 3x 2 , temos então que J
Vejamos, agora, como obter as decomposições canônicas dos divi-
p 2ª 1 ( x - y) e, daí, e 2': 2a. Por outro lado, escrevendo
sores de um inteiro maior que 1 e do mdc e mmc de dois inteiros não
x2 = (x - y) - 4(y 2 - x2 ) = (x - y)[l + 4(y + x)], nulos.
concluímos que pc I x 2 , de modo que (argumentando como acima) , • 1 . 45 . se n = p /3l
Coro 1ar10 1 ... pkf3k e, a decompos1çao canomca do
O
- A
O

e S 2a. Portanto, e = 2a, um número par. Se a 2': b, concluímos, de inteiro n > 1, então os divisores positivos de n são os números da
modo análogo, que e = 2b. forma pf 1 . . . p~\ com O S ai S /3i, para 1 S i S k. Em particular,
Por fim, como o primo p foi escolhido arbitrariamente, segue do te- denotando por d( n) o número de divisores positivos de n, temos
orema fundamental da aritmética que x -y é um produto de potências
k
de primos com expoentes pares, logo um quadrado perfeito. •
d(n) = Il(/3i + 1). (1.8)
A representação de um inteiro n > 1 como um produto de potên- i=l

cias de primos distintos é sua fatoração ou decomposição canô- Prova. Se d > 1 é divisor de n e p é um primo que divide d, então p
nica em fatores primos. Os corolários a segúir colecionam algumas também divide n, de forma que pé igual a um dos primos p 1, ... ,Pk·
consequências úteis da existência de uma tal fatoração. Mas, como isso é válido para todo divisor primo de d, segue que d =
pf 1 . . . p~k, com ai 2': O para todo i ( não escrevemos ai 2': 1 pois pode
Corolário 1.44. Sejam a, b, m e n naturais tais que am
ocorrer que Pi f d para um ou mais valores de i). Agora, se q E N é
mdc (m, n) = 1, então existe e EN tal que a= cn e b = cm.
tal que n = dq, então
Prova. É claro que a e b são divisíveis exatamente pelos mesmos
º
pnmos, p 1, ... , Pk dº1gamos. SeJam
º 1 ... Pk°'k e b -_ P/31
a = p Clél 1 ... Pkf3k ,
com ai, /3i 2': 1, as decomposições canônicas de a e b. Então,
e a parte de unicidade do teorema fundamental da aritmética permite
ma1
P1 · · ·
pm°'k
k
= am = bn = pn/31
1
pn/3k
· · · k concluir facilmente que ai S /3i para todo i. Reciprocamente, é ime-
diato que todo natural d da forma pf 1 . . . p~k, com O S ai S /3i para
e a parte de unicidade do teorema fundamental da aritmética garante
1 S i S k, é um divisor positivo de n.
que Para o que falta, o que fizemos acima, novamente em conjunção
(1.7)
com a parte de unicidade do teorema fundamental da aritmética, ga-
Assim, m I n/3i e, como m e n são primos entre si, o item (a) da rante que o número de divisores positivos de n coincide com o nú-
proposição 1.21 permite concluir que m 1 /3i- Se ui E N é tal que mero de sequências (a 1, ... , ak) de inteiros tais que O S ai S /3i para
/3i = mui, segue de (1. 7) que ai = nui; portanto, sendo e= pt 1 . . . p~k, 1 S i S k. Como há /3i + 1 possibilidades para ai, a fórmula (1.8) se-
temos gue do princípio fundamental da contagem (cf. corolário 1.9 do volume
4). •
44 Divisibilidade 1.3 Números primos 45

Ex:~pª:º:.::. ::::.l:~e::li::::::::::::c~:;eito se, esó


se, d(n) for ímpar.
I
ft
De volta ao parágrafo anterior ao corolário, agora podemos calcular
imediatamente

mdc (48, 270) = 2 · 3 = 6 e mmc (48, 270) = 24 · 33 . 5 = 2160.


l
Prova. Primeiramente, como 1 = 12 e d(l) = 1 é ímpar, podemos Suponha que saibamos que um certo natural n admite um divisor
supor que n > 1. Seja, então, n > 1 um inteiro e n = pf 1 ••• p~k sua primo p. A proposição a seguir, devido ao matemático francês Adrien-
decomposição canônica em fatores primos. Se n for quadrado perfeito, Marie Legendre5 , ensina como calcular o expoente de p na decompo-
então, para 1 ~ i ~ k, existe f3i E N tal que ai = 2f3i· Portanto, o sição canônica de n em fatores primos, mesmo que não conheçamos
corolário anterior fornece tal decomposição explicitamente. A fórmula (1.9) é conhecida como a
d(n) (a 1 + 1) ... (ak + 1) fórmula de Legendre.
(2f31 + 1) ... (2f3k + 1), Proposição 1.48. Sejam n > 1 natural e p primo. Se ep(n) denota o
um número ímpar. A recíproca é análoga.
• expoente de p na decomposição canônica de n!, então

Para o próximo corolário, é útil estendermos a decomposição ca- ep(n) = L l:J · (1.9)

i
nônica de um inteiro maior que 1 em primos permitindo expoentes k:::::1 p

iguais a O. Por exemplo, podemos escrever 48 = 24 • 3 e 270 = 2 · 33 • 5 Prova. Note, inicialmente, que a soma acima é sempre finita, uma
utilizando os primos 2, 3 e 5 em ambos os casos, i.e., escrevendo vez que, para pk > n, temos lP~ J = O. Seja k E N qualquer e
pk, 2pk, ... , mpk os múltiplos de pk menores ou iguais a n. Então
48 = 24 · 3 · 5° e 270 = 2 · 33 · 5.
mpk ~ n < (m + 1 )pk ou, equivalentemente, m ~ ; < m + 1. Por-
Corolário 1.47. Sejam a, b > 1 naturais dados, com a= pf 1 . . . p~k
e b = pf1 ••• p{k, onde p 1 < · · · < Pk são números primos e ai, f3i ~ O
1
1
tanto, m é o maior inteiro menor ou igual a p~, quer dizer, m = l p~J.
Assim, para cada k ~ 1 há exatamente
~
para 1 ~ i ~ k. Então

mdc (a, b) =
k
ITP~in{ai,.Bi}
k
e mmc(a,b) = ITP~ax{ai,.Bi}_
l;J lp: J 1

i=l i=l naturais menores ou iguais a n e que são múltiplos de pk mas não de
Prova. Façamos a prova para o mcd (a prova para o mmc é análoga). pk+I. Mas, como cada um de tais números contribui com exatamente
Como min{ ai, f3i} ~ ai, f3i para todo i, temos que o número d = k fatores p para ep(n), segue que
f17= 1 p~in{ai,.Bi} divide ambos a e b. Seja, agora, d' um divisor positivo
qualquer de a e b. Pelo corolário 1.45, a decomposição em primos de
d' é da forma d' = Pi1 . . • Pl/, com "li ~ O para todo i. Mas, d' 1 a
implica "/i ~ ai e d' 1 b implica "li ~ f3i· Assim, para todo i, temos
"li ~ min {ai, f3i}, de modo que d' 1 d. Logo, d= mdc (a, b). • 5 Matemático francês dos séculos XVIII e XIX.

46 Divisibilidade 1.3 Números primos 47

Os dois exemplos a seguir trazem aplicações interessantes da fór- Problemas - Seção 1.3
mula de Legendre.
1. Sejam p e q primos ímpares consecutivos. Prove que existem
Exemplo 1.49 (União Soviética). Ache o número de zeros consecu- inteiros a, b, e > 1, não necessariamente distintos, tais que p+q =
tivos no final do número 1000!. abc.

Prova. Escrevendo 1000! = 2e 2 (10oo) · 5es(10oo)m, com m E N, vemos f 2. (IMO - adaptado.) Seja k E N tal que p = 3k + 2 é primo. Se
que basta calcular o menor dos números e2(1000) e es(lOOO) a fim 1.
de encontrar a maior potência de 10 que divide 1000!. Por outro
1 m e n são naturais tais que

lado, o menor de tais números é claramente e5 (1000), o qual pode ser ~i


calculado por intermédio da fórmula de Legendre, levando em conta
que 55 > 1000: prove que p I m.

3. (Hungria.) Dado p > 2 primo, ache todos os x, y E N tais que


e5 (1000) = I:= llOOOJ
4
-.
51
= 200 + 40 + .8 + 1 = 249. ~
p
= .!. +.!..
X y
j=l
4. (IMO.) Ache todos os n E N tais que n(n + l)(n + 2)(n + 3)
Logo, 1000! termina em 249 zeros. • tenha exatamente três divisores primos distintos.
Exemplo 1.50 (Iugoslávia). Prove que 2n f n!, para cada n E N.
5. (Rússia.) Seja (an)n~l uma sequência de números naturais tais
Prova. O resultado é claramente verdadeiro para n = 1. Para n > 1, que mdc(ai,aJ) = mdc(i,j) sempre que i # j. Prove que
o teorema de Legendre garante que a maior potência de 2 que divide an = n, para todo n 2 1.
n! tem expoente k = ~J~l l; J.
Mas, como l x J ~ x para todo x e 6. * Dizemos que um natural n é livre de quadrados se n = 1 ou
l; J = O para todo j suficientemente grande, segue que n = P1 .. ·Pk, com P1 < · · · < Pk primos. Faça os seguintes itens:
(a) Prove que todo inteiro n > 1 pode ser unicamente escrito
na forma n = ab2 , com a livre de quadrados e b natural.
onde utilizamos a fórmula para a soma dos termos de uma série geo- (b) Use o item (a) para mostrar que, se existissem exatamente k
métrica (cf. proposição 3.21 do volume 3) na última igualdade acima. números primos, então haveria exatamente 2k inteiros livres
Logo, k < n, de sorte que 2n f n!. • de quadrados e que o número de inteiros positivos menores
ou iguais a n seria, no máximo, 2k ..jn.
(c) Use o item (b) para dar outra prova da infinitude dos pri-
mos.
48 Divisibilidade 1.3 Números primos 49

7. Mostre que há infinitos primos de cada urna das formas 3k + 2 16. (OIM.) Para cada n E N, sejam 1 = d 1 < · · · < dk = n os
e 6k + 5. divisores positivos de n. Encontre todos os n satisfazendo as
seguintes condições:
8. (Iugoslávia.) Para cada n EN, prove que o número 22n - 1 tem
ao menos n fatores primos distintos. (a) k 2 15.

9. Se pé urn primo ímpar, prove que p divide (~) - 2. (b) n = d13 + d14 + d15.
(c) (ds + 1) 3 = d15 + 1.
10. * Sejam a e n inteiros positivos, corn a > 1. Faça os itens a
seguir: 17. (Japão.) 6 Para cada inteiro n > 1, sejam I(n) a sorna dos mai-
ores divisores ímpares dos números 1, 2, ... , n e T(n) = 1 + 2 +
(a) Se an + 1 é primo, então n é urna potência de 2 (veremos no · · · + n. Prove que existem infinitos valores de n para os quais
problema 6, página 130, que a recíproca não é verdadeira): 3I(n) = 2T(n).
(b) Se n > 1 e an - 1 é primo, então n é primo e a = 2
18. (Austrália.) Ache todos os naturais n para os quais d(n) = i·
(a recíproca não é verdadeira, conforme atesta o exemplo
211 - 1 = 23 · 89). 19. (IMO.) Sejam m, n inteiros não negativos arbitrários. Prove que
o número
11. (IMO.) Ache todos os n EN, n > 6, tais que os naturais menores (2m)!(2n)!
que n e primos corn n forrnern urna progressão aritmética. m!n!(m+n)!
sempre é urn inteiro.
12. (IMO.) Sejam k, m, n naturais tais que m + k + 1 é urn primo
maior que n + 1. Se C8 = s(s + 1) para cada s EN, prove que 20. Para cada inteiro positivo n, seja an = (2:).
li
(a) Mostre que o número binomial an é sempre par.
(b) Prove que 4 1 an se, e só se, n não é potência de 2.
é divisível por c1c2 • · • Cn·
21. (Romênia.) Seja n urn natural cuja representação binária tern
13. (Hungria.) Seja n urn natural dado. Mostre que há exatamente exatamente k algarismos 1. Prove que 2n-k divide n!.
d(n 2 ) pares ordenados (u, v) tais que u, v EN e rnrnc (u, v) = n.
22. Ache todos os naturais a, b e k tais que a e b sejam primos entre
14. (IMO.) Ache todos os naturais a e b tais que ab = bª.
2 si e (a+ kb)(b + ka) seja urna potência de urn primo.

15. (BMO.) Encontre todos os n EN tais que n = d:f_ +dª+ d~+ d~, 23. (OBM - adaptado.) Para cada inteiro n > 1, seja p(n) o maior
onde 1 = d1 < d2 < d3 < d4 são os quatro menores divisores divisor primo de n. Faça os seguintes itens:
positivos de n. 1 6 Para a recíproca deste problema, veja o problema 5, página 98.

l
50 Divisibilidade

(a) Se q é um primo ímpar, prove que não podemos ter p(q2k) >
11
'' p(q2k + 1) para todo inteiro k ~ O.
(b) Se p( q2 k) < p( q2 k + 1) e k é mínimo com tal propriedade,
mostre que p( q2 k - 1) < p( q2 k).
(c) Mostre que existem infinitos naturais n tais que p( n - 1) <
p(n) < p(n + 1). CAPÍTULO 2
24. (OBM.) Mostre que existe um conjunto A, formado por inteiros
positivos e tendo as seguintes propriedades:

(a) A tem 1000 elementos.


Equações Diofantinas
(b) A soma de qualquer quantidade de elementos distintos de A
(pelo menos um) não é uma potência perfeita de. expoente
maior que 1.

25. Seja p 1 = 2,p2 = 3,p3 = 5, ... a sequência dos números primos.


Para cada k EN, sejam Xk = P1P2 .. ·Pk + 1 e A= {xi, X2, ... }.
Fixado m EN, prove que podemos escolher um subconjunto B
Nosso propósito neste capítulo é estudar algumas equações Diofan-
do conjunto A satisfazendo as seguintes propriedades:
tinas elementares, destacadas dentre essas as equações qe Pitágoras e
(a) IBI = m. de Pell, para as quais caracterizamos todas as soluções. Também apre-
:Ili
(b) Dois elementos quaisquer de B são primos entre si. sentamos ao leitor o método da descida de Fermat, o qual fornece uma
ferramenta por vezes útil para mostrar que certas equações Diofantinas
(c) A soma dos elementos de qualquer subconjunto de B com não possuem soluções não triviais, num sentido a ser precisado.
mais de um elemento não é um número primo.

2.1 Ternos Pitagóricos


Vamos começar estudando as soluções, em inteiros não nulos x, y e
z, da equação x 2+y 2 = z 2, conhecida como a equação de Pitágoras.
Após encontrá-las, veremos como utilizar as informações obtidas para
resolver outras equações em números inteiros. O resultado fundamen-
tal está contido na seguinte

1 51
52 Equações Diofantinas 2.1 Ternos Pitagóricos 53

4, conforme t seja respectivamente par ou ímpar. Portanto, se a e b


forem ímpares, então c 2 = a 2 + b2 deixará resto 2 quando dividido por
4, uma contradição.
Como mdc (a, b) = 1, restam dois casos a considerar: a ímpar
e b par ou vice-versa. Analisemos o primeiro deles, sendo a análise
do segundo totalmente análoga. Sendo a ímpar e b par, segue de
c2 = a 2 + b2 que e também é ímpar. Escreva, agora,
Figura 2.1: Pierre Simon de Fermat foi um funcioná-
rio público francês e matemático amador que viveu
b2 = (e - a) (e + a); (2.2)
no século XVII e que teve a Matemática, notada-
mente a teoria dos números, como sua maior paixão.
se d'= mdc (e - a, e+ a), então d' divide

(e+ a)+ (e - a)= 2c e (e+ a) - (e - a)= 2a


Proposição 2.1. Os ternos (x, y, z) de inteiros não nulos tais que
x 2 + y2 = z 2 são dados por: e, daí, d' mdc (2a, 2c) = 2. Mas, como e- a e e+ a são ambos pares,
1

segue que d'= 2 e podemos escrever (2.2) como


x = 2uvd x = (u 2 - v 2 )d
{ y=(u 2 -v 2 )d ou { y = 2uvd (2.1)
z = (u2 + v 2 )d z = (u 2 + v 2 )d,
onde d, u e v são inteiros não nulos, com u e v de paridades distintas
com mdc ( c;a, e!ª) = 1. Segue do item (b) do corolário 1.22 a exis-
e primos entre si.
tência de naturais primos entre si u e v tais que e + a = 2u 2 e e - a =
Prova. Sem perda de generalidade, podemos supor x, y, z > O. Se 2v 2 , de forma que
d= mdc (x, y), e·ntão d 2 1 (x 2 + y 2 ), i.e., d 2 1 z 2 ; logo, d I z. Existem,
portanto, inteiros não nulos a, b e e tais que mdc (a, b) = 1 e (x, y, z) = a=u 2 -v 2 , b=2uv, c=u 2 +v 2 .
(da, db, de). Ademais, como
Ademais, como e = u 2 + v2 é ímpar, u e v têm paridades distintas.
Finalmente, por substituição na equação original, concluímos que
os ternos acima são realmente soluções da equação, de modo que nada
basta encontrarmos as soluções inteiras não nulas a, b e e da equação mais há a fazer. •
acima, sujeitas à condição mdc (a, b) = 1.
As condições a 2 + b2 = c 2 e mdc (a, b) = 1 nos dão facilmente Um terno (x, y, z) de inteiros positivos tais que x 2 + y 2 = z 2 é
mdc (a, e) = mdc (b, e) = 1. Lembre agora (cf. corolário 1.8) que um terno Pitagórico, em alusão ao matemático grego Pitágoras de
o quadrado de um inteiro t deixa resto O ou 1 quando dividido por Samos e seu famoso teorema sobre triângulos retângulos. De fato, um
54 Equações Diofantinas 2J Ternos Pitagóricos 55

tal terno (x, y, z) determina um triângulo retângulo de catetos x e y e As equações analisadas acima são, em um certo sentido, privilegi-
hipotenusa z. adas, uma vez que possuem uma infinidade de soluções inteiras não
Vejamos como aplicar o resultado acima para encontrar as soluções nulas. Nosso próximo exemplo será o de uma equação que só admite a
inteiras de outra equação diofantina. solução inteira x = y = z = O; a prova que apresentaremos ilustra um
método (a descida de Fermat) que pode, por vezes, ser utilizado a
Exemplo 2.2. Ache todas as soluções inteiras não nulas da equação
fim de provar que uma equação Diofantina não possui soluções inteiras
x 2 + y 2 = 2z 2, com x-/=- ±y.
não nulas.
Solução. Em uma qualquer dessas soluções, devemos ter x e y ambos Esquematicamente, o método da descida de Fermat consiste no
pares ou ambos ímpares, pois, caso contrário, x 2 + y 2 seria ímpar. cumprimento das seguintes etapas:
Assim, tomando a= e b=x1y x;u,
temos a, b E Z \ {O} ex= a+ b, 1. Supor que uma dada equação possui uma solução em inteiros
y = a - b. Substituindo tais expressões para x e y na equação original, não nulos.
concluímos que
ii. Concluir, a partir daí, que ela possui uma solução em inteiros
x2 + y2 = 2 z2 {::}- ª2 + b2 = z2. , não nulos que seja, em algum sentido, mínima.

Mas, uma vez que essa última equação é a equação de pitágoras, segue iii. Deduzir a existência de uma solução em inteiros não nulos menor
da proposição anterior a existência de inteiros não nulos d, u e v, com que a mínima (no sentido do item ii.), chegando, assim, a uma
u e v primos entre si e de paridades distintas, tais que contradição.
,i
a= 2uvd, b = (u 2 - v 2 )d, z = (u 2 + v 2 )d Exemplo 2.3. Prove que a equação 3x 2+y 2 = 2z2 não possui soluções
,ll!i,
inteiras não nulas.
ou
.,111::
Prova. Inicialmente, observe que não podemos ter exatamente um
1,,
'!1' a= (u2 - v 2 )d, b = 2uvd, z = (u2 + v 2 )d.
dos inteiros x, y, z igual a O. Suponha, pois, que a equação dada
Portanto, as soluções (x, y, z) da equação original, com x -/=- ±y, possua uma solução (x, y, z) com x, y, z EN (uma vez que, se (x, y, z)
são de um dos tipos abaixo, onde d, u, v satisfazem as condições acima for solução, então (±x, ±y, ±z) também o será). Então, dentre todas
descritas: tais soluções (x, y, z), existe uma para a qual z é o menor possível,
digamos x = a, y = b, z = e. Trabalhemos tal solução.
x = (u2 - v 2 + 2uv)d, y = (-u 2 + v 2 + 2uv)d, z = (u 2 + v 2 )d
Se 3 f b, temos de 3a2 + b2 = 2c2 que 3 f e. Mas aí, o corolário
ou 1.8 garante que b2 e c2 deixam resto 1 na divisão por 3 e a igualdade
3a2 + b2 = 2c2 nos dá uma contradição. Logo, 3 1 b, digamos b = 3b1
x = (u 2 - v 2 + 2uv)d, y = (u2 - v 2 - 2uv)d, z = (u2 + v 2 )d. para algum b1 E N, e segue de

2c2 = 3a2 + b2 = 3(a2 + 3bi)


56 Equações Diofantinas 2.1 Ternos Pitagóricos 57

que 3 1 e. Sendo e = 3c1 , para algum c1 E N, a igualdade acima nos x, y, z quando n > 2, utilizando, para tanto, matemática muitíssimo
dá avançada1 .
6cf = a 2 + 3bf, Podemos, entretanto, aproveitar o método da descida para analisar
um caso simples da equação (2.3), aquele em que 4 1 n.
de modo que 3 1 a, digamos a= 3a 1 , com a 1 EN. Portanto, a última
igualdade acima fornece Exemplo 2.4. Se n for um natural múltiplo de 4, então não existem
inteiros não nulos x, y, z tais que xn + yn = zn.

Prova. Seja n = 4k, com k natural. De xn + yn = zn, obtemos


de sorte que (a 1 , b1 , c1 ) é outra solução da equação original no conjunto
dos números naturais. Contudo, a relação O < c1 = i < e é uma
contradição, uma vez que partimos de uma solução em naturais (a, b, e)
para a qual e era o menor possível. Logo, nossa equação não possui ou seja, (xk, yk, z 2k) é solução não nula da equação a 4 +b4 = c2 . Assim,
soluções não nulas. • basta mostrarmos que essa última equação não admite soluções em
inteiros não nulos.
Voltando à equação de Pitágoras, uma generalização natural da Por absurdo, suponhamos que existam a, b, e E N tais que a4 +
mesma seria estudar a equação Diofantina mais geral abaixo, denomi- b = c2 . Podemos também supor que a, b, e foram escolhidos de tal
4
nada equação de Fermat, onde n > 2 é um inteiro fixado: modo que não haja outra solução positiva (a,/3,1) com 'Y < e (esta
é a hipótese que vamos usar no método da descida). Como (a 2 , b2 , e)
(2.3) resolve a equação de Pitágoras, a proposição 2.1, juntamente com a
lil:,: minimalidade de e, garante que mdc (a 2 , b2 ) = 1 e que existem naturais
Por cerca de três séculos, muitos dos mais eminentes matemáticos
,:ili::: u e v, primos entre si e de paridades distintas, tais que
il:'
. '
defrontaram-se com o problema de decidir sobre a existência de so-
luções, em inteiros não nulos x, y, z, para essa equação. O próprio a2 = u2 - v 2 , b2 = 2uv, e= u 2 + v 2
Fermat acreditava haver conseguido provar, provavelmente utilizando
seu método da descida, que tal equação não possui soluções não nulas ou
quando n > 2, e desde então esse resultado passou a ser conhecido a 2 = 2uv, b2 = u 2 - v2 , e = u 2 + v2 .
como o grande teorema de Fermat. Entretanto, seu argumento
nunca foi explicitamente escrito e, muito provavelmente, estava errado. Analisemos o primeiro caso acima, sendo a análise do segundo to-
Na última década do século XX, o matemático inglês Andrew Wi- talmente análoga. Então, a é ímpar e, como a 2 + v2 = u 2 , segue
,1, les, com a colaboração do também inglês Richard Taylor e apoiando- 1 Veja
também [9], onde é apresentado um relato romanceado super interessante

j
: i
se em importantes trabalhos anteriores de vários matemáticos, provou da saga de Andrew Wiles e de outros matemáticos na busca de uma prova para o
·:1
·.·1!.,,
que a equação de Fermat não possui soluções em inteiros não nulos grande teorema de Fermat.
J
'li
:.1'
:::.
1
58 Equações Diofantinas • 2.1 Ternos Pitagóricos 59

novamente da caracterização dos ternos pitagóricos a existência de Suponha, por contradição, que ab + cd = p, p primo. A condição
naturais p e q, primos entre si e de paridades distintas, tais que a > b > e > d > O garante que p = ab + cd ~ 4 · 3 + 2 · 1 = 14, de
modo que p ~ 17. Por outro lado,
a = p2 - q2' V = 2pq' U = p2 + q2.
Mas aí, 2p = 2ab + 2cd = (2a - c)b + (b + 2d)c,
b2 = 2uv = 4pq(p2 + q2 )
de modo que mdc (2a - e, b + 2d) divide 2p. Para mdc (2a - e, b + 2d)
e, como mdc (p, q) = 1, temos que ambos p e q são também primos ser par deveríamos ter b e e pares, donde p = ab + cd seria par, o que
com p 2 + q2. Portanto, a fim de que 4pq(p2 + q2) seja um quadrado é um absurdo. Logo,
perfeito, devemos ter p, q e p 2 + q2 quadrados perfeitos, digamos
mdc (2a - e, b + 2d) = 1 ou p.

com a, /3, 1 EN. Segue, então, que Afirmamos que mdc (2a - e, b + 2d) = 1. Suponha que mdc (2a -
e, b + 2d) = p. Então (2.4) nos daria que p 2 1 3(b2 - c2) e, daí,
p2 1 (b2 - c2), uma vez que p # 3. Porém, p = ab + cd > b, de modo
que O < b2 - c2 < p 2, um absurdo.
com
Agora, sejam x = 2a-c, y = b+2d. Então mdc(x,y) = 1 e segue
e= u2 + v2 > u = P2 + q2 = ,2 ~ r,
de (2.4) que x 2 - y 2 = 3(b2 - c2) ou, ainda,
contrariando a minimalidade de e. Logo, não há soluções não nulas
de xn + yn = zn quando 4 1 n. • (x - y)(x + y) = 3(b - c)(b + e).

Terminamos esta seção apresentando um exemplo de uso bem mais


Consideraremos dois casos separadamente:
elaborado do método de descida.

Exemplo 2.5 (IMO). Sejam a, b, e, d inteiros tais que a > b > e > (a) b e e têm paridades distintas: o fato de ser p = ab + cd garante
d > O. Suponha que que mdc (b, e) = 1. Como b + e e b - e são ímpares, isto implica em
rndc (b + e, b - e) = 1. Um argumento análogo implica em mdc (x +
ac + bd = (b + d + a - e) (b + d - a + e). y, x - y) = 1 também. Se 3 1 (x + y) então
Prove que ab + cd nunca é um número primo.
x-y = mdc (x-y, (b+c)(b-c)) = mdc (x-y, b+c) mdc (x-y, b-c)
Prova. Simplificando a condição do enunciado obtemos a 2 + c2 - ac =
b2 + d2 + bd ou, ainda, e

(2a - c) 2 + 3c2 = (b + 2d) 2 + 3b2. (2.4) x+y = 3 mdc (x+y, (b+c)(b-c)) = 3 mdc (x+y, b+c) mdc (x+y, b-c).
60 Equações Diofantinas : 2.1 Ternos Pitagóricos 61

Escrevendo a = mdc (x - y, b - e), /3 = mdc (x - y, b + e), '"'( ==, chegamos, como acima, a 2a = a/3 + /38 + 3'"'(8 - ª'"'!, b = ª'"'! + /38,
mdc (x + y, b - e) e 8 = mdc (x + y, b + e), obtemos x - y = a/3 e e= -a'"'(+ /38 e 2d = -a/3 - /38 + 3'"'(8 - ª'"'!· Daí,
x + y = 3'"'(8. Por outro lado,
2p = 2(ab + cd) = /38(2a 2 + 38 2).
b - e = mdc (b - e, x - y) mdc (b - e, x + y) = a'"'(
Nem /3 nem 8 são iguais a p, pois, do contrário, teríamos b > p, o que
e, analogamente, b +e= /38. Resolvendo para a, b, e, d obtemos contradiz ser ab+cd = p. Logo /38 :S 2, donde O< d< e::::; 2-a'"'(::::; 1,
um absurdo. •
4a = a/3 + /38 + 3'"'(8 - ª'"'!, 2b = ª'"'! + /38,

2c = -a'"'(+ /38 e 4d = -a/3 - /38 + 3'"'(8 - ª'"'!·


Daí
Problemas - Seção 2.1
8p = 8(ab + cd) = /38(2a 2 + 382).
Mas, como b + e ímpar, temos /3 e 8 são ímpares e,. portanto, ', 1. (OBM - adaptado.) Dados n, k EN, prove que a equação xn +
/38(2a 2 + 382) é ímpar, o que é um absurdo. Se 3 r (x - y) chegamos kyn = zn+l possui infinitas soluções em inteiros positivos x, y, z.
a uma contradição de modo análogo.
2. Encontre todas as soluções, em inteiros positivos, da equação

(b) b e e têm paridades iguais: nesse caso, b e e devem ser ímpares, (x + y)2 + (y + z) 2 = (x + z) 2.
pois, do contrário, 2 dividiria ab + cd = p. Assim, nas notações acima,
temos x e y também ímpares. Segue que 3. Mostre que as soluções em inteiros não nulos da equação x 2 +
2y 2 = z 2 são dadas por
mdc(b+c,b-c) = mdc(x+y,x-y) =2,
x = ±(u 2 - 2v 2)d, y = ±2uvd, z = ±(u 2 + 2v 2)d,
poisjátemos mdc(x,y) = 1. Se3 I (x+y) (ooutrocasoénovamente
análogo), escrevendo onde d, u e v são inteiros não nulos, com u e 2v primos entre si.

4. Mostre que nenhuma das equações a seguir possui soluções in-


teiras não nulas:
e pondo
(a) x4 + 4y4 = z2.
a = mdc ( x ; y, b ; e) , /3 = mdc ( x ; Y, b ; e) , (b) x4 + 2y4 = z2.
(c) x2 + y2 = 3z2.
'"Y -_ mdc (x- +2-y, -b -2-e) , ui. -_ md c (x- +2-y, -b +2-e) (d) x 3 + 5y 3 = 9z 3 .

,,
1,
:!
62 Equações Diofantinas 2.2 A equação de Pell 63

5. (a) (Hungria.) Mostre que não existem soluções racionais x e onde d> 1 é um inteiro livre de quadrados (cf. problema 6, página 47)
y para a equação x 2 + xy + y 2 = 2. e m é um inteiro qualquer. A equação acima é conhecida como a
(b) Encontre todas as soluções racionais x e y para a equação equação de Pell 2 .
x2 + xy + y2 = 1.
Uma vez que d é um produto de primos distintos, o exemplo 1.23
garante que ,/d, é irracional. Em particular, quando m = O a equação
6. Prove o seguinte teorema de Euler: não existem inteiros nao acima não admite soluções inteiras além da trivial x = y = O, pois, se
nulos w, x, y e z tais que w 2 + x 2 + y 2 = 7z 2 . não fosse este o caso, teríamos x, y i- O e, daí, ,/d,= :Ey E Q. Por outro
lado, se d, m < O, então a equação de Pell não tem soluções e, se d<
7. (Bulgária.) Prove que não existem x, y e z racionais, tais que O< m, então (2.5) tem no máximo um número finito de soluções. No
entanto, mesmo para d, m > O, (2.5) pode não ter nenhuma solução,
x2 + y 2 + z 2 + 3(x + y + z) + 5 = O.
conforme atesta o problema 1.
Doravante, suporemos que m = 1 ( o caso geral é parcialmente
8. (Crux.) Seja r um inteiro positivo dado. Queremos calcular o respondido pelo problema 5; a esse respeito, veja também [5]). O
número de triângulos retângulos ABC, dois á dois não congru- exemplo a seguir mostra, nesse caso, que (2.5) pode ter uma infinidade
entes, satisfazendo as condições a seguir: de soluções.

(a) O raio do círculo inscrito em ABC mede r. Exemplo 2.6. A equação x 2 - 2y 2 1 possm uma infinidade de
soluções inteiras positivas.
(b) Os comprimentos dos lados de ABC são números inteiros
primos entre si. Prova. Note que x = 3, y = 2 é uma solução. Por outro lado,
podemos gerar infinitas soluções dessa equação a partir de uma solução
Mostre que o número de tais triângulos é 2k, onde k é o número não nula (a, b), do seguinte modo: partindo de a 2 - 2b2 = 1, temos
de fatores primos distintos de r.
(a+ bv'2)(a - bv'2) = 1
9. (IMO.) Dados n EN e um círculo de raio 1, mostre que podemos
escolher n pontos A 1 , A2 , ... , An sobre o mesmo, tais que AiAj e, daí,
é racional, quaisquer que sejam 1 ~ i < j ~ n. (a+ bv'2)2(a - bv'2) 2 = 1.
Desenvolvendo os binômios, chegamos a

2.2 A equação de Pell (a 2 + 2b2 + 2abv'2)(a 2 + 2b2 - 2abv'2) = 1

ou, ainda, a
Examinamos, nesta seção, as soluções de equações do tipo
(a 2 + 2b2 )2 - 2(2ab) 2 = 1.
x2 - dy 2 = m, (2.5) , 2 Após John Pell, matemático inglês do século XVII.

j
64 Equações Diofantinas 2.2 A equação de Pell 65

Portanto, (a 2 + 2b2 , 2ab) também será solução e, sendo a e b naturais, : Segue, daí, que
temos a < a 2 + 2b2 . Repetindo o argumento acima sucessivas vezes,
obtemos uma infinidade de soluções para a equação dada. • , a_ lJaJ - lkaJ < 1 1 < 1 .
(2.6)
l
j - k (j - k )n - (j - k )2'
O método utilizado no exemplo acima se generaliza facilmente para
mostrar que a equação de Pell x 2 - dy 2 = 1 admite infinitas soluções , e fazendo x = lja J- l ka J e y = j - k, temos O y - n e 1 !!'.y - a 1 << < -1..
y2

não nulas, desde que admita pelo menos uma tal solução; ademais, Por outro lado, se d= mdc (x, y) ex= dx 1 , y = dy 1 , então
com poucas modificações podemos tratar equações mais gerais (cf. ,
X1 1 1 1
problemas 5 e 7, por exemplo). I Y1 - a < Y2 S Yi '
Apesar de, em certos casos particulares, podermos encontrar facil- ;
de modo que podemos supor que mdc (x, y) = 1.
mente infinitas soluções da equação (2.5), por enquanto não sabemos
se há outras. A fim de responder essa pergunta, precisamos de um Para garantirmos a existência de infinitos tais pares, sejam x e y
resultado preliminar sobre aproximação de irracionais por racionais, inteiros não nulos, primos entre si e tais que 1 ~ - a 1 < ; 2 • Escolhendo
devido a G. L. Dirichlet. Para a prova do mesmo, lembre-·se de que um natural n1 tal que 1 ~ - a 1 > ; 1 e repetindo (com n1 no lugar de
(cf. dicussão que precede o problema 15, página 32), para x E IR, a_ n) o argumento que levou a (2.6), obtemos inteiros não nulos e primos
parte fracionária de x é o número real {x} E [O, 1), definido por entre si X1 e Y1, tais que O < y 1 S n 1 e
{x}=x-lxJ.
,,1:
,, '
Lema 2.7 (Dirichlet). Se a é um irracional qualquer, então existem.
infinitos racionais ~, com x e y inteiros não nulos, primos entre si e
Por outro lado,
tais que

l -ai<
x1
Y1
_1_
n1Y1 -
<2_ <1:-:-al
n1 y . '
Prova. Seja n > 1 um inteiro qualquer e considere os n + 1 números ,
de sorte que ;: =/:- ~. Repetindo esse argumento sucessivas vezes,
{ja} E [O, 1), com j = O, 1, ... ,n. Como
obtemos uma infinidade de pares (x, y) com as propriedades desejadas.
[O, 1) = [ O, ~) u [ ~, ~) u ... u [ n : l, 1) ,
O lema de Dirichlet permite mostrar que, fixado um natural d livre

uma união de n conjuntos, o princípio da casa dos pombos garante a ;
de quadrados, a equação (2.5) admite uma infinidade de soluções para
existência de índices O S k < j S n tais que {j a}· e { ka} pertencem
pelo menos um valor inteiro de m.
a um mesmo intervalo dos que aparecem no lado direito da igualdade
i
acima. Então l{ja} - {ka}I < ou, o que é o mesmo, Lema 2.8. Se d > 1 é um natural livre de quadrados, então existe
m E Z \ {O} tal que a equação x 2 - dy 2 = m admite infinitas soluções
l(j - k)a - (ljaJ - lkaJ)I < .!..
n inteiras.
2.2 A equação de Pell 67
66 Equações Diofantinas

e, se x2 - x1 = mr e Y2 - Y1 = ms, com r, s E Z, temos


Prova. Como v'd, (/:. Q, o lema de Dirichlet garante que o conjunto
S dos pares (x, y) de inteiros não nulos, primos entre si e tais que X1 (x2 - x1) + (xi - dyi) + (Y1 - Y2)dy1
1; - v'd,I< : 2 é infinito. Mas, se (x, y) for um tal par ordenado, m(rx 1 + 1 - sdy 1 )
!
então lx - yv'd,I < 1 1 e a desigualdade triangular nos dá
e

lx2- dy2 = lx - yv'dllx + yv'dl < l~I (lx - yv'dl + 2lylv'd)


1

Por simplicidade de notação, escrevamos x 1x 2 - dy 1y2 mu e


x 2y1 - X1Y2 = mv, com u, v E Z. Segue de (2.7) que
l~I ( l~I + 2lylv'd) < 2v'd + i.
(2.8)
De outro modo, para (x, y) E S, o conjunto dos inteiros x2 -
dy 2 está contido no conjunto dos inteiros não nulos situados entre os e, daí,
números reais -(2v'd, + 1) e 2v'd, + 1. Mas, uma vez que tal conjunto
é finito, existe um inteiro mi= Oentre -(2v'd, + 1) .e 2v'd, + 1, o qual
Multiplicando ordenadamente essas duas igualdades, chegamos a
se repete um número infinito de vezes dentre os valores de x 2 - dy 2 ,
para (x, y) E S. Por fim, isso é o mesmo que dizer que a equação
x 2 - dy 2 = m admite uma infinidade de soluções inteiras. •
de maneira que u 2 - dv 2 = 1.
Estamos finalmente em condições de caracterizar todas as soluções Resta mostrar que u, v i= O. Se u = O, teríamos -dv 2 = 1, cla-
de (2.5) quando m = 1; os ingredientes-chave serão o lema anterior e ramente um absurdo. Se v = O, teríamos u = ±1 e seguiria de (2.8)
"'
'!i'I,
o método da descida de Fermat. que
Proposição 2.9. Se d > 1 é um natural livre de quadrados, então
a equação x 2 - dy 2 = 1 admite pelo menos uma solução em inteiros logo,
positivos x, y. X1 + Y1 v'd = ±(x2 + Y2v'd)
e, daí, lx1I = lx2I, uma vez que v'd, (/:. Q. Mas isso contradiz nossas
Prova. Tome, pelo lema anterior, m E Z \ {O} tal que a equação
x 2 - dy 2 = m tenha uma infinidade de soluções. Como os restos da escolhas de x 1 e x 2 . •

divisão de um inteiro por m são em número finito, podemos escolher Chegamos finalmente ao resultado desejado, o qual caracteriza to-
duas dessas soluções, (x1, Y1) e (x2, Y2) digamos, tais que lx1I -=!=- lx2I e das as soluções da equação x 2 - dy 2 = 1, quando d> 1 é um natural
m divida x2 - X1, Y2 - Y1· Então livre de quadrados. Observe que, pela proposição anterior, tal equação
admite pelo menos uma solução.
68 Equações Diofantinas 2.2 A equação de Pell 69

Teorema 2.10. Se d > 1 é um natural livre de quadrados, então a ou, ainda,


equação x 2 - dy 2 = 1 admite infinitas soluções em inteiros positivos 1 < X+ yvd <a. (2.9)
x e y. Mais precisamente, se x = x 1 e y = y 1 é a solução em inteiros an
positivos para a qual a soma x + y,vd é a menor possível, então as Mas, como an = Xn + Ynvd ex~ - dy~ = 1, seguiria então que
demais soluções inteiras positivas da equação são dadas pelos naturais
Xn, Yn que satisfazem a igualdade X+ y,vd X+ y,vd
Xn + Ynvd
(xn - YnVd)(x + yVd)
onde n é um número natural. (xxn - dyyn) + (xny - YnX)Vd,

Prova. Seja a = x1 + Y1 vd, com x 1, y 1 E N escolhidos como no com


enunciado.
Dado n E N, sabemos (cf. exemplo 6.11 do volume 1) que existem
Xn, Yn EN tais que Portanto, (xxn - dyyn, XnY - YnX) também é solução, tal que (por
(2.9))

Assim,

1 (x~ - dyft = (x1 + Y1 Vdt(x1 - Y1 Jdt Assim, se mostrarmos que XXn - dyyn, XnY-YnX > O, obteremos uma
(xn + YnVd)(xn -ynVd) = X~ - dy~, contradição à minimalidade de a (observe que, aqui, operamos com a
descida de Fermat).
de modo que todos os pares (xn, Yn) do enunciado são soluções da Para o que falta, sendo
equaçao.
Agora, se (x, y) é uma solução qualquer em inteiros positivos, basta a= XXn - dyyn e b = XnY - YnX,
mostrarmos que existe n E N tal que
temos a + bvd = x+ynv'd
a
> 1 e a2 - db 2 = 1, de modo que
x+yVd=d1 • 1
a - bVd = vd > O.
a+b d
Suponha o contrário. Como a > 1, temos limn-++oo an +oo, de
sorte que existe n E N tal que Então, por um lado,

2a = ( a - bVd) + (a + bVd) > O;


70 Equações Diofantinas 2.2 A equação de Pell 71

por outro, 3. Prove, sem usar o teorema 2.10, que a equação x 2 - 2y 2 = -1


a- bv'd = 1 v'd < 1 admite uma infinidade de soluções inteiras.
a+b d
4. Mostre que há infinitos inteiros positivos n tais que n 2 + (n + 1) 2
nos dá bvd, > a - 1 ~ O, de sorte que b > O.
Exemplo 2.11. Ache todas as soluções da equação x2 - 2y 2 =

1 em
é um quadrado perfeito.

naturais x e y. 5. * Sejam d, m E N, sendo d > 1 livre de quadrados. Se a equação


x2 - dy 2 = rri tiver uma solução (x 0 , y0 ) em inteiros positivos,
Solução. O teorema anterior ensina que as soluções inteiras positivas prove que ela terá infinitas soluções.
dessa equação são da forma (xn, Yn), onde Xn e Yn são os únicos inteiros
positivos para os quais 6. Ache uma infinidade de soluções inteiras positivas para a equação

y2 + 1 = x(x + y).
e (xi, Y1) é a solução positiva tal que x 1 + y 1vÍ2 é o menor possível.
::;
Como os pares (1, 1), (1, 2), (2, 1), (2, 2), (3, 1), (1,-3), (2, 3) e (4, 1) 7. Generalize o problema anterior do seguinte modo: sejam a, b, e E
111
Ji Z tais que ~ = b2 - 4ac é maior que 1 e livre de quadrados. Se
,! li!i não são soluções da equação mas (3, 2) o é, é imediato verificar que
n E Z for tal que a equação
11: 1
:

x1 = 3, Y1 = 2. Portanto, as soluções positivas são os pares (xn, Yn)


111: ,11·IJ dados pela igualdade
11!:
x2 - ~y 2 = 4an
1

tem ao menos uma solução inteira (x 0 , y0 ), tal que 2a 1 (x 0 -by0 ),

• mostre que a equação

ax 2 + bxy + cy 2 = n

Problemas - Seção 2.2 tem infinitas soluções inteiras.

1. * Se d e m deixam resto 3 quando divididos por 4, mostre que a


equação (2.5) não possui soluções inteiras.

2. Em relação ao exemplo 2.11, prove que as soluções inteiras positi-


vas (xn, Yn) da equação x 2 -2y 2 = 1 são definidas recursivamente
por x1 = 3, Y1 = 2 e, para n ~ 1 inteiro,
72 Equações Diofantinas

CAPÍTULO 3

Funções Aritméticas Multiplicativas

Este breve capítulo introduz uma importante classe de funções, di-


tas aritméticas multiplicativas, as quais desempenham papel de relevo
na teoria elementar dos números. Dentre as muitas funções aritméti-
cas multiplicativas que estudaremos, duas destacam-se, de nosso ponto
de vista: a função de Mobius, essencial à obtenção da célebre fórmula
de inversão de Mobius, e a função <p de Euler, que se revelará impres-
cindível para os desenvolvimentos teóricos constantes dos capítulos
subsequentes.
Em tudo o que segue, nos referiremos a uma função f : N -+ IR
como uma função aritmética.

Definição 3.1. Uma função aritmética f : N-+ IR é multiplicativa


se, para todos m, n EN primos entre si, tivermos f(mn) = f(m)f(n).

73
74 Funções Aritméticas Multiplicativas .
75

Como 1 é relativamente primo consigo mesmo, segue que, se f :


m = q1!31 ... q1f3z , t emos Pi ,__j_ qj para t o d os z,· J.· P ort ant o, a d ecompos1-
·
N--+ :IR. for uma função aritmética multiplicativa, então f(l) = J(1)2,
ção canônica de mn em primos é
de modo que f(l) = O ou 1. Caso seja f(l) = O, teremos
_ a1 °'k (31 f3z
mn - Pi · · · Pk qi · · · ql
f(n) = f(n · 1) = f(n)f(l) = O, V n EN,
e, daí,
i.e., f será a função identicamente nula. Portanto, doravante supore-
mos, salvo menção explícita em contrário, que se f : N --+ :IR. for uma '. d(mn) = (a1 + 1) ... (ak + l)(,81 + 1) ... (,Bz + 1) = d(m)d(n).
função aritmética multiplicativa, então f(l) = 1.
Note ainda que, se f: N--+ :IR. for uma função aritmética multipli- · No que segue, para n EN, denotaremos por D(n) o conjunto dos
cativa e n > 1 é um inteiro com decomposição canônica em primos .da • divisores positivos de n, de sorte que d(n) = ID(n)I.
forma n = pf 1 . . . p~k , então repetidas aplicações da definição nos dão • Lema 3.3. Se m e n são naturais primos entre si, então a aplicação
f(n) = f(pf 1 ) • • • f(p~k). (3.1) f: D(m) x D(n) -----+ D(mn)
(X, y) f-----t XY
Em palavras, a igualdade acima significa que, para sabermos calcular
os valores f (n), com n E N, é suficiente sabermos calcular os valores é uma bijeção.
f(pª), com p primo e a EN.
Por fim, se f: N--+ :IR. for uma função aritmética tal que f(l) = 1, Prova. Segue do exemplo 3.2 e do princípio fundamental da contagem
então a relação f (mn) = f (m) f (n) é sempre satisfeita quando m = 1 (cf. corolário 1.9 do volume 4) que
ou n = 1; portanto, para provarmos que uma tal f é multiplicativa,
ID(mn)I = d(mn) = d(m)d(n) = ID(m)l · ID(n)I = ID(m) x D(n)I.
basta considerarmos o caso m, n > 1.
Utilizaremos várias vezes as observações acima sem maiores co- Portanto, o domínio e o contradomínio de f têm quantidades iguais de
mentários. elementos, de modo que, para provarmos que f é uma bijeção, basta
estabelecermos sua sobrejetividade. Para tanto, apliquemos o item
Exemplo 3.2. Vimos no corolário 1.45 que, se n = pf 1 ••• p~k é a
(d) da proposição 1.21: se k I mn, então a condição mdc (m, n) = 1
decomposição canônica de um natural n > 1 em primos, então
garante que

illi I
k = mdc (k, mn) = mdc (k, m) · mdc (k, n).
;1,
'j·.''.
,1
é o número de divisores positivos do natural n. Afirmamos que a
,1 Portanto, fixado k E D(mn) e pondo a= mdc (k, m) e b = mdc (k, n),
função d : N --+ :IR. assim obtida, denominada função número de
'li temos a E D(m), b E D(n) e f((a, b)) = ab = k, de sorte que k per-
!l. divisores positivos, é multiplicativa. De fato, para m, n > 1 primos
1:1 '' tence à imagem de f. Mas, como k E D(mn) foi escolhido arbitraria-
entre si, com decomposições canônicas em primos n = pf 1 ••• p~k e
1: 1 mente, segue que f é sobrejetiva. •
:i1'
:,·
76 Funções Aritméticas Multiplicativas 77

Em tudo o que segue, escreveremos Lo<dln f (d) para denotar o Portant o, se n -_ p a1 °'k •
1 ... pk , com P1 < · · · < Pk primos e 0:1, ... , O:k E
somatório dos valores f(d) quando d varia em D(n). A proposição a N, segue de (3.1) e dos cálculos acima que
seguir estabelece uma das mais importantes propriedades das funções
aritméticas multiplicativas. (3.2)
Proposição 3.4. Se f : N ----+ IR é uma função aritmética multiplica-
tiva, então a função F : N ----+ IR dada por Para o exemplo a seguir, lembramos (cf. problema 6, página 47)
F(n) = L J(d) que um inteiro n > 1 é livre de quadrados se n = P1 ... Pk, com p 1 <
... < Pk primos; equivalentemente, n é livre de quadrados se não existe
O<dln
um inteiro q > 1 tal que q2 n. 1

também é multiplicativa.
A próxima definição apresenta uma das mais importantes funções
Prova. Se mdc (m, n) = 1, segue do lema anterior e do caráter mul- aritméticas multiplicativas, a função de Mobius.
tiplicativo de f que
Definição 3.6. A função de Mõbius 1 é a função µ : N ----+ IR dada
por
O<dlmn
1, se n = 1
={ O, se q2 n, para algum inteiro q > 1
(01mf(d1)) (O~nf(dz)) µ(n) 1

(-1) k, se n = P1 ... Pk, com P1, ... , Pk primos distintos


F(m)F(n). Para verificar que a função de Mobius é multiplicativa, sejam

Exemplo 3.5. A função f: N----+ IR dada por f(n) = n é obviamente


• m, n > 1 inteiros primos entre si. Então, mn será divisível por um
quadrado maior que 1 se, e só se, m ou n o forem; sendo esse o caso,
é imediato que
multiplicativa. Portanto, segue da proposição anterior que a função f(mn) =O= f(m)f(n).
s : N ----+ IR dada por
Por outro lado, se m e n forem livres de quadrados, digamos n =
s( n) = L f (d) = L d P1 ... Pk e m = q1 ... qz, com P1 < · · · < Pk e q1 < · · · < qz primos, a
O<dln O<dln condição mdc (m, n) = 1 garante que Pi -=/: qj para todos i, j. Portanto,
também é multiplicativa. A função s é a função soma de divisores mn = p 1 ... pkq1 ... qz é a decomposição canônica de mn em primos,
positivos. Para n = pª, com p primo e a E N, segue do corolário de sorte que
1.45 que
a . pa+l - 1 µ(mn) = (-ll+z = (-ll(-l)z = µ(m)µ(n).
s(pª) = ~ d = ~ p1 = .
~ ~ p-1 1 Após o matemático alemão do século XIX August Mobius.
O<dlp°' j=O
78 Funções Aritméticas Multiplicativas 79

A proposição a seguir destaca uma importante propriedade da fun- é uma bijeção; realmente, como f o f = Idv(n), segue do exemplo 1.40
ção de Mõbius. do volume 3 que f é uma bijeção.
Nas notações da proposição 3.4, a fórmula (3.4) a seguir, conhecida
Proposição 3.7. Seµ: N---+ lR é a função de Mõbius, então
como a fórmula de inversão de Mõbius, ensina como recuperar a
1 sen=l função f a partir da função F (mesmo quando f não for multiplica-
L µ(d)
O<dln
=
{ o'se n > 1
'
tiva).
Teorema 3.8 (Mõbius). Seja f : N ---+ lR uma função aritmética
Prova. Seja F: N---+ lR a função dada por
qualquer. Se F : N ---+ lR é a função dada por F (n) = Z:o<dln f (d),
F(n) = L µ(d). então
O<dln f(n) = L
F (~)µ(d)= F(d)µ (~). L (3.4)
O<dln O<dln
Pela proposição 3.4, Fé multiplicativa, e queremos mostrar que F(l) = ·
Prova. A segunda igualdade em (3.4) segue da bijetividade da função
1 e F(n) = O para n > 1. Consideremos três casos separada~ente:
em (3.3). Para a primeira igualdade, note inicialmente que

(i) n = 1: temos F(l) = Z:o<dll µ(d) = µ(1) = 1.


o~n F G) µ(d) - O~n c(d) o~;; /(d')) = L L µ(d)J(d').
O<dln O<d' 1~
(ii) n = pk, com p primo e k 2: 1 inteiro: então
k
Mas, como d I n e d' 1 ~ se, e só se, d' 1 n e d 1 !f,, segue daí que
F(pk) = L µ(d) = L µ(i}) = µ(1) + µ(p) = o.
O<dlpk j=O
L F (~)µ(d)= L L µ(d)f(d') = L (f(d') L µ(d)) .
O<dln d O<d'lnO<dl;ft" O<d'ln O<dl;ft"
(iii) n = pf 1 ••• p~k, com p 1 < · · · < Pk primos: como F é multiplica-
tiva, segue de (3.1) e do item (ii) que Agora, se !J, > 1 (i.e., se d'< n) a proposição 3.7 fornece
F(n) = F(pf 1 ) .•• F(p~k) = O. L µ(d)= o.
O<dl;ft"
• Portanto, o penúltimo somatório acima se reduz à parcela correspon-
dente a d' = n, de sorte que
O teorema 3.8 a seguir evidencia o papel central da função de Mõ-
bius na teoria das funções aritméticas multiplicativas. Para sua prova,
observe que a função L (f(d') L µ(d)) = f(n) L µ(d) = f(n)µ(l) = f(n).
O<d'ln O<dl;ft" O<dll
f: D(n) -t D(n)
(3,3)

0

d 1-------t n/ d
80 Funções Aritméticas Multiplicativas 81

Antes de prosseguir com o desenvolvimento da teoria, mostremos Definição 3.10. A função cp de Euler é a função cp: N-+ N dada
por um exemplo como a fórmula de inversão de Mobius pode ser uti- por
lizada como ferramenta de contagem. cp(n) = #{1 :s; k :s; n; mdc (k, n) = 1}.
Exemplo 3.9. Fixado n EN, dizemos que uma sequência (x 1 , x 2 , ... , Em palavras, cp( n) conta quantos inteiros de 1 a n são primos com
Xn), tal que x j E {O, 1} para 1 :s; j :s; n, é aperiódica se não existir n. No que segue, dentre outras propriedades, vamos mostrar que fun-
divisor O < d < n de n tal que a sequência seja formada pela justa- ção cp é multiplicativa e usar esse resultado para calcular cp( n) em
posição de J cópias do bloco (x 1 , ... , xd)· Calcule, em função de n, o função da decomposição canônica de n em fatores primos2 . Comece-
número de sequências aperiódicas de n termos. mos com um resultado que será útil em outras circunstâncias.

Prova. Note inicialmente que, pelo princípio fundamental da conta:- Proposição 3.11. Se cp: N-+ N é a função de Euler, então
gem, há exatamente 2n sequências (x 1 , x 2 , ... , Xn) tal que Xj E {O, 1}
para 1 :s; j :s; n. L cp(d) = L cp (~) = n.
O<dln O<dln
Por outro lado, para uma sequência (x 1 , x 2 , ... , xn) como no enun-
ciado, definimos seu período d como o menor divisor positivo de n tal Prova. A primeira igualdade segue da bijetividade da função f em
que a sequência seja formada pela justaposição de n/ d cópias do bloco (3.3). Para a segunda igualdade, seja D(n) = {1 = a 1 < a 2 < · · · <
(xi, ... , xd)· Em particular, uma sequência aperiódica (x 1 , x 2 , ... , xn) at = n }; se 1 :s; k :s; n, temos mdc (k, n) E D(n), i.e., mdc (k, n) = ai
!'' tem período n. para algum 1 :s; i :s; t; portanto, sendo Ai = {1 :s; k :s; n; mdc (k, n) =
Mais geralmente, se a sequência (x 1 , x 2 , ... , xn) tiver período d, ai}, segue que
então (x 1 , ... , xd) será aperiódica, e reciprocamente. Portanto, se ak ln= A1 U ... U At,
denota o número de sequências aperiódicas de k termos, temos
uma união disjunta e, daí, n = L1= IAil·
1 Note, agora, que

{1 :s; k :s; n; mdc (k, n) = ai}


{ 1 :s; k / ai :s; n /ai; k / ai E N e mdc (k /ai, n /ai) = 1}
Aplicando a fórmula de inversão de Mobius, obtemos então {1 :s; l :s; n/ai; mdc (l, n/ai) = 1},

de maneira que IAil = cp ( ! ) . Portanto,


A teoria desenvolvida até o presente momento nos permite intro-
duzir e estudar as principais propriedades de outra importante função
aritmética, conforme ensina a definição a seguir. 2 Para uma outra abordagem, veja o problema 2.1.6 do volume 4.

82 Funções Aritméticas Multiplicativas 83

Para o teorema a seguir, também devido a Euler, precisamos do Prova. Calculemos inicialmente o valor de <p(pª), com p primo e a ~
fato, deixado como exercício para o leitor (veja o problema 1), de que 1 inteiro:
o produto de duas funções aritméticas multiplicativas também é uma
função multiplicativa. <p(pª) #{1::; k::; pª; mdc (k,pª) = 1}
#{1::; k::; pª; mdc (k,p) = 1}
Teorema 3.12 (Euler). A função de Euler <p : N-+ N é multiplicativa. # ({1,2,3, ... ,pª} \ {p,2p,3p, ... ,pª- 1p})
pª - pª-1
Prova. Fazendo G(n) = n, a proposição anterior nos dá I:o<dln <p(d) =
G(n). Portanto, segue da fórmula de inversão de Mobius que pª ( 1-i).
<p(n) =L µ(d)G CD= L µ(d)·~= L µ~d).
O<~n O<~n
n
O<~n
(3.5)
Agora, como <pé multiplicativa, segue de (3.1) que

Mas como J(d) = µ,~) é uma função multiplicàtiva (verifique este


fato!), segue da proposição 3.4 que a função F : N -+ IR, definida por

F(n) = L µ~d)'
O<dln

também é multiplicativa. Logo, <p(n)


problema 1.
= nF(n) é multiplicativa pelo
• •
A fórmula do corolário anterior nos permite deduzir várias propri-
Conforme prometido anteriormente, no corolário a seguir relacio- edades interessantes (e úteis) da função <p de Euler. Colecionamos, a
namos <p(n) com a decomposição canônica do inteiro n > 1 em fatores seguir, um exemplo nesse sentido.
pnmos.
Exemplo 3.14. Dado x EN, mostre que a equação <p(y) = x admite
no máximo um número finito de soluções y EN.
Corolário 3.13 (Euler). Se a decomposição canônica do inteiro n > 1
·
em pnmos e, n = p a1 °'k t-
1 ... pk , en ao Prova. Primeiramente, se y = pª z, com a ~ 1, p primo e mdc (p, z) =
1, então o caráter multiplicativo da função <p garante que
<p( n) = n ( 1-: 1
) ... (1 - :k) . (3.6)
84 Funções Aritméticas Multiplicativas 85

uma vez que cp(z) ~ 1ep ~ 2. Portanto, tomando logaritmos na base o segundo estudante deveria fazer o mesmo em relação à equação
2, obtemos 1 1 1
a :::; 1 + log 2 x. X y n
Ademais, cálculos análogos aos acima fornecem Sabendo que a soma das respostas dos estudantes foi 78, mostre
que ao menos um deles cometeu um erro de cálculo.
X= pª- 1 (p - l)cp(z) ~ p - 1,
5. (Hungria - adàptado.) Para n E N e O :::; r :::; 3, seja Dr(n) o
donde p :::; x + 1.
conjunto dos divisores positivos de n que deixam resto r quando
Portanto, se y = pf 1 . . . p~k é a fatoração canônica de y em primos
divididos por 4.
e a = max {a 1 , ... , ak}, segue do que fizemos acima que a :::; 1 + log2 x
e, daí, (a) Se mdc (m, n) = 1, prove que existe uma bijeção natural

(D1(m) x D1(n)) U (D3(m) x D 3 (n))--+ D1(mn);


faça o mesmo para (D1(m) x D 3 (n)) U (D3(m) x D1(n)) e

• D3(mn) .
(b) Prove que ID1(n)I ~ ID3(n)I.
,11·11111

6. Um natural n > 1 é perfeito se s(n) = 2n, onde s(n) é a soma


dos divisores positivos de n. Prove que:
Problemas - Capítulo 3
(a) Se n > 1 é perfeito, então Eo<dln â= 2.
1. * Prove que o produto de duas funções aritméticas multiplicati-
(b) Se p é um primo tal que 2P - 1 também é primo, então
vas também é uma função multiplicativa.
2P- 1 (2P - 1) é perfeito.
2. Prove que, para todo n EN, temos Ilo<dln d= nd(n)/ 2 •
7. O propósito deste problema é estabelecer a recíproca do pro-
3. Prove que, para todo n EN, temos :~:~ ~ yn. blema anterior, provando o seguinte teorema de Euler: se n é
um número perfeito par3 , então existe um primo p tal que 2P - 1
4. (OCM.) Um professor escolheu um inteiro positivo n e, em se- é primo e n = 2P- 1 (2P- l). Para tanto, seja n = 2kq um número
guida, propôs a dois estudantes os problemas a seguir: o primeiro perfeito par, onde k, q EN e q ímpar. Faça os seguintes itens:
estudante deveria calcular o número de pares ordenados (x, y),
com x e y inteiros positivos, satisfazendo a equação (a) Conclua que (2k+1 - l)s(q) = 2k+1q e, a partir daí, mostre
que existe a EN tal que q = (2k+l - l)a e s(q) = 2k+Ia.
1 1 1
-+-
X y
= -;
n 3 Até hoje não se sabe se existem números perfeitos ímpares.
86 Funções Aritméticas Multiplicativas 87

(b) Se a= 1, mostre que 2k+1 - 1 é primo, donde k +1 = p, 12. * Seja f : N -+ ~ uma função qualquer e F : N -+ ~ a função
um primo. dada por F(n) = LO<dln f(d). Prove que
(c) Se a= 2k+1 _ 1, mostre que s(q) 2 1 +a+a2 > (a+ l)a = ,
2k+Ia, uma contradição.
(d) Se a > 1 e a -=/:- 2k+1 - 1, então q tem pelo menos quatro
divisores positivos distintos: 1, 2k+1 - 1, a e (2k+1 - l)a.
13. Seja f : N -+ {-1, 1} a função definida por f(l) = 1 e, para
Conclua, a partir daí, que s(q) > 2k+la, chegando a uma
n > 1 inteiro, f(n) = (-l)ª 1 +·+ 0 k, onde n = pf 1 .• • pik é a
nova contradição.
decomposição canônica de n em fatores primos. Prove que, para
8. Um natural n é abundante se s(n) > 2n. Se a for abundante, todo inteiro n 2 1, tem-se
mostre que ab é abundante, qualquer que seja b EN.

9. Seja f: N-+ ~ a função definida por f(l)

f(n) = (-l)k ,
= 1 e, para n > 1, t lYJ J(j) = Lv'nJ.

P1P2 · · ·Pk 14. (OBM). Prove que, para todo natural n > 1, temos
onde p 1 ,p2 , ... ,Pk são os primos distintos que dividem n. Ache
todos os n EN tais que Lo<dln J(d) = O. n ( -1 + -1 + · · · + -1) <
2 3 n
L d(k) ::; n
n (
1 + -1 + -1 + .. · + -1) .
2 3 n
k=l
10. Se f : N -+ ~ é uma função aritmética multiplicativa, prove que
z:
O<dln
µ(d)J(d) = II (1 -
pprimo
J(p)). 15. Seja F uma função aritmética multiplicativa e f : N -+ ~ a
função definida implicitamente por F(n) = Lo<dln J(d). Prove
Pin
que f também é uma função aritmética multiplicativa.
Em seguida, use esse resultado para provar os dois itens a seguir:
16. O objetivo deste problema é dar outra prova do caráter multi-
(a) LO<dln dµ(d) = npprimo(l - p). plicativo da função <p. Para tanto, para m, n > 1 inteiros primos
pln
(b) Lo<dln µ( d) 2 = 2k, onde k é o número de fatores primos entre si, arranje os naturais de 1 a mn na tabela
distintos de n (observe que k = O se n = 1).
1 2 k m
11. Prove o teorema de Liouville4 : para cada n EN, tem-se m+l m+2 m+k 2m
2m+l 2m+2 2m+k 3m
í:: d(j))' = L d(j)'.
(O<jln (n - l)m + 1 (n-l)m+2 (n - l)m + k mn
O<jln
4 Após Joseph Liouville, matemático francês do século XIX. e faça os seguintes itens:
88 Funções Aritméticas Multiplicativas 89

(a) Um inteiro da tabela é primo com mn se só se for primo (c) Se m for ímpar, então
com me com n.
(b) Em uma coluna qualquer, ou todos os elementos são primos
com m ou nenhum é primo com m.
(c) Há exatamente cp(m) colunas formadas de inteiros primos Conclua, a partir daí, que, n I Sm (n) se m for ímpar.
com m, cada uma delas contendo exatamente cp(n) inteiros
21. Nas notações do enunciado do problema anterior, prove os se-
primos com n.
guintes itens:
(d) Conclua que cp(mn) = cp(m)cp(n).
(a) Todo 1 ~ m ~ n pode ser unicamente escrito da forma
17. Se F (n) = Eo<dln cp~d) , calcule F (n) em termos da decomposição. m= ~ · a, com a, d E N tais que d I n e mdc (a, d) = 1.
canônica de n. (b) L..,Q<dln
~ Sk(d) _
dk -
1 k+2k+ .. +nk
nk ·

:li,:i'.•
18. Para cada m E N, seja Am o conjunto dos pares ordenados (c) Sk(n) = nk Í:o<dln µ (~) (1k+2kJ·+dk).
l
,li, (d, n). tais que d é um divisor positivo de m, 1 ~ n ~ m e
(d) S 1 (n) = !ncp(n).
li! mdc (d, n) = 1. Ache todos os m EN tais que IAml = 1993.
'11 (e) S 2 (n) = }n2 cp(n) + !n Tipprimo(l - p).
! pln
19. * Para n > 2 inteiro, prove os itens a seguir:
(a) Se Pn = {1 ~ k ~ n; mdc (k, n) = 1}, então a correspon-
dência d H n - d é uma bijeção de Pn.
(b) cp( n) = 2l, onde l é o número de elementos de Pn menores
ou iguais a n;1; em particular, cp(n) é par.

20. Dados m, ri E N, com n > 2, sejam 1 = a 1 < · · · < ak = n - 1


os inteiros positivos primos com n e menores ou iguais a n, e
Sm(n) = I::7=
1 a'f a soma de suas m-ésimas potências. Prove
os seguintes itens:

(a) Se k = 2l, então Sm(n) = E!=l (a'f + (n - ai)m). Conclua,


a partir daí, que Sm (n) é par se n o for.
(b) Sm(n) = E7F=0 (-l)i (7)nm-j Sj(n).
90 Funções Aritméticas Multiplicativas
1'

CAPÍTULO 4

Cálculo e Teoria dos Números

Neste capítulo, pressupomos que o leitor tenha conhecimentos rudi-


mentares de Cálculo, mais precisamente familiaridade com sequências
e séries convergentes, limites de funções e derivadas, sendo o material
do volume 3 desta coleção suficiente para nossos propósitos. Apre-
sentaremos alguns exemplos e resultados básicos sobre a distribuição
dos números primos ao longo dos naturais, seguindo essencialmente o
clássico [2], bem como um interessante resultado assintótico de Cesàro,
sobre pares de naturais primos entre si, seguindo o artigo [6].

4.1 Sobre a distribuição dos primos


Um dos primeiros resultados que aprendemos sobre números pri-
mos neste livro foi o teorema de Euclides, o qual garante a existência
de infinitos números primos. Estes, contudo, não se distribuem de

91
92 Cálculo e Teoria dos Números I 4.1 Sobre a distribuição dos primos 93

maneira uniforme ao longo dos naturais. O teorema a seguir, conhe- , Em palavras, (4.1) significa que a quantidade de números primos
ciclo como o teorema do número primo, torna essa afirmativa mais , menores ou iguais a x é um infinitésimo em relação a x, i.e., cresce
precisa. Antes de enunciá-lo, fixemos uma notação: para cada real bem mais lentamente que x, à medida que x --+ +oo. No que se-
positivo x, denotemos por 1r(x) o número de primos menores ou iguais .i gue, deduziremos a validade desse resultado diretamente, para o quê
a X. precisamos, inicialmente, do seguinte resultado.
Teorema 4.1 (Hadamard1 ). Nas notações acima, temos Proposição 4.2. Se k é um natural qualquer, então
lim 1r(x) = 1 1r(x) <p(k) 2k
x--++oo X j log X ' --<--+-
X k X'
onde log : (O, +oo) --+ R denota a função logaritmo natural.
onde <p é a função de Euler.
A prova do teorema acima foge largamente ao escopo destas notas ,
1·.,1
mas, para o leitor interessado, recomendamos o clássico [3]. Ressal- ' Prova. Seja x > O real, com lxJ = kq + r, onde O Sr< k, e note
tamos, contudo, que o teorema de Hadamard garant~ que as .funções que
;x
1r(x) e 10 são assintoticamente (i.e., à medida que x--+ +oo) iguais. e

{ 1, 2, ... , lX j } = LJJ:~ {kj


+ 1, kj + 2, ... , k (j + 1)}
Anteriormente ao teorema de Hadamard, P. Chebyshev2 obteve um , (4.2)
resultado bem mais simples (mas, ainda assim, muito interessante), U {kq + 1, kq + 2, ... , kq + r }.
garantindo a existência de constantes positivas e e C tais que
De 1 a k não há mais do que k números primos, obviamente. Por
X X
e - - S 1r(x) S C--, outro lado, se j ~ 1, então, para um número do conjunto {kj + 1, kj +
logx logx
2, ... , k(j + 1)} ser primo, tal número deve, necessariamente, ser primo
para todo x ~ 2. com k. Portanto, não há mais do de <p(k) números primos no conjunto
A obtenção da segunda desigualdade acima será o objeto do pro- {kj + 1, kj + 2, ... , k(j + 1)}. Agora, como q = líJ,
(4.2) permite
blema 4; para uma prova da primeira, recomendamos ao leitor o ex- estimar 1r( x) pela desigualdade
celente livro de G. Andrews ([2]). Por outro lado, segue prontamente
da segunda desigualdade que 1r(x) :::; k + (q - l)cp(k) + r < 2k + lfJ <p(k) S 2k + f
<p(k).
1r(x) < _!!__
x - logx' Para obter o resultado do enunciado, basta dividirmos ambos os mem-
bros da desigualdade acima por x. •
para todo real x ~ 2; em particular, temos

lim 1r(x) = O. Mesmo tendo sido obtida por intermédio de uma contagem bas-
(4.1)
x--++oo X tante simples, a proposição anterior permite mostrar que 1r(x) é, quando
1 Após Jacques Hadamard, matemático francês do século XIX. x--+ +oo, um infinitésimo em relação a x. Lembremos, inicialmente,
2 Após Pafnuty Chebyshev, matemático russo do século XIX. o seguinte resultado (cf. exemplo 3.20 do volume 3).
94 Cálculo e Teoria dos Números 4.1 Sobre a distribuição dos primos 95

Lema 4.3. A série harmônica Ln:::::i ~ diverge. Prova. Temos de provar que, dado E > O arbitrariamente, existe x 0 >
O tal que x > Xo ::::} ~~) < E. Para tanto, sejam P1,P2, ... ,Pn os
Precisamos de mais um lema técnico. n primeiros números primos e k = P1P2 ... Pni a proposição 4.2 e a
fórmula para cp(k) nos dão
Lema 4.4. Sem> 1 é inteiro e p 1 ,p2 , ... ,Pk são os primos menores
ou iguais a m, então 1r(x) <
X
(l ___!_) (l ___!_) ... (l _~) +
Pl. P2 Pn
2P1P2 .. · Pn.
X
(4 .3 )

Agora, os dois lemas anteriores garantem que

Prova. Pela fórmula para a soma dos termos de uma série geométrica
lim
n~+oo
[(1 - __!_) ... (1 - ~)]-l
~ ~
= +oo,

convergente (cf. proposição 3.21 do volume 3), temos


de sorte que
lim
n~+oo
(1 - __!_) ... (1 -~)
PI Pn
= O.

Podemos, pois, escolher um natural n tal que

Por outro lado, a escolha de P1, P2, ... , Pk, juntamente com o E
<-
teorema fundamental da aritmética, garante a validade da inclusão 2

e, a partir daí, pôr x 0 = 4P 1P~··-Pn De posse de tais escolhas, para


1 1}
{ 1,2,···,m e {
r1i1rJ.i2 1... p7,/'. . . · >
J1,J2,···,Jk_
o} . x > x 0 temos, a partir de (4.3), que

1r(x) E 2P1P2 · · · Pn E 2P1P2 · · · Pn


-- < - + <- + = E.
Portanto, X 2 X 2 ~

Heuristicamente, podemos dizer que Ln>I ~ diverge por ter muitos



naturais (na verdade, todos eles) dentre se~s termos. Por outro lado,
e o resultado desejado segue imediatamente. • a proposição 3.22 do volume 3 garante que
1
De posse dos resultados acima, podemos provar a seguinte versão L
n:::::1
n2 < +oo; (4.4)
fraca do teorema de Hadamard.
portanto, também de um ponto de vista heurístico, podemos atribuir
.
Teorema 4.5. 1Imx-t+oo----;;-
~(x) -
-
O.
tal convergência ao fato de que há poucos naturais dentre seus termos.
96 Cálculo e Teoria dos Números 4.1 Sobre a distribuição dos primos 97

' 1
Surge, então, uma pergunta natural no contexto de números primos: diverge. Por fim, suponha que a série dos inversos dos primos convirja
a série En;:::,:l P~, onde Pn é o n-ésimo primo, tem muitos ou poucos para um certo real a. Então, para todo n EN, temos, a partir do lema
naturais, no sentido acima? O teorema 4.5 nos encorajaria a dizer que anterior, que
tal série possui poucos naturais. Mas nossa intuição falha nesse ponto,
pois, conforme mostraremos a seguir, a série En>l ....!... é divergente.
- Pn
Antes, contudo, precisamos de um resultado auxiliar.
Lema 4.6. Para todo x > O tem-se ex> 1 + x.
Prova. Imediato a partir do exemplo 3.9 do volume 4. onde exp : lR. --+ lR. denota a função exponencial de base e. Mas isto
é um absurdo, pois já sabemos que a soma dos inversos dos naturais
Estamos, agora, em condições de apresentar mais um importante
livres de quadrados diverge. •
resultado devido ao grande L. Euler.
''
1
Teorema 4. 7 (Euler). Se Pk denota o k-ésimo primo, então a série
Ek>l ...!.. é divergente.
- Pk

Prova. Se m E N e p 1 , p 2 , ... , Pk são os primos menores ou iguais a Problemas - Seção 4.1


m, é imediato que
l. Prove que existe um real positivo x0 tal que 1r(x) > 1~~x para

( L ~1) ( + L
15,j<vm J
1 L . .1 .
15:i5:k 15:11 <···<íi5:k P11P12 ... P1i
) 1
2:: L -:-
m
í=l J
X> Xo.

2. Para cada k 2:: 1, seja ak o k-ésimo natural que não é quadrado


Observe, agora, que o segundo fator do primeiro membro da desigual- perfeito. Decida se a série Ek;:::,:i ª1k converge.
dade acima é simplesmente a soma dos inversos dos naturais livres de
3. Para cada k 2:: 1, seja ak o k-ésimo natural composto. Decida
quadrados e menores ou iguais a m, a qual será denotada, doravante,
se a série Ek>l ...!.. converge.
j ldq

por z:= 1 '.'Õi'.'Õm Portanto, temos, a partir da desigualdade acima, que - ªk

4. O objetivo deste problema é mostrar que

( L -P1) (L -i1)· >L-·


15,j<vm -
1
i
1f{f;
m
j=l
1r(x) ::; (30 log 2)_:._,
logx
(4.5)

para todo real x 2:: 8, onde log : (O, +oo) --+ lR. denota a função
Como Z::::1::::: 1 y diverge e Z::::1::::: 1 ] 2 converge, concluímos que a
logaritmo natural. Para tanto, faça os seguintes itens:

L; J
(a) Para todo n E N, prove que (2;) é divisível por todos os
primos p tais que n < p ::; 2n. Ademais, (2;) < 22n.
">1
J_
j ldq
Cálculo e Teoria dos Números O teorema de Chebyshev 99
98

(b) Prove que, para n 2 2, tem-se 1r(2n) < 1r(n) + (2log2) 10 ;n. (d) Use o fato de que t = Llog2 n J :::; n para obte a estimativa

(c) Se f: (O, +oo)--+ lR é a função dada por f(x) = 10


1
mos- :x' I(n) < 4n2 + 15n + 3
- 12 .
tre que f é crescente em (e, +oo) e que f (x! ) < Í f(x),
2

para x 2 8. (e) Utilize as desigualdades L1og 2 nJ 2 Oe 2liog2nJ 2 2Iog2n-1 =


(d) Use os itens (b) e (c) para concluir que1r(2n) < (32log2) 10 ;n, !! e
2
para n 2 2. . 7(qk) >
- qk - 1 = l!!_J
2k - 1 >
- !!_
2k - 2
(e) Deduza que, para todo número real x 2 8, vale (4.5).
para mostrar, de maneira análoga, que
5. O objetivo deste problema é estabelecer a recíproca do resul- · I(n) > 4n2 - 12n - 1
tado do problema 17, página 49. Para tanto, dado n E N, seja 1 - 12 .
I (n) a soma dos maiores divisores positivos ímpares dos números ,
(f) Se T(n) = 1 + 2 + · · · + n, conclua que
1, 2, ... , n e faça os seguintes itens:
4n 2 - 12n - 1 J(n) 4n2 + 15n + 3
(a) Para n E N, sejam 7(n) o maior divisor ímpar de n e i(n) ----- < -- < -----
6n2 + 6n - T(n) - 6n2 + 6n
o maior ímpar menor ou igual a n. Prove que
e, a partir daí, que, se r E (Q \ { i}, então ~~~) = r para no
I(n) = (7(1) + 7(3) + · · · + 7(i(n))) máximo um número finito de valores de n.
+ (7(2) + 7(4) + · · · + 7 (2Ln/2J))
. 1
= 4(i(n) + 1) 2 + I Ln/2J). 4.2 O teorema de Chebyshev
(b) Nas notações de (a), mostre que O teorema do número primo garante que a diferença 1r(2x) - 1r(x)
é assintoticamente igual a
1 t
I(n) = 4 I::(i(qk) + 1)2, 2x x x (logx - log2)
k=O log(2x) - logx = logx logx + log2 ·

onde 2t é a maior potência de 2 menor ou igual a n e, para , Mas, como


O :::; k :::; t, qk é o quociente da divisão de n por 2k. .
11m log x - log 2
= 1
x-++oo log x + log 2 '
(c) Use o fato de que i(qk) :::; ; para obter a estimativa
concluímos que 1r(2x) - 1r(x) é assintoticamente igual a __!!___.
logx Ocorre

J(n) < !
- 4
(n (4 - _!_) (4 --1)
3
2
4t
+n
2t-l
+t+ 1) . que
.
11m X
- - = +oo,
x-++oo logx
100 Cálculo e Teoria dos Números 4.2 O teorema de Chebyshev 101

de sorte que o argumento acima garante que o número de primos entre


x e 2x cresce sem limite, à medida que x-+ +oo.

Lema 4.9. Se x 2 2 é um número real, então
Nesta seção, provaremos um resultado bem mais fraco que este,
também devido a Chebyshev, o qual garante que sempre há pelo menos
um número primo entre n e 2n, para todo inteiro n > 1. Para a p<x
p primo
demonstração do mesmo, precisamos dos dois resultados auxiliares a
seguir. Prova. O lema é claramente válido para 2 :::;; x :::;; 3. Por outro lado,
Lema 4.8. Sejam n, p E N, sendo p primo, e µP o expoente da maior se ele for válido para x = n, onde n 2 3 é um inteiro ímpar, então ele
potência de p que divide o coeficiente binomial (2:). Então também será verdadeiro para n:::;; y < n + 2; de fato, nesse caso n + 1
é par, de sorte que

P~Y p~n
Ademais, se Vp é o único inteiro tal que pvp :::;; 2n < pvp+l então µP. :::;; vP. p primo p primo

Prova. Para a primeira parte, veja que, como (2:) = a maior i~~};, Basta, então, mostrarmos que o lema é verdadeiro para x = n,
pot encia d e p que dºlVlºd e (2n) e, p
Pep(2n) d ( f - ) ( ) onde n 2 3 é um inteiro ímpar. Para tanto, façamos indução sobre
2 ep(n), on e C . seçao 1.3 ep 2n e
A •
1
n
ep(n) denotam, respectivamente, os expoentes das maiores potências n 2 3 inteiro ímpar, observando que já temos a validade do resultado
1 ,, '

de p que dividem (2n)! e n!. Portanto, para n = 3. Assuma, como hipótese de indução, que ele também é
válido para todos os inteiros ímpares menores que um certo inteiro
11,
1

µP = ep(2n) - 2ep(n) ímpar n 2 5. Defina k = n~l, onde escolhemos o sinal de tal modo
e basta usar a proposição 1.48 para obter a fórmula do enunciado. que k seja ímpar. Então, k 2 3 e n - k é um natural par, tal que
Para a segunda parte, segue da definição de vP que
= 2k =f 1 - + 1.
l~J- l; J
n - k k :::;; k
j > Vp ==} pl > 2n ==} 2 = 0.
Daí, se pé um primo tal que k < p:::;; n, então pé ímpar e p I n!, p f k!
Por outro lado, para j 2 1 temos e p f (n - k)! (não pode ser n - k = k + 1 = p, uma vez que n - k é

l
2nj _ 2
pJ
ln_J
pJ
< 2n _ 2 (
pJ
n__ 1)
pJ
= 2
'
par). Concluímos, então, que o produto de todos esses primos divide

n!
de sorte que l!~ J- l; J : :;
2 1, para j 2 1. Portanto, segue dos fatos
(;) k!(n - k)!'
acima que de modo que

:'.S;
1/p

L 1= Vp·
II
k<p~n
p:::;; (;). (4.6)
j=l p primo
102 Cálculo e Teoria dos Números 4.2 O teorema de Chebyshev 103

Agora, nossas escolhas garantem que k e n - k são distintos. Mas Mas, para qualquer primo p tal que 2; < p s n, temos
como G) = (n:k) e tais números binomiais são parcelas da expansão
2 2 n 3 2n
binomial de 2n = (1 + ir,
segue que (~) S 2n-l e (4.6) fornece p ~ 3, p > 3np, 1 S p
< 2, 2 S <3 p

k<p$n
(4.7) e, daí, !~ < ! S 1; segue que
(l; J-2 l; J) = l:J-2 l~J = 2- 2 = O
p primo

Portanto, segue da hipótese de indução e de (4. 7) que µ, =~

Por outro lado, para qualquer primo p tal que ffn < p s
2; , temos
II p = ( II p) ( II p) < 4k. 2n-l = 22k+n-1 S 22n = 4n_ p2 > 2n e segue, do lema 4.8, que 1 S µP S vP = 1. Finalmente, para
p$n p$k k<p$n
p primo p primo p primo os primos p S ffn, temos, novamente pelo lema 4.8, que pµP pvp s s

Chegamos, por fim, ao resultado prometido, . conhecido como o


• 2n .
As estimativas acima nos dão (no que segue, p denota um primo)

teorema de Chebyshev.
Teorema 4.10 (Chebyshev). Para cada inteiro n > 1, há ao menos J
um primo entre n e 2n.
Prova. Faremos um argumento geral para mostrar o resultado para (4.8)
n ~ 128. Para n < 128 tome p = 3 para n = 2, p = 5 para n = 3 e

p = 7 para 4 S n S 6;
p = 13 para 7 S n S 12;
p = 23 para 13 S n S 22;
onde utilizamos o lema 4.9 na desigualdade acima. Por outro lado,
p = 43 para 23 S n S 42;
uma vez que 9 e os números pares maiores que 2 não são primos, a
p = 83 para 43 S n S 82;
condição n ~ 128 ({::} ffn ~ 16) garante que
p = 131 para 83 S n S 127.
Suponha, agora, que o resultado seja falso para algum n ~ 128. 1r
r,:;-:-) S ffn -
( v2n 1
-1.
2
Nas notações do lema 4.8, tal suposição garante que
Substituindo essa última estimativa em (4.8), chegamos a
(2:) = II
p$2n
pµP = II
p$n
~P.

( 2: ) < 4 2; (2nyrr(ffn) < 4 2an (2n)~- 1. (4.9)


p primo p primo
104 Cálculo e Teoria dos Números 4.3 O teorema de Cesaro 105

Por fim, note que (2;) é a maior dentre as 2n + 1 parcelas do 4. Ache todos os m, n, x, y EN tais quem, n > 1 e (m!)x = (n!)Y.
desenvolvimento binomial de 4n = (1 + 1) 2n. Como a primeira e a
última de tais parcelas são iguais a 1, temos 2n(2;) > 4n ou, ainda,
Para os dois problemas a seguir, o leitor achará conveniente uti-
( 2n) > 4 n estimativa que, combinada com (4.9), fornece
n 2n' lizar a seguinte versão forte do teorema de Chebyschev: para
todo inteiro n 2:: 6, há pelo menos dois primos entre n e 2n.
4n < 4 2; (2n)~-1
2n
5. Ache todos os m, n EN tais que (n - l)!n! = m!.
ou, ainda,
2 2; < (2n)~. 6. Encontre todos os naturais n > 1 tais que todo natural 1 <
m < n que seja relativamente primo com n seja, em verdade,
Tomando logaritmos naturais e dividindo por ~, a desigualdade um número primo.
acima pode ser reescrita como

V8nlog2 - 3log(2n) < O. . (4.10) O teorema de Cêsaro


4.3
Denotando f(x) = \1'8xlog2-3log(2x), vem que !(128) = 8log2 > O ,.. Em 1881, E. Cesàro 3 provou que a probabilidade de dois núme-
e ros naturais, escolhidos aleatoriamente, serem relativamente primos, é
f '( X ) = y12xlog2
X
- 3 0
> igual a ; 2 • Terminamos este capítulo apresentando, nesta seção, uma
demonstração elementar deste resultado, devido ao professor Hudson
para x 2:: 128. Assim, f é crescente para x 2:: 128 e (4.10) é falsa, o i
N. Lima (cf. [6]). Comecemos com alguns preliminares sobre o cálculo
que nos dá uma contradição. •
de probabilidades.
Sejam E um conjunto finito e não vazio, dito o espaço amostral,
e Puma distribuição de probabilidades em E, i.e., uma função
P : E ---+ [O, 1] tal que
Problemas - Seção 4.2 LP(x) = 1.
xEE
1. Se Pn é o n-ésimo primo, prove que Pn+l < 2Pn· Para cada x E E, denominamos P(x) a probabilidade de x e dizemos
que os elementos de E são equiprováveis se
2. * Prove que, para todo inteiro n > 1, existe um primo p cujo
expoente na decomposição canônica de n! em primos é igual 1. 1
P(x) = IEI' V x E E.
3. (Torneio das Cidades.) Prove que, para todo inteiro n > 1, o
número 1!2! · · · n! não é um quadrado perfeito. 3 Ernesto Cesàro, matemático italiano do século XIX.
106 Cálculo e Teoria dos Números 4.3 O teorema de Cesaro 107

Um evento em E é um subconjunto não vazio X de E, sendo sua Lema 4.11. Se f : N -+ lR é uma função qualquer, então, para n
probabilidade definida por natural, temos:

P(X) = L P(x).
xEX (a) Í::~=1 Í:o<dlk f(d) = Í::~=1 f(k) liJ ·
Voltando, agora, a nosso problema, seja4 E= ln x ln e suponha
que os elementos de E são equiprováveis, de sorte que a probabilidade
de cada um deles é igual a ,;2 • Dado n E N, se Pn é a probabilidade
de um par ordenado (a, b) E E ter suas entradas a e b primas entre si, Prova. O item (a) é o conteúdo do problema 12, página 87. Quanto
queremos mostrar que ao item (b), observe inicialmente que o conjunto dos pares ordenados
lim Pn = 6 . (d, k) de inteiros tais que 1 :::; k :::; n e O < d I k coincide com o
2
n-too 7f
conjunto dos pares ordenados (d, k) de inteiros tais que 1 :::; d :::; n e
Para tanto, devemos, primeiramente, encontrar uma expressão ade-
k = ld, para algum 1 :S l :S n/ d. Portanto, podemos trocar a ordem
quada para Pn e, para tal fim, começamos observando que Pn _= l~I,
dos somatórios envolvidos, obtendo
onde
X= {(a,b) E ln X ln; mdc(a,b) = 1} n n

é o evento de nosso interesse. I:k I:: J(d) I:: I:: kJ(d)


k=l O<dlk d=l k=ld
Uma contagem elementar fornece 1:9::;;}
n

1:,
IX! = 2#{(a, b) E ln
-#{(a,a)
X

E ln X ln;
ln; mdc (a, b)
mdc(a,a)
= 1 e a :S b}
= 1}
I:: J(d) . ( d + 2d + ... + !!_ J d)
d=l d
l
~t l~J (l~ J + 1) .
n
2L#{a E ln; mdc(a,b) = 1 e a :S b}-1 df (d)
b=l d=l
n
2I::cp(b)-l,
b=l

onde cp é a função de Euler. Portanto,
n
Lembre-se, agora, de que, de acordo com (3.5),
2I::cp(b)- l
b=l
Pn= -----
n2
(4.11) cp(n) = n L µ~d)'
O<dln
Precisamos, agora, do seguinte resultado auxiliar.
4 Como no volume 4, pomos In = {1, 2, ... , n }. onde µ : N -+ lR é a função de Mobius. Portanto, o item (b) do lema
108 Cálculo e Teoria dos Números 4.3 O teorema de Cesaro 109

anterior fornece Proposição 4.12. Se Pn é definido como acima, então limn--+oo Pn


existe e é tal que
2tcp(k) =2tk I: µ~d) 1.1m .Ln
n--+oo
D = L µ(k)
k2 .
k=l k=l O<dlk k::::1

= tµ(k) l~J (l~J + 1) (4.12) Prova. Inicialmente, obtemos a partir de (4.13) que

=
k=l
n 2
I: µ(k) l~J + I: µ(k) l~J ·
n
P. -t. µi:) - t.µ(k) G, lIJ' -:, )
k=l k=l
(4.14)
n 11 1 21
A segunda parcela da última soma acima pode ser calculada com :S ~ k2 - n2 l~ J ·
o auxílio do item (a) do lema anterior, juntamente com o resultado da
A fim de estimar o último somatório acima, afirmamos que, para
proposição 3.7. De fato, temos
n e k naturais, com 1 :S k :S n, vale
n n
I: µ(k) l~J = I: I: µ(d) = ·1,
k=l k=l O<dlk
De fato,
uma vez que Eo<dlk µ( d) só não vale O para k = 1. Segue, pois, de
n
(4.11) e (4.12) que <-
- k
=}

1 n
Pn =
n
n2 Lµ(k)
k=l
lkJ 2
(4.13)

De posse da fórmula acima, podemos enunciar e provar a proposi-


ção a seguir, a qual garante que limn--+oo Pn existe e o expressa como conforme desejado.
um outro limite que, conforme veremos, pode ser efetivamente calcu- Voltando a (4.14), obtemos, a partir da estimativa acima, que
lado. Para o enunciado da mesma, observe inicialmente que, como
Pn - ~ µ(k) < ~ (_3__ - __!_) 3_ ~ _!_
f
lµ(k)I :S 1 para todo k EN e a série Ek:::: 1 2 é convergente, as propo- . L k2 L nk n2
=
nL k
- _!__
n
k=l k=l k=l
sições 3.25 e 3.29 do volume 3 garantem que a série
Mas, a partir do exemplo 6.47 do volume 3 e da regra de l'Hôspital
(cf. problema 6.2.6 do volume 3), temos que

também é convergente.
-n2 Ln -k1 < -n2 ( 1 + Jn -dt
1 )
t1
=
2
-(log
n
n + 1) ---+ O
k=l
r
1

110 Cálculo e Teoria dos Números 4.3 O teorema de Cêsaro 111

quando n --+ +oo, onde log : (O, +oo) --+ R denota a função logaritmo Prova. A primeira fórmula de de Moivre (cf. proposição 1.6 do volu-
natural. Portanto, me 6) nos dá

1 1) sen (nO) Irn(( cos O+ i sen Ot)


lirn (
n--++oo n

e nossos cálculos acima garantem que


-I:---
2 n

k=l
k n

Im { t, (;);;( cos 8)"-; (sen O)i}

Ln-1 J

lirn
n--++oo
(Pn - t
k=l
µ(k))
k2
= O.
e, daí,
t(-ll(
k=O
n )(cosO)(n-2k-l)(senO)C2k+l)
2k + 1

Por fim, segue, daí, que sen (nO)


(sen O)n
t
Ln-1 J

k=O
(-1t( n )(ctgo)Cn-2k-1),
2k + 1
lirn lirn (Pn - ~ µ(k)) + ~ µ(k) = ~ µ(k):
Pn = n--+oo sempre que sen O -=/=- O.
n--+oo L....,; k2 L....,; k2 . L.....J k2
k=l k2':1 k2':1 Para n = 2m + 1, ternos

• sen((2m+l)O) = ~(-l)k(2m+l)(ct 0)(2m-2k)


(sen 0)( 2m+1) L....,; 2k + 1
k=O
g ·
A proposição anterior reduziu a prova do teorema de Cesàro a
Assim, definindo
mostrarmos que
~µ(k) = ~
L.._.; k2 7r2' f(x) = I)-1t(2m+ l)xm-k,
k2':1 k=O 2k + 1
o que faremos com o auxílio dos três resultados seguintes. mostramos que
A prova do teorema a seguir é devido a A. Yaglorn e I. Yaglorn
f(( ctg0)2) = sen ((2m + 1)0)
(cf. capítulo 8 de [1]) e utiliza alguns fatos elementares sobre números (sen 0)(2m+1) ·
complexos e raízes de polinômios. Para urna discussão autocontida, re-
Definindo Ok = ( 2:+1)1r, para k = 1, 2, ... , m, é imediato que
ferimos o leitor aos capítulos 3 e 4 do volume 6 desta coleção. Para urna
( ctg 01)2, ( ctg 02)2, ... , ( ctg Om) 2 são m raízes distintas de f. Mas,
prova alternativa, utilizando a teoria elementar de séries de Fourier,
corno f tem grau m, concluímos que essas são todas as raízes comple-
recomendamos o capítulo 1 de [4] ou o capítulo 2 de [10].
xas de f, de sorte que as relações de Girard entre raízes e coeficientes
Teorema 4.13. de um polinômio (cf. proposição 4.6 do volume 6) fornecem
m(2m -1)
3
112 Cálculo e Teoria dos Números 4.3 O teorema de Cêsaro 113

ou, ainda, Teorema 4.14. Se Pn é o n-ésimo número primo, então

(csc (Ji) 2 + · · · + (csc 11m) 2 (1 + ( ctg 01)2) + · · · + (1 + ( ctg Om)2)


m + (( ctg01) 2 + · · · + ( ctg0m)2)
m(2m - 1) 2m(m + 1)
- m+ 3 = 3 · Prova. Como Pn > n para todo n E N, temos

Por fim, como Oi E (O, i), para 1 :::; j :::; m, as desigualdades (6.8) n 1
"""'-:::;
~~ p2
1
( 1+-+···+-
.
p~
... 1) (1+-+···+-
1 1)
p2 p~
do volume 3 garantem que k=l 1 1 n n

para cada l 2: 1. Além disso,

para 1 :::; j :::; m e, daí, que 1 1) ( 1 1) Pi ...p!, 1


(
1 + p2 + · · · + p2l · . . 1 + p2
1 1 n
+···+ p2ln :::; L
k=l
k2

para cada l 2: 1, posto que cada parcela obtida a partir do desenvol-


para 1 :::; j :::; m. Portanto, vimento do primeiro membro aparece no segundo membro.
Mas, dado que

ou, o que é o mesmo, temos

m(2m - 1)
---~<
Lm (2m + 1) 2 < 2m(m + 1)
.
3 - . 1r2 • j2 - 3
J=l
Fazendo l ---+ +oo e observando (cf. proposição 3.21 do volume 3)
Multiplicando as desigualdades acima por (2; ;1)2, fazendo m ---+ +oo que
e observando que
1 1 ) = ( 1--
lim ( 1+-+···+-
p; p;l
1
p;
)-l '
lim m(2m - 1) _ lim 2m(m + 1) _ ! 1-too

m-too 3(2m + 1)2 m-too 3(2m + 1)2 - 6' obtemos

obtemos o resultado desejado com o auxílio do teorema do confronto.


"""'-<
n 1 ( 1 )-l ( 1 )-l <"""'- 1
• ~
k=l
k2 - 1 - -2
P1
... 1 - -
2
Pn
- ~
k2'.1
k2
114 Cálculo e Teoria dos Números 4.3 O teorema de Cêsaro 115

ou, ainda, Como Pn > n para todo n E N, temos An Ç Ãn C Ap1p2 ...pn e

I: 1 II
n
k2 ::;
n (
1- 2
1 )-1 ::; I: 1 k2 ·
Bn Ç Bn Ç Bp 1p2 ...pn; logo, valem as desigualdades
k=l k=l Pk k21
an -<ãn <a
- P1P2···Pn e bn <b
- n <b
- P1P2···Pn .
O resultado segue, novamente, do teorema do confronto.

Teorema 4.15. Se Pn é o n-ésimo número primo, então


• Portanto, uma vez mais a partir do teorema do confronto, obtemos

lim an = lim ãn e lim bn = lim bn,


lim IIn (1- ~) = ~ µ(k). n---+oo n---+oo n---+oo n---+oo
n---+oo p2 L.....J k2
k=l k k21 de sorte que
Prova. Em tudo o que segue, convencionamos que l < m ==} i1 < Ím ..
Para n EN, sejam: Ano conjunto dos números da formapi 1 Pi2 •• ·Pi2 . , lim
n---+oo
IT
k=l
(1- ~)
P2k
lim ãn - lim bn = lim an - lim bn
n---+oo n---+oo n----too n---+oo
onde s E N, Pii, Pi 2, ... , Pi 2• são primos e Pi 1 Pi2 .•• Pi2 • ::; n; Ãn o
conjunto dos números da forma PiiPi 2 ••• Pi 2., onde ~ E N, Piu Pi 2,
... , Pi2 • são primos e i1, i2, .•• , i2s ::; n; Bn o conjunto dos núme-
ros da forma PiiPi2 ••• Pi2 s+u onde s E N, Piu Pi2 , ••• , Pi2 .+i são pri-
mos e Pi 1 Pi2 ••• Pi2.+ 1 ::; n; Bn o conjunto dos números da forma
PiiPi2 ••• Pi2.+i, onde s E N, Pii, Pi2 , ••• , Pi2.+i são primos e i1, i 2 , ••• , i 2s+l;
conforme desejado.

Podemos, finalmente, juntar os resultados acima para calcular
n; Sejam, ainda, 1

lim Pn ~ µ(k) = lim IIn (1 - ~)


n---+oo L.....J k 2 n---+oo p2
k21 k=l k

É imediato verificar que

µ(k) -
.
an - bn = L~n

k=l
e ãn - bn = II
n

k=l
(
1- 2
1)
·
Pk

Além disso, os limites lim an, lim bn, lim ãn e lim bn todos existem,
pois as sequências correspondentes são não decrescentes e limitadas
f
superiormente por E%': 1 2 • Em particular, esse argumento garante a
existência do limite
lim IIn (1 - ~) .
n---+oo P2k
k=l
116 Cálculo e Teoria dos Números

CAPÍTULO 5

A Relação de Congruência

Introduzimos ao leitor, neste capítulo, a importante relação, em


Z, de conguência módulo n > 1. Nosso objetivo central é provar
um famoso teorema de P. de Fermat, conhecido na literatura como o
pequeno teorema de Fermat, bem como sua generalização igualmente
famosa, devido a Euler e conhecida como o teorema de Euler. A per-
vasividade desses resultados em teoria elementar dos números se deve,
dentre outros, ao fato dos mesmos iniciarem um estudo sistemático do
comportamento dos restos da divisão das potências de um natural a
por um natural n > 1 fixado, no caso em que a e n são relativamente
primos. De fato, de certa forma, tal estudo será nosso objeto principal
de investigação a partir de agora, sendo retomado explicitamente no
capítulo 7. Apresentamos, também, o não menos famoso teorema chi-
nês dos restos, o qual possui muitas aplicações interessantes em teoria
elementar dos números.

117
118 A Relação de Congruência 5.1 Definições e propriedades básicas 119

5.1 Definições e propriedades básicas Proposição 5.3. Sejam a e n inteiros dados, com n > 1.

O objeto central de estudo nesta seção é a relação entre números (a) Se a deixa resto r na divisão por n, então a - r (mod n). Em
inteiros explicitada na definição a seguir. particular, todo inteiro é congruente, módulo n, a exatamente
um dos números O, 1, 2, ... , n - 2, n - 1.
Definição 5.1. Sejam a, b e n inteiros dados, sendo n > 1. Dizemos I

que a é congruente a b, módulo n, e denotamos a _ b (mod n), (b) a b (mod n) {:::} a e b deixam um mesmo resto na divisão por
se n 1 (a - b). Se a não for congruente a b módulo n, denotamos n.
a-;/= b (mod n).
Prova.
Exemplos 5.2. De acordo com a definição acima, podemos escrever: (a) Suponha que a deixa resto r quando dividido por n. Pelo algoritmo
(a) 3 5 (mod2), pois 2 1 (3 - 5). da divisão, temos a= qn+r para algum inteiro q. Daí, a-r = qn, ou
seja, n 1 (a - r). Mas isso é o mesmo que escrevermos a - r (mod n).
(b) -1 11 (mod 12), pois 12 1 (-1 - 11).
O resto é imediato.
(c) 2 -1 (mod3), pois 3 I (2- (-1)).
(b) Se a - b (mod n), então n 1 ( a - b) e segue, do corolário 1. 9 (com
(d) x - -x (mod2), pois 21 (x - (-x)). a e b nos lugares de a 1 e a 2 e n no lugar de b), que a e b deixam
(e) 1-;/= 2 (mod3), pois 3 f (1 - 2). um mesmo resto na divisão por n. Reciprocamente, se a, b deixam
um mesmo resto r na divisão por n, podemos escrever a = nq1 + r
(f) 20-;/= 15 (mod 7), pois 7 f (20 - 15). e b = nq2 + r, com q1, q2 E Z. Logo, a - b = n(q1 - q2), ou seja,
O que estamos realmente investigando em um número quando con- n 1 (a - b). Portanto, a b (mod n). •
sideramos congruências módulo n? Para responder essa pergunta, ob-
Observação 5.4. Na definição da relação de congruência, a razão pela
servemos o que ocorre com os números inteiros módulo 4, por exemplo:
qual excluímos o módulo n = 1 é a seguinte: se usássemos congruên-
4k - O(mod4), 4k + 1 1 (mod4), cias módulo 1, obteríamos a - b (mod 1) como sinônimo de 1 1 (a-b),
o que é sempre verdade. Portanto, dois inteiros quaisquer seriam in-
4k + 2 2 (mod4) e 4k + 3 3 (mod4).
distinguíveis módulo 1.
Assim, a sequência ... , -5, -4, -3, -2, -1, O, 1, 2, 3, 4, 5, ... dos
números inteiros é igual, módulo 4, à sequência Uma vez que a notação de congruência módulo n enxerga apenas
o resto da divisão de um número por n, o leitor pode estar se per-
... , 3, o, 1, 2, 3, o, 1, 2, 3, o, 1, ...
guntando quais vantagens teremos em utilizá-la. O primeiro ganho
e vemos que todo inteiro é congruente, módulo 4, ao resto de sua ao se usar congruências é computacional: nas duas proposições a se-
divisão por 4. Esse resultado continua válido em geral, como mostrado guir, provamos algumas propriedades elementares de congruências, as
no seguinte resultado. quais vão nos permitir, por exemplo, calcular mecanica e rapidamente
120 A Relação de Congruência •· 5.1 Definições e propriedades básicas 121

o resto da divisão de 172002 por 13, tarefa que não é fácil de cumprir .prova.
com os métodos de que dispomos até o presente momento. (a) Como (a+c)-(b+d) = (a-b)+(c-d), ac-bd = a(c-d)+(a-b)d
e n 1 (a-b), n 1 (e-d), segue do item (c) da proposição 1.5 (veja tam-
Proposição 5.5. Dados inteiros a, b, e e n, sendo n > 1, temos:
bém o item i. da observação 1.6) que n 1 [(a+c)-(b+d)] e n 1 (ac-bd).
(a) a_ a (mod n). Mas isso é o mesmo que a + e - b + d (mod n) e ac - bd (mod n). Por
fim, o caso particular segue de e - e (mod n).
(b) a _ b (mod n) =} b _ a (mod n).

(c) a b(modn) e b- c(modn) =}a= c(modn). (b) Fazendo e = a e d = b na segunda parte do item (a), obtemos

Prova. Os itens (a) e (b) são imediatos. Quanto a (c), se a == ·


a2= b2 (mod n). Se já mostramos que a 1 b1 (mod n), para um certo
l EN, então, novamente da segunda parte de (a) (dessa vez com e= a1
b (mod n) e b - e (mod n), então n 1 (a - b) e n 1 (b - e), e o iterri J e d= b1), obtemos
i. da observação 1.6 garante que n divide a - e= (a - b) + (b - e). ;
Mas isso é o mesmo que a - e (mod n). • a1+1 =a· a1 = b · b1 = b1+1 (modn).

Proposição 5.6. Sejam a, b, e, d, m e n inteiros dados, com m, n > 1. O item (b) segue, por indução sobre k.

(a) Se a = b (mod n) e e - d (mod n), então a + e - b + d (mod n) e


(c) Se a - b (mod n), temos, a partir dos itens (a) e (b), que ckak -
ac = bd (mod n). Em particular, ac = bc (mod n).
ckbk (mod n), para O:::; k :::; m. Portanto, segue do problema 1 que
(b) Se a b (mod n), então ak - bk (mod n), para todo k EN. m m

(c) Seco, c1, ... , Cm E Z e J(x) = CmXm + · · · + c1x + Co, então


f(a) = L ckak = L ckbk = f(b) (mod n).
k=O k=O

a - b (mod n) =} f(a) . f(b) (mod n). (d) Como a - b (mod n), existe q E Z tal que a= b + nq. Queremos,
pois, mostrar que
(d) Se a= b(modn), então mdc(a,n) = mdc(b,n).
mdc (b + nq, n) = mdc (b, n).
(e) Se a + e - b + e (mod n), então a - b (mod n).
Mas isso é imediato a partir do item (b) da proposição 1.21.
(f) Se ac _ bc (mod n) e mdc (e, n) = d, então a_ b (mod J)· Em
particular, se mdc (e, n) = 1, então a_ b (mod n). (e) Se a+ e= b + e (mod n), então n divide (a+ e) - (b + e) = a - b,
o que é o mesmo que a - b (mod n).
(g) Se a= b (mod mn), então a_ b (mod m) e a= b (mod n).

(h) Se a - b (mod n) e a b (mod m), com mdc (m, n) = 1, então (f) Sejam n = dn' e e = de', com e' e n' inteiros primos entre si.
a - b (mod mn). De ac _ bc (mod n), segue que (dn') 1 [dc'(a - b)] ou, ainda, que
122 A Relação de Congruência 5.1 Definições e propriedades básicas 123

n' c'(a - b). Mas, como mdc (n', e') = 1, segue do item (a) da pro-
1 Prova. Se n = (akak-1 ... a1ao)10 é a representação decimal do natu-
posição 1.21 que n' 1 (a - b) ou, o que é o mesmo, a - b (mod ~'). O ral n, podemos escrever
resto é imediato.

(g) Se a= b (mod mn), então mn (a-b) e, daí, m (a-b). Mas essa


1 1

última relação equivale a a _ b (mod m); analogamente, a - b (mod n). Como 10 - 1 (mod 9), aplicando repetidas vezes as propriedades da
proposição 5.6 obtemos, módulo 9, que
(h) Como m, n 1 (a - b) e mdc (m, n) = 1, segue do item (d) da
proposição 1.21 que mn (a - b), que é o que queríamos provar.
1 •
n aklOk + ªk-110k-l + · · · + a1l0 + ao
ªk . 1k + ªk-1 . 1k-l + ... + ª1 . 1 + ªº
Conforme prometido, temos o exemplo seguinte.
ak + ªk-1 + · · · + a1 + ao.
Exemplo 5.7. Calcule o resto da divisão do número 172002 por 13.

Solução. Como 17 _ 4(modl3) e 16 =3(mod13), -segue dó item


O resto segue do item (b) da proposição 5.3.

(b) da proposição 5.6 que, módulo 13, Como exemplo adicional da simplificação computacional que a re-
lação de congruência nos oferece (e para referência futura), provaremos
172002 = 42002 = 151001 - 31001 _ novamente, a seguir, o corolário 1.8 e o exemplo 1.10. Antes, contudo,
é útil fazermos uma pequena observação.
Notando, agora, que 33 =1 (mod 13) e aplicando os itens (a) e (b) da Como já sabemos, todo inteiro é congruente a O, 1, 2, 3, 4, 5 ou 6,
proposição 5.6, obtemos
módulo 7; mas como
31001 = 32 . 3999 = 9. (33)333 - 9. 1333 = 9,
4 = -3(mod 7), 5 = -2(mod 7) e 6 _ -l(mod 7),
módulo 13. Então, segue da proposição 5.3 que 172 2 deixa resto 9 ºº
segue que todo inteiro é congruente, módulo 7, a um dos números
na divisão por 13. •
O, ±1, ±2, ±3. Por outro lado, todo inteiro é congruente a O, 1, 2, 3,
As propriedades elementares de congruências deduzidas na propo- 4, 5, 6 ou 7, módulo 8; mas, como
sição 5.6 nos permitem provar o critério de divisibilidade por 9 de
maneira mais direta que aquela sugerida no problema 1, página 10, 5 - -3(mod8), 6 -2(mod8) e 7 =-l(mod8),
conforme o próximo exemplo.
segue que todo inteiro é congruente a O, ±1, ±2, ±3 ou 4, módulo 8. A
Exemplo 5.8. Para n E N mostremos, utilizando congruências, que vantagem de trocar, módulo 7, os números de O a 6 por O, ±1, ±2, ±3
o resto da divisão de n por 9 é igual ao resto da divisão da soma dos é que, se precisarmos elevar os restos na divisão por 7 a um expo-
algarismos da representação decimal de n por 9. ente par, teremos menos trabalho usando O, ±1, ±2, ±3 em vez de
124 A Relação de Congruência 5.1 Definições e propriedades básicas 125

O, 1, 2, 3, 4, 5, 6, posto que (-x) 2 = x2 . Pela mesma razão, por ve- · algarismo de a 2 é igual a um dos números O, 1, 4, 5, 6 ou 9.
zes é vantajoso trocar, módulo 8, os números O, 1, 2, 3, 4, 5, 6, 7 por •.
o, ±1, ±2, ±3, 4. (b) Sabemos que a = O ou ±1 (mod 3), de modo que a 2 02 ou
Generalizando a discussão do parágrafo anterior, não é difícil veri- (±1) 2 (mod3). Portanto, a 2 _ O ou 1 (mod3).
ficar que:
(e) Sabemos que a O, ±1 ou 2 (mod4), de maneira que a2 _ 02 , (±1) 2
i. Se n = 2k, então todo inteiro é congruente, módulo n, a um dos
ou 22 (mod4). Como 22 - O(mod4), segue que a 2 _ O ou 1 (mod4).
números
o, ±1, ±2, ... , ±(k - 1), k. (d) Como a - O, ±1, ±2, ±3 ou 4 (mod8), segue que
ii. Se n = 2k + 1, então todo inteiro é congruente, módulo n, a um
dos números a2 = 02 , (±1)2, (±2)2, (±3) 2 ou 42 (mod8).
o, ±1, ±2, ... , ±k. Agora, 32 = 9 - 1 e 42 = 16 = O(mod8), de modo que a 2 - O, 1 ou
Formalizaremos um pouco mais a discussão acima quando éstudar- 4(mod8).
mos sistemas completos de restos, na seção 6.1. Por enquanto, vamos
ao resultado prometido. (e) Pelo item (d), temos que a 2 = 8q + r, com q EN e r = O, 1 ou 4.
Portanto,
Proposição 5.9. Para todo a E Z temos:
a4 = (8q + r) 2 = 16(4q2 + qr) + r 2 = 16q' + O ou 16q' + 1,
(a) a 2 _ O, 1, 4, 5, 6 ou 9 (mod 10).

(b) a 2 - O ou 1 (mod3). com q' EN.



(c) a 2 = O ou 1 (mod4). Os exemplos a seguir mostram como podemos usar o que apren-
demos até agora sobre congruências para resolver vários problemas
(d) a 2 = O, 1 ou 4 (mod8). interessantes. O leitor deve esforçar-se por assimilar o mais possível
as ideias desenvolvidas ao longo das soluções dos mesmos, posto que
(e) a4 = O ou 1 (mod 16).
muitas delas serão de utilidade para a resolução de vários dos proble-
Prova. mas propostos nesta seção.
(a) Módulo 10, temos a_ O, ±1, ±2, ±3, ±4 ou 5, de sorte que
Exemplo 5.10 (Itália). Ache todos os x, y EN tais que x 2 +615 = 2Y.

Solução. Analisando a equação dada módulo 3, obtemos


ou, ainda, a 2 _ O, 1, 4, 9, 6 ou 5. Mas, como o último algarismo de
um número é igual ao resto de sua divisão por 10, segue que o último
126 A Relação de Congruência . 5.1 Definições e propriedades básicas 127

Mas, pelo itern (b) da proposição 5.9, ternos x 2 O ou 1 (rnod3), de' de onde segue que n deve ser par, digamos n = 2u, corn u E N.
sorte que a congruência acima nos dá as seguintes possibilidades: Fazendo esta substituição na equação do enunciado, obtemos 2m + 1 =
o_ (-l)Y (rnod3) ou 1 (-l)Y (rnod3). 32u ou, ainda,
2m = (3u - 1)(3u + 1).
A primeira possibilidade claramente nunca ocorre. Quanto à segunda, ;
Para vermos a que fatoração de 2m corresponde (3u - 1)(3u + 1),
se y for ímpar obtemos 1 _ -1 (rnod3), o que tarnbérn nunca ocorre.'.
seja d= rndc (3u - 1, 3u + 1). Então
Portanto, y deve ser par, digamos y = 2z, corn z > O inteiro. A·•
equação do enunciado pode ser, agora, escrita corno d 1 [(3u + 1) - (3u - 1)],

615 = 22z - x 2 = (2z - x)(2z + x). ou seja, d 1 2; rnas, corno 3u - 1 e 3u + 1 são ambos pares, deve ser
d = 2. Por outro lado, urna vez que o produto de 3u - 1 e 3u + 1 é
!1 Por fim, corno 2z + x > 2z - x e 615 = 3 · 5 · 41, ternos sarnente as, uma potência de 2, cada urn dos fatores 3u - 1 e 3u + 1 deve ser urna
1,:
possibilidades potência de 2. Ocorre que o rndc de duas potências de 2 só é igual a
2z +x 205 2 quando a menor de tais potências for igual a 2, de rnodo que a única
{ 2z - X
615
1 '
{ 2z +x
2z - X
-

- 3 possibilidade é termos

2z +x 41
{ 2z - X
123
5 OU
{ 2z +x -
2z -x - 15
Sornando rnernbro a rnernbro as duas equações ern cada urna das possi-
bilidades acima, obtemos respectivamente 2z+1 = 616, 208, 128 ou 56. :'
Logo, u = 1, m = 3 e, daí, n = 2.
Exemplo 5.12. Faça os seguintes itens:

Mas, urna vez que 2z+1 urna potência de 2, a única possibilidade viável!
é que seja 2z+1 = 128, de sorte que , (a) Prove que existem x, y, z EN tais que 13x4 + 3y4 - z 4 = 2013.

{b} Prove que não existem x, y, z EN tais que 13x4 +3y 4 -z4 = 2014.
{ 2z + X. = 123
2z - X = 5 Prova ( Sketch).
Então z 6 e X 59, de rnodo que a única solução é x (a) Fazendo z = 2x, obtemos y 4 - x 4 = 671 ou, ainda, (y 2 - x 2 )(y 2 +
y = 12. x2 ) = 11 · 61. Portanto, y2 - x 2 = 11 e y2 + x 2 = 61, de forma que
X = 5, y = 6 e Z = 10.
Exemplo 5.11 (OIM). Ache todos os m, n EN tais que 2m + 1 = 3n_,

Solução. Se m = 1, então é claro que n = 1. Se m ~ 2, então (b) Suponha que haja urna solução. Pelo itern (e) da Proposição 5.9,
2m O(rnod4); portanto, analisando a equação módulo 4, obtemos temos a4 - O ou l(rnod.16), para todo a E Z. Logo, 13x4 +3y 4 -z4
O, 2, 12, 13 ou 15 (rnod. 16). Mas, corno 2014 _ 14 (rnod. 16), chegamos
1 (-ir (rnod4), a uma contradição. •
128 A Relação de Congruência 5.1 Definições e propriedades básicas 129

Os dois últimos exemplos que apresentamos resolvidos de forma • Se k = 1, então m = 2, n = 4 e a= 97, novamente um número
um pouco mais tersa que os anteriores. Como exercício para o leitor, primo.
sugerimos esforçar-se por preencher eventuais detalhes omitidos.
• Se k > 1, então
Exemplo 5.13 (Romênia). Sejam m, n,p EN, com p primo ímpar.
(49 - 1)(49k-l + · · · + 49 + 1) = 49k - 1 = 3 · 2n
Se ~=~;:~:
E N, prove que tal número é primo.

.
P rova. SeJa -
a-
7m+p·2n
7m_p,2n.
corno ou, ainda, 49k-l + · · · + 49 + 1 = 2n- 3 _ Essa igualdade nos dá,
como acima, que k é par. Por fim, sendo k = 2l, segue que
p · 2n+1 2 · 7m
a=l+ 7m - p . 2n
=-l+----
7m - p . 2n,
3 · 2n = 49 2l - 1 - ( -1) 2l - 1 _ O(mod 5),

um absurdo.
segue que (7m - p · 2n) \ mdc (p · 2n+1, 2 · 7m). Há, agora, duas possi-
bilidades:
Exemplo 5.14 (BMO). Seja (an)n~l a sequência definida para n 1 2

(i) p = 7: neste caso,
por an = 2n + 49. Ache todos os n 2 1 tais que an = pq e an+l = rs,
com p, q, r, s primos tais que p < q, r <se q - p = 8 - r.

e, daí, (7m-l - 2n) \ 2. Isso implica que 7m-l - 2n = 1; porém, anali- Solução. Seja n um natural tal que os termos an e ªn+i satisfaçam as
sando essa equação módulo 3, obtemos 1m-l - (-lt - 1 (mod3), o condições do enunciado. Se p, r > 3, então as condições do enunciado
garantem que p, q, r, s - 1 ou 5 (mod6). Mas, como 52 _ 1 (mod6),
que é um absurdo.
em qualquer caso temos pq, rs _ 1 ou 5 (mod 6). Por outro lado, se k
(ii) p -1, 7: neste caso, temos mdc (p-2n+1, 2·7m)2 e, daí, (7m-p·2n) \
=
for o número ímpar dentre n e n + 1, temos 2k _ 2 (mod 6), de sorte
2. Novamente, isso implica 7m - p · 2n = 1 e, analisando tal equação que 2k + 49 - 2 + 1 = 3 (mod 6), o que é um absurdo. Assim, devemos
também módulo 3, obtemos 1m-p·2n - 1 (mod3), de sorte que p = 3. ter P :S 3 ou r :S 3 e, daí, ao menos um dentre p e r deve ser igual a 3
(já que an e an+l são ímpares).
Segue, então, que 7m - 3 · 2n = 1 e a = 7m + 3 · 2n.
Se m = 1, então n = 1 e, daí, a = 13, um número primo. Se Se P 2 r, então q = s+p-r 2 s, de modo que an = pq 2 rs = an+l,
uma contradição. Assim, p < r, de sorte que p = 3 e q = 3 + 8 - r.
m > 1, temos n > 1 e
Logo, an = pq = 3(3 + s - r). Por outro lado, também temos
7m -1
2n-l = 6 = 7m-l + ... + 7 + 1. 2an = 2n+1 + 98 = (2n+l + 49) + 49 = an+l + 49 = rs + 49
Como a soma dos m números ímpares do segundo membro da igual- e, portanto, 6(3 + s - r) = rs + 49. Segue que
dade acima deve ser par, segue quem é par, digamos m = 2k. Daí,
67
obtemos 49k - 1 = 3 · 2n, a partir de onde consideramos dois subcasos: r=6---
s+6
130 A Relação de Congruência 5.1 Definições e propriedades básicas 131

e, daí, s + 6 = 67. Assim, s = 61, r = 5, q = 59. Então, 8. Encontre todos os inteiros positivos n tais que 7 1 (2n + 3n).

3 · 59 = an = 2n + 49 ::::} 2n = 128 ::::} n = 7. 9. Seja n = (akak-1 ... a1aoho a representação decimal de n. Prove
que o resto da divisão de n por 11 é igual ao resto da divisão por
• 11 do número

Problemas - Seção 5.1 10. A sequência de Fibonacci (Fk)k2'.l é definida por F 1 = F 2 = 1 e,


para n 2: 1, Fn+2 = Fn+l + Fn. Qual o resto da divisão de F 2002
1. * Generalize o item (a) da proposição 5.6, mostrando que se por 5?
m, n > 1 são naturais e a1, ... , am, b1, ... , bm são inteiros tais
que ak _ bk (mod n) para 1 ::::; k::::; m, então 11. Ache todos os números primos p e q tais que p 2 + 3pq + q2 seja
um quadrado perfeito.
m m m m
1··
,1,,i

12. (Estados Unidos.) Prove que a equação


k=l k=l k=l k=l
4
+ X24 + X34 + · · · + X14
4
11'1'·1

!1·1 ... X1 = 15999


2. Encontre o resto da divisão de 100055 + 55 10ºº por 7.
não possui soluções inteiras.
1!'
'11'
3. Mostre que o número 8100 + 32001 não é um quadrado perfeito.
·.lj,, 13. (Romênia.) Ache todos os naturais m, n, k tais que 2m + 3n = k2 .
,1il',,' 4. Encontre o resto da divisão do número 7310 por 5.
14. (União Soviética.) Ache todas as soluções em inteiros m, n,p >1
5. Seja n E N tal que n - -1 (mod 4). Prove que existe um primo
da equação
p tal que p I n e p - -1 (mod 4).
1! + 2! + .. · + n! = mP.
6. * Use o fato de que 24 + 54 = 641 = 27 · 5 + 1 para provar que 15. (Bulgária.) Ache todos os inteiros positivos x, y e z tais que
225 + 1 não é um número primo. 3x + 4Y = 5z,
7. (IMO.) Encontre o menor inteiro positivo n satisfazendo as duas
16. (Tchecoslováquia.) Ache todos os inteiros positivos x, y e p tais
condições a seguir:
que p é primo e px - y 3 = 1.
(a) O último algarismo da representação decimal de n é 6;
17. Ache todos os a, b, e EN tais que a e b são pares e ab + bª = 2c.
(b) Se apagarmos o algarismo 6 do final de n e o escrevermos
imediatamente à esquerda do primeiro algarismo de n, ob- 18. (Hungria.) Sejam a e b naturais dados e num inteiro não nega-
temos o número 4n. tivo tal que an I b. Prove que an+l divide (a+ ll - 1.
132 A Relação de Congruência 5.2 Os teoremas de Euler e Fermat 133

19. (OBM.) Ache todas as soluções inteiras positivas da equação (c) Mostre que mdc (3t - 2, 3t + 2) = 1; conclua, a partir daí,
x 2 + 15ª = 2b. que: (i) 3t - 2 = 1 e 3t + 2 = 5x7y, (ii) 3t - 2 = 5x e
3t + 2 = 7Y ou (iii) 3t - 2 = 7Y e 3t + 2 = 5x,
20. (França.) Qual o algarismo das unidades da parte inteira de (d) No caso (ii), use módulo 3 para concluir que não há solu-
101992 ? J 'fi
10s3+ 7 . ust1 que sua resposta. ções.
21. (China - adaptado.) O objetivo deste problema é mostrar que, (e) No caso (iii), temos 5x - 7Y = 4. A partir daí, use módulo
se x,y EN são tais que 7x - 3Y = 4, então x = y = 1. Para 4 para concluir que y é par. Por fim, se x ~ 2, use módulo
tanto, faça os seguintes itens: 25 para chegar a uma contradição.

23. (Miklós-Schweitzer.) Para resolver, em inteiros x, y, z > 1, a


(a) Use módulo 8 para concluir que não há soluções se x for
equação (x + l)Y - xz = 1, faça os seguintes itens:
par.
(b) Se x for ímpar e y > 1, analise a equação módulo 9 para (a) Analise a equação módulo x+l para concluir que zé ímpar.
concluir que x - 2 (mod3) e, portanto, que x - 5 (mod6). (b) Escreva (x + 1)Y = xz + 1 e fatore o segundo membro para
concluir que x é par; em seguida, escreva xz-l = (x+l)y_l
(c) Escrevendo x = 6q + 5, com q E N, conclua que 7x X

e fatore o segundo membro para concluir que y também é .


±2 (mod 13).
par.
(d) Mostre que toda potência de 3 é congruente a 1, 3 ou 9, (c) Se x 2s e y = 2t, com s, t EN, mostre que (x + l)t -1 e
=
módulo 13. (x + 1/ + 1 têm mdc igual a 2. A partir daí, use módulo x
(e) Supondo que y > 1, use os itens (c) e (d) para chegar a para concluir que (x + 1/ - 1 = 2sz e (x + 1/ + 1 = 2z- 1.
uma contradição. (d) Conclua, a partir de 2sz < 2z-1, que s = 1. Daí, obtenha
sucessivamente x = 2, t = 1, y = 2 e z = 3.
22. (Bulgária - adaptado.) O propósito deste problema é mostrar
que, em inteiros não negativos x, y e z, a equação 5x7y + 4 = 3z
só tem a solução x = 1, y = O e z = 2. Para tanto, faça os 5.2 Os teoremas de Euler e Fermat
seguintes itens:
O uso efetivo de congruências para calcular restos é consideravel-
(a) Se x = O, a equação se reduz a 7Y + 4 = 3z, de maneira que mente simplificado se encontrarmos expoentes que tornem uma certa
z ~2. Use módulo 9 para mostrar que, neste caso, não há potência congruente a 1. Por exemplo, sabendo que 73 - 1 (mod 9)
soluções. º
fica muito mais fácil calcular o resto da divisão de 25 10 1 por 9: como
25 7 (mod 9), temos
(b) Para este e os demais itens, suponha x ~ 1. Use modulo 5
para conclur que z = 2t, para algum t EN. 251001 - 71001 = (73)333. 72 1333. 49 4 (mod9).
134 A Relação de Congruência 5.2 Os teoremas de Euler e Fermat 135

Nessa direção, o propósito desda seção é, fixados inteiros a e n primos


entre si, com n > 1, encontrar um expoente k EN para o qual
A seguir, colecionamos algumas aplicações interessantes do pe-

ak - 1 (mod n). queno teorema de Fermat.

Para tanto, analisaremos inicialmente o caso em que n é primo, pro- Exemplo 5.16. Se p e q são primos distintos, prove que pq divide
vando um dos mais importantes resultados da teoria elementar de
+ qP-1 _ 1.
pq-1

congruências, conhecido na literatura como o pequeno teorema de Prova. Como p e q são primos distintos, temos mdc (p, q) = 1. Por-
Fermat. tanto, pelo pequeno teorema de Fermat, q divide pª- 1 -1. Mas, como
q também divide qP- 1, segue que q divide qª- 1 + (pª- 1 -1). Analoga-
Teorema 5.15 (Fermat). Para a,p E Z, com p primo, temos aP =
mente, p divide pª- 1 + (qP-l - 1). Por fim, como ambos p e q dividem
a (mod p). Em particular, se mdc (a, p) = 1, então
pq-l + qP-l -1 e mdc (p, q) = 1, o item (d) da proposição 1.21 garante
ap-l - 1 (modp). (5.1) que pq divide pª- 1 + qP-l - 1. •

Prova. Se aP - a (modp), então p divide aP - a=. a(aP-l - 1);· por- Exemplo 5.17 (Romênia). Sejam p e q números primos, com q =/:- 5.
tanto, se mdc (a,p) = 1, (5.1) segue do item (a) da proposição 1.21. Se q 1 (2P + 3P), prove que q > p.
Basta, pois, mostrarmos que aP - a (modp), para todo a E Z. Prova. Como q 1 (2P + 3P), temos claramente q =/:- 2, 3, de modo
Se p = 2 o resultado é óbvio, uma vez que a 2 - a = a( a - 1), sendo que q > 5. Portanto, podemos supor que p > 3. Se q ~ p, então
o produto de dois inteiros consecutivos, é par. Suponhamos, então, q - 1 < p, de sorte que q - 1 e p são primos entre si. Nesse caso,
que p > 2 e provemos o resultado, primeiramente para a > O, por o teorema de Bézout garante a existência de x, y E N tais que px =
indução sobre a. Para a= 1 nada há a fazer. Suponha, por hipótese (q- l)y + 1. Portanto, a partir de 2P _ -3P(mod q), obtemos (2PY -
de indução, o teorema válido para um certo valor natural de a, i.e., (-3P)x(modq); mas, como -3P = (-3)P, segue que
suponha que kP - k (modp), para algum k E N. Para a = k + 1,
2(q-l)y+l _ (-3)(q-l)y+l (mod q).
temos
(k + l)P- (k + 1) = (kP - k) + Í: (~)kp-j.
j=l J
Por fim, uma vez que q =/:- 2, 3, o pequeno teorema de Fermat nos dá
2ª- 1 , (-3)ª- 1 _ 1 (mod q); a partir daí, a congruência acima se reduz
a2 -3(modq), de maneira que q = 5. Por fim, tal conclusão é,
Mas, como p 1 (kP - k) (pela hipótese de indução) e p 1 (;) para
claramente, uma contradição. •
1 ~ j ~ p-1 (pelo exemplo 1.41), segue que p divide (k + 1)P - (k + 1),
ou seja, que (k + l)P - (k + 1) (modp). Exemplo 5.18 (BMO). Sejam p > 2 um número primo tal que 3 1

Analisemos, agora, o caso a ~ O: se a = O, nada há a fazer; se (p- 2) e


a< O, então, uma vez que pé ímpar, segue do que fizemos acima que S = {y 2 - x 3 - 1; O~ x, y < p ex, y E Z}.
aP = -(-a)P - -(-a)= a (modp). Prove que S contém não mais que p - 1 múltiplos de p.
r
. '

136 A Relação de Congruência 5.2 Os teoremas de Euler e Fermat 137

Prova. Se O:::; u, v:::; p - 1 e u 3 _ v3 (modp), afirmamos que u = v. Prova. Sejam p um fator primo de n e k um natural. Provemos
De fato, note primeiro que u 3 - v3 O(mod p) se, e só se, u = v = O; inicialmente, por indução sobre k, que
por outro lado, para 1 :::; u, v :::; p - 1, segue do pequeno teorema de
Fermat e de p - 1 = 3k + 1 que
u 3k+I _ v 3k+I (modp). O caso k = 1 se resume ao pequeno teorema de Fermat. Por hipótese
de indução, suponha que a'P(p1) 1 (modp1), para algum l natural.
Agora, Então acp(p1) = p1q + 1 para algum inteiro q e temos ·
u3 - v3 (modp) ::::} u3k - v 3k (modp),
ap·cp(p1) - 1= (a'P(P 1))P - 1
e segue de (5.2) que
(plq + l)P -1 = t (~)
j=O J
(plq)i - 1

Por fim, cancelando u3 na congruência acima, obtemos u


a condição 1 :::; u, v :::; p - 1 implica, então, u = v.
v (modp); .
(i)p'q+ t, C)iJ'q!
A discussão acima garante que

{x 3 j Ü '.S X '.S p - 1} = { Ü, 1, ... , p - 1};


pl+Iq + p2l tj=2
(~)p(j-2)lqj.
J
por outro lado, y 2 - x 3 - 1 E S é múltiplo de p se, e só se, y 2 = - O(modp1+1),
x 3 -1 (modp). Portanto, segue de (5.3) que, para cada O :S y :S p-1,
existe um único O :::; x :::; p - 1 para o qual p divide y 2 - x 3 - 1. uma vez que 2l 2::: l + 1 para todo l 2::: 1.
Isso nos daria no máximo p múltiplos de p em S. Porém, note que , Para terminar, seja n = pf 1 p~ 2 ••• p~k a decomposição canônica de
O = ·12 - 03 - 1 = 33 - 23 - 1 é representado duas vezes em S, de sorte nem fatores primos, de maneira que (cf. teorema 3.12)
que S contém não mais do que p - 1 múltiplos de p. •

Generalizamos o pequeno teorema de Fermat com o seguinte resul-


tado de L. Euler, o qual evidencia a importância da função cp. Para
-
Fazendo mi = cp(pf 1 ) ••• cp(p;i) ... cp(p~k) (onde o,,.._ sobre um fator
o enunciado do mesmo, observe que, se ak - 1 (mod n), então o item significa que o mesmo é omitido do produto correspondente), temos
(d) da proposição 5.6 garante que mdc (ak, n) = mdc (1, n) = 1; em rp(n) = cp(p;i)mj e segue de (5.5) que
particular, mdc (a, n) = 1.
acp(n) = (acp(p?))mi _ lmi l(modp?);
Teorema 5.19 (Euler). Se a e n são inteiros primos entre si,
n > 1, então mas, como os P? são dois a dois primos entre si, o item (h) da propo-
acp(n) - 1 (mod n). sição 5.6 garante que acp(n) _ 1 (mod n). •
138 A Relação de Congruência 5.2 Os teoremas de Euler e Fermat 139

Colecionamos, nos exemplos a seguir, algumas aplicações interes- Como m _ 1 (moda), temos mdc (a, m) = 1. Daí, o teorema de Euler
santes do teorema de Euler. nos dá a'P(m) - 1 (mod m).
Se y = a'P(m)+l+a'P(m)_l, então y E A; afirmamos que mdc (y, x) =
Exemplo 5.20 (OBM). Prove que existe um inteiro k > 2 tal que o
1, para todo x E Bk. Para tanto, basta mostrarmos que mdc (y, m) =
número 199 ... 91 é um múltiplo de 1991.
'-...,,----" 1. Mas, novamente pelo teorema de Euler,
k

Prova. Observe primeiramente que y = ac.p(m)+l + ac.p(m) - 1 a+ 1 - 1 = a (mod m),


1 99 ... 91 = 2 · 10k+ 1 - 9.
'-...,,----"
k
e o item (d) da proposição 5.6 garante que
Portanto, queremos achar k > 2 inteiro tal que mdc (y, m) = mdc (a, m) = 1.
2. 10k+ 1 - 9 _ O (mod 1991)
Portanto, o conjunto Bk+l = BkU{y} é formado por k+ 1 elementos
ou, ainda, 2. lQk+l - 9 (mod 1991). Mas, como 2 · 103 - 9 (mod 1991), de A, dois a dois primos entre si, o que completa o passo de indução.
temos
2. 10k+1 _ 9 (mod 1991) {:} 2 · 10k+1 _ 2 · 103 (mod 1991)

Para o próximo exemplo observe que, se a, k E N, então a repre-
{:} 10k- 2 - 1 (mod 1991).
sentação decimal de ak tem exatamente l k log 10 aJ+ 1 algarismos. De
Para o que falta, veja que 1991 = 11 · 181 e 181 é primo (pelo fato, sendo m o número de algarismos de ak, temos 10m-l ~ ak < 10m
crivo de Eratóstenes, por exemplo). Portanto, cp(1991) = cp(ll) · e, daí,
cp(181) = 10 · 180 = 1800 e segue, do teorema de Euler, que 101800 = m - 1 ~ log 10 ak < m;
1 (mod 1991). Logo, basta tomarmos k - 2 = 1800. •
portanto, m - 1 = llog 10 ak J = l k log 10 aJ, conforme desejado.
Exemplo 5.21 (Romênia). Sejam a e n naturais dados. Mostre que,
do conjunto Exemplo 5.22. Prove que existe uma potência de 2 com 1000 zeros
consecutivos em sua representação decimal.
{ª2 + a - 1, ª3 + ª2 - 1, a4 + ª3 - 1, ... } '
Prova. Queremos encontrar m E N tal que a representação decimal
podemos escolher n elementos dois a dois primos entre si.
de 2m seja da forma
Prova. Por indução, seja k E N e suponha que já provamos a exis-
tência de um conjunto Bk e A tal que IBkl = k e os elementos de Bk 2m = (* * ... * '-...,,----"
00 .. , O** . · · *)10.
1000
sejam dois a dois primos entre si. Seja

li
m= rr
xEBk
X.
Alternativamente, denotando por A e B os naturais formados pelos
algarismos de 2m situados respectivamente à direita e à esquerda da
li
li
140 A Relação de Congruência ' 5.2 Os teoremas de Euler e Fermat 141

sequência de 1000 zeros consecutivos, queremos encontrar m E N tal Problemas - Seção 5.2
que
2m = B . 1010oo+a + A, 1. (Eslovênia.) São dados vários inteiros cuja soma é igual a 1496.
É possível que a soma de suas sétimas potências seja 1999?
onde a representação decimal de A tem, no máximo, a algarismos.
Para tanto, se fizermos k = 1000 + a e A = 2k, teremos 2m 2. (Austrália.) Se pé um número primo, prove que o número
B. 10 1000+a + A se, e só se, a representação decimal de 2k tiver no
máximo k-1000 algarismos e 2m-k = B · 5k + 1; esta última condição, 11 ... 11 22 ... 22 ... 99 ... 9 -123456789
~~ '--v-'
p p p
por sua vez, se dará se, e só se, 2m-k _ 1 (mod5k).
Temos, pois, de mostrar que é possível obter k, m E N tais que: é divisível por p.
(i) m > k; (ii) a representação decimal de 2k tem no máximo k - ·
1000 algarismos; (iii) 2m-k 1 (mod5k). Comecemos analisando as 3. Dada uma sequência (x1, X2, X3, ... , X2n-2, X2n-1, X2n) de núme-
condições (i) e (iii). ros reais, uma operação permitida é trocá-la pela sequência
Pelo teorema de Euler, temos 2rp(5 k) 1 (mod5~). Mas; como
1
cp(5k) = 4 · 5k- , deve existir um inteiro positivo' q tal que 24·5k-i = .
5kq + 1. Fazendo, agora, m = k + 4 · 5k- 1, temos m > k e 2m-k = Suponha que começamos com a sequência (1, 2, 3, ... , 2n-2, 2n-
1 (mod5k). 1, 2n), onde 2n + 1 é um número primo. Mostre que, após 2n
Resta, agora, mostrarmos ser possível encontrar k E N tal que a repetições da operação acima, todos os números voltarão às suas
representação decimal de 2k tenha no máximo k - 1000 algarismos. posições originais.
Para tanto, a discussão que precede este exemplo garante que a repre-
sentação decimal de 2k tem exatamente l k log10 2J + 1, de sorte que é 4. (BMO.) Prove que a equação y 2 = x 5 - 4 não possui soluções
suficiente pedir que valha a desigualdade inteiras.

lklog10 2J +1~ k- 1000. 5. (Estados Unidos.) Dado um primo p, prove que há infinitos
naturais n tais que p divide 2n - n.
Mas, como
6. (Romênia.) Prove que não existe um inteiro n > 1 tal que n
divida 3n - 2n.

basta tomar, para começar, um natural k tal que k log 10 5 > 1001 (o 7. (Bulgária - adaptado.) Dados números primos p e q, faça os
que é certamente possível). • seguintes itens:

(a) Se p 1 (5P - 2P), então p = 3.


(b) Se p 2:: q e q 1 (5P - 2P), então q = 3.
142 A Relação de Congruência 5.2 Os teoremas de Euler e Fermat 143

(c) Ache todos os p e q tais que pq 1 (5P - 2P)(5q - 2q). 12. O propósito deste problema é provar o seguinte resultado, conhe-
cido como o teorema de Sophie Germain 1 : se p é um primo
8. (BMO.) Prove que, para todo natural n dado, existe um natural tal que 2p + 1 = q também é primo e, se x, y, z são inteiros tais
m > n tal que a representação decimal de 5m é obtida da repre- que xP + yP + zP = O, então p divide ao menos um dentre x, y, z.
sentação decimal de 5n pelo acréscimo de algarismos à esquerda Para tanto, mostre as afirmações a seguir:
de 5n,
(a) Podemos supor que x, y e z são dois a dois relativamente
9. (IMO.) Prove que, para cada inteiro n > 1, existem inteiros k1, primos e que p > 2.
k2 , .•• , kn > 1, dois a dois distintos e tais que os números 2k 1 -3, (b) Se xP + yP + zP = O, então p 1 (x + y + z).
2k2 - 3, ... , 2kn - 3 são dois a dois primos entre si.
(c) Por contradição suponha, doravante, que p f x, y, z. Se
r =J. p é um divisor primo de y + z, então
10. Se (an)n~l é a sequência de inteiros positivos definida implicita-
mente por yP-l - yP- 2z + · · · - yzP- 2 + zP-l _ pyP-l (mod r).

(d) Conclua, a partir de (c), quer f (yP- 1 -yP- 2 z+·. ·-yzP- 2 +


zP-l) e, daí, que mdc (y + z, yP-l - yP- 2 z + · .. - yzP- 2 +
prove que n I an para todo n E N.
zP-1) = 1.
11. (Irã - adaptado.) O teorema de Euler garante que, para n >2 (e) Mostre que existem a, b, e, d E Z tais que y+z = aP, z+x =
inteiro, todo fator primo de n também é fator primo de 2rp(n) -1. bP, X+ y = cP e yp-l - yP- 2 z + · · · - yzP- 2 + zP-l = dP.
A afirmação recíproca, entretanto, não é verdadeira, i.e., se n > 3 (f) Comop= !l=..!. q-1 q-1 q-1
2 ,temosx-2-+y-2-+z-2- =Ü. Useopequeno
é ímpar, então 2rp(n) - 1 sempre tem fatores primos que não são teorema de Fermat para concluir que q I xyz.
fatores primos de n. Para provar esse fato faça os seguintes itens:
(g) Se q I x,
deduza que bP + cP - aP O(modq). Substitua
(a) Sejam p 1 , ... ,Pk primos ímpares e dois a dois distintos, tais p = q; 1
e use novamente o pequeno teorema de Fermat
que k > 1 ou p 1 =J. 3. Mostre que não existem inteiros para mostrar que q I abc.
a 1 , ... , ak 2: O, não todos nulos, tais que 1. Se q I b ou q I e, conclua que q I x, y ou q I x, z,
contradizendo o item (a). Logo, q I a= (y + z).
ii. Segue de (c) que dP - pyP-l - pcP(p-l) (modq). Subs-
titua p = q;l e use o pequeno teorema de Fermat uma
(b) Conclua que, se n =J. 1, 3k é um inteiro positivo ímpar, então vez mais para concluir que ±1 - p(modq), chegando
existe um primo p tal que p f n mas p 1 (2rp(n) - 1). assim a uma nova contradição.
(c) Se n = 3k, k 2: 2, mostre que 2rp(n) - 1 _ O(mod 7). 1 Marie-Sophie Germain, matemática francesa dos séculos XVIII e XIX.
144 A Relação de Congruência 5.3 Congruências lineares e o teorema chinês dos restos 145

13. (Índia - adaptado.) Para cada x EN, seja S(x) o conjunto 5.3 Congruências lineares e o teorema chi-
S(x) = {y EN; <p(k\y) = x, para algum k EN}, nês dos restos
onde <pé a função de Euler, <p(l) = <p e <p(k) é a composta k vezes
de <p, para k > 1. Seja também Nesta seção, estudamos equações e sistemas de equações simples
envolvendo congruências.
T = {2ª · 3b; a,b E Z+,a 2:: 1}. Consideremos inicialmente congruências lineares, i.e., congruên-
Os passos abaixo mostram que x E T {:} S (x) é infinito: cias da forma
ax _ b (mod n), (5.6)
(a) Prove que x E T::::} S(x) infinito.
(b) Para cada x E N, seja u(x) o expoente da maior potência. onde a, b, n são inteiros dados, com a # O, n > 1, e procuramos as
de 2 que divide x. Prove que u(r.p(x)) 2:: u(x). soluções (ou raízes) x E Z, i.e., os inteiros x para os quais (5.6) é
verdadeira.
(c) Se x (/:. T, prove que u(r.p(x)) = u(x) se, e só se, x = 2mpª,
onde m EN e p 2:: 7 é um primo tal que p . 3 (mod4). Como caso particular da situação acima, dizemos que a é invertí-
vel, módulo n, se, para b = 1, a congruência linear (5.6) tiver solução,
(d) Se x (/:. T é tal que u(r.p( 2)(x)) = u(r.p(x)) = u(x), prove que
i.e., se existir x E Z tal que
o natural a do item (c) é igual a 1.
(e) Prove que, sendo S(x) infinito, existem a 1 , a 2 , a3 , ... ax - 1 (modn).
tais que
Um tal inteiro x é denominado um inverso de a módulo n e, a esse
x = a1, a1 = r.p(a2), a2 = r.p(a3), a3 = r.p(a4), ...
respeito, temos o seguinte resultado.
(f) Suponha ainda S(x) infinito. Utilizando o item (b) prove
que existe n E N tal que Proposição 5.23. Um inteiro a é invertível módulo n se e só se
' '
mdc (a, n) = 1. Neste caso, quaisquer dois inversos de a módulo n são
u( an) = u( an+1) = u( an+2) = · · · . congruentes, módulo n.
(g) Conclua do item (d) que, em relação ao inteiro n do item
(f), se i 2:: n + 2, então existem um primo Pi 2:: 7 tal que Prova. Se a for invertível módulo n, existe x E Z tal que ax _
Pi - 3 (mod4) e mi E N tal que ai = 2miPi· Prove, em 1 (mod n). Daí, existe y E Z para o qual ax = ny + 1 ou, ainda, xa +
seguida, que mn+2 = mn+3 = · · · e, para todo i > 1, Pi= (-y )n = 1. Sabemos pelo corolário 1.15 que, neste caso, mdc (a, n) =
1.
2Pi-1 + 1.
Reciprocamente, recorde que o teorema de Bézout garante que o
(h) Prove que não há sequência infinita q1 , q2 , q3 , ... de primos
mdc de dois inteiros sempre pode ser escrito como combinação linear
com qi = 2qi-l + 1, para todo i > 1.
dos mesmos. Portanto, se mdc (a, n) = 1, existem x, y E Z tais que
(i) Conclua que S(x) infinito ::::} x E T. ax + ny = 1 e segue, daí, que ax - 1 (mod n).
r.
.

146 A Relação de Congruência 5.3 Congruências lineares e o teorema chinês dos restos 147

Por fim, sejam x e y inversos de a, módulo n. Então Teorema 5.26 (Wilson). Um natural pé primo se e só se

ax - 1 ay (mod n), (p - 1)! -1 (modp).

de sorte que ax - ay (mod n). Mas, como mdc (a, n) = 1, segue da Prova. Se p = mn, com 1 < n < p, então (p - 1)! _ -1 (modp)
proposição 5.6 que x - y (mod n). Assim a possui, módulo n, um implica (p - 1) ! - -1 (mod n), uma vez que n I p. Por outro lado,
único inverso. • : como 1 < n 5: p - 1, temos que n 1 (p - 1)!, i.e., (p - 1)! _ O (mod n).
Corolário 5.24. Se a, p E Z, com p primo, então a é invertível módulo Portanto, O - (p - 1)! -1 (mod n), de maneira que n 1 1, uma
p se, e só se, p f a. Ademais, nesse caso o inverso de a módulo p é contradição.
único. Reciprocamente, se p é primo, considere a função

Prova. Imediata a partir do fato de que mdc (a, p) = 1 {:::> p f a. f : {1, 2, ... , p - 1} ---+ {1, 2, ... , p - 1},
Corolário 5.25. Sejam a, b, n E Z, com n > 1. Se a for invertí-
que associa a cada a E {1, 2, ... ,p-1} seu inverso a- 1 E {1, 2, ... ,p-
vel, módulo n, então a congruência ax b (mod n) possui uma única•
1}, módulo p. O corolário 5.25 garante que f é uma bijeção, com
solução, módulo n, qual seja,
f(a) = b {:::> f(b) = a. Ademais,
x - acp(n)-lb (mod n).
f(a) = a {:::> a 2 - 1 (modp)
Em particular, todo inverso de a, módulo n, é congruente a acp(n)-1, .
{:::> p 1 (a2 - 1)
módulo n.
{:::> p 1 ( a - 1) ou p 1 ( a + 1)
Prova. Se a for invertível, módulo n, então mdc (a, n) = 1. Portanto, ·
{:::> a= 1 ou p-1.
aplicando sucessivamente o item (f) da proposição 5.6 e o teorema de
Euler, obtemos Portanto, existem a 1 , ... , a E.=2. E { 1, 2, ... , p - 1} tais que
2
ax b (mod n) {:::> acp(n)x acp(n)-lb (mod n) {:::> x - acp(n)-lb (mod n).
p-3
-2-

{ 1, 2, ... , p - 1} = { 1, p - 1} LJ {ai, a; 1 }.

i=l
Uma vez que dois inversos de a módulo n são sempre congruentes, .
módulo n, diremos doravante que a possui um único inverso, módulo Por fim, segue, daí e de aia; 1 1 (modp) para 1 5: i 5: P; 3 , que
n, o qual será denotado por a- 1 quando não houver perigo de confusão .
p-3
com o inverso usual ~ de a em relação à multiplicação em (Q. -2-

Como aplicação da noção de inverso módulo n, provemos um fa- (p - 1)! = (p - 1) II (aia; =-1 (modp).
1)

i=l
moso critério de primalidade conhecido como o teorema de Wilson 2..
2 Após John Wilson, matemático inglês do século XVII. •
A Relação de Congruência 5.3 Congruências lineares e o teorema chinês dos restos 149
148

Voltando a equações envolvendo congruências, uma generalização de sorte que mdc (yj, mi) = 1. Seja bi o inverso de Yi módulo mi e
natural da congruência linear (5.6) é obtida considerando soluções x = L;=l aibiYi· Fixado 1 :S l :S k, temos que m 1 1 Yi para j =/:- l e,
de um sistema de congruências lineares. Nesse sentido, o teorema a daí, módulo m 1 temos que
seguir, conhecido como o teorema chinês dos restos, examina a
x a1b1Y1 - a1 · 1 a1 (mod m1).
existência de soluções para tais sistemas.
Teorema 5.27. Sejam m 1 , m 2 , ... , mk naturais maiores que 1 e dois a
dois primos entre si. Dados inteiros quaisquer a1, a2, ... , ak, o sistema

O problema 5 oferece uma demonstração alternativa para a exis-
de congruências lineares
tência de soluções para o sistema de congruências lineares (5.7). Para
x = a 1 (mod m1) terminar esta seção, vejamos um exemplo que mostra como é possível
x =a2 (mod m2) (5.7) aplicar o teorema chinês dos restos para conseguir resultados interes-
santes3.
'1!é]~1..,\·.
1

Exemplo 5.28. Mostre, utilizando o teorema chinês dos restos, que,


admite uma única solução, módulo m 1 m 2 ... mk· De outro modo, dado n > 1 inteiro, existem n naturais consecutivos, todos compostos.
existe um único o :s; y < m 1 m 2 .•. mk tal que x E Z satisfaz o sistema
Prova. Escolha n primos distintos p 1 , p 2 , ... , Pn e considere o sistema
acima se, e só se,
x _ y (mod m1m2 ... mk)· de congruências
x -1 (modpD
Prova. Note primeiramente que, se x 1 e x 2 forem duas soluções quais- x -2 (modp~)
quer do sistema acima, então
x _ -n (modp~)
x 1 = ai - x 2 (mod mi), V 1 :Si :S k.
Como P1,P2, ... ,Pn são dois a dois primos entre si, o teorema chinês
Mas como m 1 , ... , mk são dois a dois primos entre si, segue da pro- .. dos restos garante a existência de m E N satisfazendo o sistema acima.
' .
posição 5.6 que Portanto, PJ 1 (m + j) para 1 :S j :S n, de maneira que m + 1, m +
2, ... , m + n são naturais consecutivos e compostos. •
Portanto, se O sistema (5. 7) tiver uma solução, esta será única, módulo
m1m2.,.mk·
Para a existência de solução defina, para 1 :S j :S k,
Problemas - Seção 5.3
Yi = II mi,
l<i<k 3A esse respeito, veja também os problemas 10, página 200, e 4, página 207.
T,l3
r 150

1. Sejam a, b, n inteiros dados, com n


A Relação de Congruência

> 1. Em relação à congruên-


Congruências lineares e o teorema chinês dos restos

(c) Utilize o procedimento delineado nos itens (a) e (b) para


151

cia linear ax =b (mod n), prove que: encontrar todas as soluções do sistema de congruências li-
neares
(a) Há solução se, e só se, mdc(a,n) 1 b. x _ 2 (mod5)
(b) Se mdc (a, n) 1 b, então a congruência linear do enunci- =
{ x 4 (mod 11) .
ado possui exatamente mdc (a, n) soluções incongruentes, x-9(modl3)
módulo n.
5. Nas notações do teorema 5.27, se Yj = f}i:;i:c;k mi para 1 :s; j :s; k,
icjcj
2. Sejam a1, a2, ... , ak, b, n inteiros dados, com n > L Prove que a
congruência
prove que x = :z=t 1 ajytmJ) resolve o sistema de congruências
lineares (5. 7).

6. Seja p 1 = 2, p 2 = 3, p3 = 5, ... a sequência dos números primos.


possui solução se, e só se, mdc (a 1 , a 2 , ... , ak, n) b.
1 Mostre que existe um número natural que deixa resto 2Pk- 1 - 1
quando dividido por 2Pk - 1, para 2 :s; k :s; 100.
3. Um grupamento de soldados foi disposto em .um bloco retangu-
lar, com várias fileiras. O comandante observou que, ao colocar 7. Prove que há sequências arbitrariamente longas de inteiros po-
12 soldados por fileira, sobraram 7 soldados e, ao colocar 13 sol- sitivos e cqnsecutivos, nenhum dos quais é uma potência de ex-
dados por fileira, sobraram 5 soldados. Sabendo que o total de poente maior que 1.
soldados estava situado entre 600 e 700, pergunta-se: quantos
soldados havia? 8. Sejam m = pf 1 . . . p~k a decomposição canônica do inteiro m > 1
em fatores primos e f um polinômio de coeficientes inteiros.
4. Nas hipóteses do teorema 5.27, faça os seguintes itens:
(a) Prove que a congruência f (x) - O (mod m) tem uma solu-
(a) Mostre que resolver (5.7) para k = 2 equivale a resolver, ção inteira se, e só se, cada uma das congruências f (x) _
em u, v E Z, a equação Diofantina linear O( mod pfi), 1 :s; i :s; k, tiver solução 4 .
(b) Para t EN, denote por N(t) o número de soluções inteiras,
duas a duas incongruentes, módulo t, para a congruência
(b) Mostre que resolver (5. 7) equivale a encontrar tods os y E Z f(x) - O(mod t). Mostre que N(m) = N(pf 1 ) . . . N(p~k).
que resolvem o sistema de congruências lineares
9. Se, em uma progressão aritmética não constante de números na-
m1y - a2 - a1 (mod m2)
turais, com termo inicial e razão primos entre si, um dos termos
m1y - a3 - a1 (mod m 3 )
4 Se p é primo e a é natural, uma condição suficiente para a existência de
soluções para uma congruência da forma f(x) =
O(modp°') será apresentada no
problema 3.4.8 do volume 6.
152 A Relação de Congruência

for um quadrado perfeito e outro for um cubo perfeito,


que há um termo que é uma sexta potência5 .

CAPÍTULO 6

Classes de Congruência

Fixado um inteiro n > 1, a proposição 5.5 garante que a relação


"", definida em Z por

a"" b {::} a - b (mod n),

é uma relação de equivalência em Z, denominada a relação de con-


gruência módulo n. Nesta seção, estudamos tal relação sob o ponto
de vista mais geral das relações de equivalência, para o quê o leitor
pode acha conveniente reler as partes pertinentes da seção 2.3 do vo-
lume 4. Alternativamente, sugerimos a referência [8].

6.1 Sistemas de restos


5 As exigências de que a PA seja não constante e tenha primeiro termo e razão

primos entre si são desnecessárias, só tendo sido colocadas a fim de simplificar a Para a E Z, definimos a classe de congruência de a, módulo
solução do problema. n, como a classe de equivalência a de a com respeito à relação de

153
154 Classes de Congruência 6.1 Sistemas de restos 155

congruência módulo n: Proposição 6.3.

{x E Z; x _ a (modn)} (a) Sejam a, b e n inteiros dados, com n > 1 e a e n primos entre


{x E Z; x =a+ nq, :3 q E Z} si. Se o conjunto { xo, x 1 , ... , Xn-d é um SCR, módulo n, então
{ ... , -2n + a, -n + a, a, n + a, 2n + a, ... } . o conjunto {axo + b, ax 1 + b, ... , axn-l + b} também o é.

(b) Sejam m e n inteiros maiores do que 1 e primos entre si. Se


Fixado a E Z, sabemos que existe um único inteiro O :S r <
n (o resto da divisão de a por n) tal que a r (mod n). Assim, os conjuntos {xo,x1, ... ,Xm-1} e {Yo,Yl,···,Yn-1} são SCR's
módulos m e n, respectivamente, então o conjunto
{O, 1, ... , n - 1} é um SRD para a relação de congruência módulo n e
a proposição 2.15 e a relação (2.10) do volume 4 fornecem a partição

Z=OUlU···Un-1. (6.1)
é um SCR módulo mn.
Mais geralmente, temos a definição a seguir.
Prova.
Definição 6.1. Um sistema completo de restos módulo n, (abre- (a) Como todo SCR módulo n tem n elementos, basta mostrarmos
viamos SCR) é um SRD para a relação de congruência módulo n. que, se O :S i, j < n e

Dado n > 1 inteiro, segue de (6.1) que todo SCR, módulo n, é um axi +b axj + b (mod n),
conjunto {a0 , a 1, ... , ªn-i} de inteiros tais que ar - r (mod n) para
O :S r < n. Equivalentemente, um conjunto de n inteiros é um SCR, então i = j. Para tanto, segue da congruência acima que axi -
módulo n, se, e só se, seus elementos forem dois a dois incongruentes, axj (mod n). Agora, como mdc (a, n) = 1, o item (f) da proposição
módulo n. 5.6 garante que Xi Xj (modn). Por fim, como {x 0 ,x 1 , ... ,Xn-i} é
um SCR, módulo n, esta última congruência nos dá i = j.
Exemplo 6.2. Para n > 1 inteiro, o conjunto { x E Z; -% :S x < %}
é um SCR, módulo n. De fato, observe inicialmente que, se-% :S x < (b) Analogamente ao item (a), basta mostrarmos que, se O :S i,j < m,
y <%,então x "'I=- y (mod n), uma vez que O< y-x < n; basta, agora, O :S k, l < n e
considerar separadamente os casos n par e n ímpar a fim de mostrar
que o conjunto em questão possui exatamente n elementos.

A proposição a seguir ensina como construir novos SCR's a partir então i = j e k = l. Supondo a validade da congruência acima, temos,
de outros já conhecidos. pelo item (g) da proposição 5.6,
156 Classes de Congruência 6.1 Sistemas de restos 157

e, daí, portanto, a classe A terá tantos conjuntos quantos forem os conjuntos


nxi - nxj (mod m) e myk - my1 (mod n). distintos da forma (A n X)+ r, quando r varia de O a p - 1.
Denote A' = A n X e S' = I:xEA' x. Como A -=f. X, Y, temos
Mas, como me n são primos entre si, aplicando novamente o item (f)
da proposição 5.6, obtemos
0 -=f. A' -=f. X, de maneira que 1 :::; IA'I :::; p - 1. Portanto,

xi= Xj (mod m) e Yk - Yz (mod n). L x= L x + ri A' 1 = S' + ri A' 1-


xEA'+r xEA'

Por fim, uma vez que {xo, Xi, ... , Xm-1} e {Yo, y 1 , ... , Yn-i} são SCR's Por outro lado, uma vez que mdc (IA'l,P) = 1, a proposição 6.3 ga-
módulos me n, respectivamente, temos i = j e k = l. • rante que, à medida que r varia de O a p - 1, os números S' + r IA' 1 for-
O exemplo a seguir traz uma bela aplicação combinatória do con- mam um SCR módulo p; em particular, as somas I:xEA'+r x são duas
ceito de sistema completo de restos. a duas distintas, de maneira que os conjuntos A', A'+ 1, ... , A'+ (p- 1)
também são dois a dois distintos. Logo, a classe A tem exatamente p
Exemplo 6.4 (IMO). Seja pum primo ímpar. Calcule quantos sub- conjuntos, e segue da proposição 2.17 do volume 4 que há exatamente
conjuntos de p elementos do conjunto {1, 2, ... , 2p} são tais que a soma
de seus elementos é divisível por p.

Solução. Seja F a família dos (2:) - 2 subconjuntos de p elementos


do conjunto {1, 2, ... , 2p}, diferentes dos conjuntos X= {1, 2, ... ,p} classes de equivalência distintas.
e Y = {p + 1, p + 2, ... , 2p} (observe que esses dois conjuntos têm Se mostrarmos que em cada classe há exatamente um conjunto
soma de elementos iguais a múltiplos de p). cuja soma de elementos é um múltiplo de p, seguirá que o número
Definamos em F uma relação ,. . . , tal que A ,. . . , B se, e somente se, pedido de conjuntos é
as seguintes condições forem satisfeitas:

(a) Existe O:::; r:::; p - 1 tal que (A n X)+ r = B n X (modp).

(b) A n Y = B n Y. (não podemos esquecer dos conjuntos X e Y!).


Para o que falta, fixado A E F (e, portanto, a classe A), seJa
(Aqui, consoante a discussão que precede o exemplo 2.21 do volume A" = A n Y. Queremos contar quantos são os inteiros O :::; r :::; p - 1
4, parar E ffi. e Z e ffi., definimos Z + r = {z + r; z E Z}.) tais que a soma dos p elementos do conjunto
É imediato verificar que ,. . . , é uma relação de equivalência em F;
por outro lado, fixado A E F e B E A (a classe de equivalência de A B = [(AnX) +r] u (AnY) = (A' +r) uA"
em F), temos
seja um múltiplo de p (lembre-se de que o conjunto B acima é um
B = (B n X) u (B n Y) = [(A n X)+ r] u (A n Y); elemento genérico da classe A). Mas, para um tal B, segue do que
Classes de Congruência 6.1 Sistemas de restos 159
158
Fixado n > 1 inteiro, como caso particular mais importante da
fizemos acima que
definição acima temos que o conjunto
L x+ Lx {xEZ; mdc(x,n)=lel:Sx:Sn}
xEB xEA'+r xEA"

L x + r\A'\ + L x é um ser módulo n.


xEA' xEA"
Mais geralmente, se I for um ser módulo n, então (6.2) garante
Lx + r\A'\ = S + r\A'\, facilmente que III = cp(n), onde cp é a função de Euler. Por outro lado,
xEA
é claro que l pode ser aumentado a um SCR módulo n e que todo
~ Portanto, tal soma será congruente a O módulo p SCR módulo n contém um SCr módulo n.
ond e S = L..,xEA x.
A proposição a seguir ensina como construir novos· SCr's a partir
se, e só se,
\A'\r -S (modp). de outros já conhecidos.

Mas, como mdc (\A'\,p) = 1, sabemos do corolário 5.24 que existe um Proposição 6.6. Sejam m, n > 1 inteiros dados.
único O :::::; r :::::; p - 1 tal que a congruência acima é satisfeita. · •
1' (a) Se a é um inteiro primo com n e { X1, X2, ... , Xcp(n)} é um ser mó-
1.
Voltando ao desenvolvimento da teoria, dados q, r E Z, com O :S dulo n, então {ax1, ax2, ... , aXcp(n)} também é um SCr módulo
r < n, lembre que mdc (nq + r, n) = mdc (r, n). Assim, podemos n.
definir o mdc entre uma classe de congruência módulo n e n pondo
(b) Se {x1,X2, ... ,Xcp(m)} e {Y1,Y2,···,Ycp(n)} são ser módulos me
mdc (r, n) = mdc (x, n), para qualquer x E r; (6.2) n, respectivamente, então

(6.3)
em particular,
mdc (r, n) = mdc (r, n). é um ser módulo mn.
Note que, pela proposição 5.23, as classes de congruência r tais que
Prova.
mdc (r, n) = 1 são exatamente as formadas pelos inteiros invertíveis
(a) Como todo SCr é parte de um SCR, segue do item (a) da propo-
módulo n. Temos, então, a seguinte definição. sição 6.3 que
Definição 6.5. Um sistema completo de invertíveis módulo n
axi i:. axj (mod n), V 1 :::::; i < j :::::; cp(n).
(abreviamos SCI) é um conjunto I de inteiros tal que
Basta, pois, mostrarmos que mdc (axi, n) = 1, o que é imediato.
_ { 1, se mdc(r,n) = 1
\I n r\ = O, se mdc (r, n) # 1 '
(b) Utilizando novamente o fato de que todo ser é parte de um SCR,
para toda classe de congruência r, módulo n. o item (b) da proposição 6.3 garante que o conjunto em (6.3) tem
160 Classes de Congruência 6.2 O conjunto quociente Zn 161

exatamente cp(m)cp(n) elementos. Mas, como cp(m)cp(n) = cp(mn) 6.2 O conjunto quociente Zn
(recorde-se de que mdc (m, n) = 1), concluímos que tal conjunto tem
cp( mn) elementos. Portanto, para mostrarmos que se trata de um SCI Examinemos, agora, o conjunto quociente
módulo mn, basta provarmos que
Zn = {Õ, I, ... , n - 1} (6.5)

mdc (nxi + myj, mn) = 1 de Z pela relação de congruência módulo n. A proposição 5.6 nos
permite introduzir em Zn duas operações, às quais também nos re-
para todos 1 ::; i ::; c.p( m), 1 ::; j ::; c.p( n). Para tanto, segue dos itens feriremos como adição e multiplicação e que gozam de propriedades
(b) e (c) da proposição 1.21 que análogas às das operações usuais de adição e multiplicação de inteiros.
De fato, temos o seguinte resultado fundamental.
mdc (nxi + myj, m) = mdc (nxi, m) = mdc (xi, m) = 1
Proposição 6.8. Em Zn, as operações
e, analogamente, mdc (nxi + myj, n) = 1; portanto, aplicando mais
uma vez o item (c) daquele resultado, obtemos mdc (nxi 4-myj, rrin) = (6.6)
1. •
estão bem definidas, são comutativas, associativas e têm elementos
neutros respectivamente iguais a Oe I. Ademais, 8 é distributiva em
De posse do conceito de SCI e do item (a) da proposição anterior
relação a EB.
podemos dar uma outra prova do teorema de Euler.
Prova. Inicialmente, temos de mostrar que as operações definidas por
Teorema 6.7 (Euler). Se a e n são inteiros primos entre si, com n > 1, (6.6) estão bem definidas, no sentido de que os resultados de a EB b e
então a 8 b independem dos representantes escolhidos para as classes de
a'P(n) - 1 (mod n). (6.4) congruência a e b.
Para tanto, se a= e e b = d, então a_ e (mod n) e b d (mod n)
Prova. Sejam k = cp(n) e {x 1 ,x 2 , ... ,xk} um SCI módulo n. Pela e segue, da proposição 5.6, que
proposição 6.6, o conjunto { ax 1, ax 2, ... , axk} também é um SCI mó-
dulo n; portanto, módulo n, os inteiros ax 1 , ax 2 , ... , axk são, em al- a + b _ e + d (mod n) e a · b _ e · d (mod n).
guma ordem, congruentes aos inteiros x 1 , x 2 , ... , Xk e segue, daí, que
Mas isso é o mesmo que

a +b= e+d e a · b = e · d,

Por fim, como mdc (x 1 x 2 ... xk, n) = 1, segue da congruência acima e de sorte que
do item (f) da proposição 5.6 que ak _ 1 (mod n). • a EB b = e EB d e a0 b = e0 d.
Classes de Congruência 6.2 O conjunto quociente Zn 163
162

O que falta é mais simples. Por exemplo, para a comutatividade Tábua de Multiplicação em Z 6
de EB, temos, a partir da comutatividade da adição de inteiros, que
8 o 1 2 3 4 5
a EB b = a + b = b + a = b EB a; o o o o o o o
1 o 1 2 3 4 5
analogamente, provamos que 8 é comutativa e que EB e 8 são associ- 2 o 2 4 o 2 4
ativas, i.e., tais que 3 o 3 o 3 o 3
4 o 4 2 o 4 2
a EB (b EB e) = (a EB b) EB e 5 o 5 4 3 2 1

e Sempre que não houver perigo de confusão, escreveremos simples-


a 8 (b 8 e) = (a 8 b) 8 e. mente + e ·, em vez de EB e 8, para denotar as operações de adição e
multiplicação em Zn. Assim, o leitor deve permanecer atento para o
A verificação dos elementos neutros também é imediata:
fato de que, nas igualdades
a EB õ = a + O = a e a 8 I = a · 1 _: a. a+ b = a + b e a · b = a · b,
Por fim, a verificação da distributividade de 8 em relação a EB será os dois sinais + e · denotam operações distintas: o primeiro sinal +
deixada como exercício para o leitor (cf. problema 2). • denota a operação de adição em Zn, ao passo que o segundo sinal +
denota a adição usual de inteiros (valendo uma observação análoga
À guisa de exemplo, mostramos, a seguir, as tábuas de adição e
para os sinais ·).
multiplicação em Z 6 :
A associatividade da adição de Zn fornece imediatamente a lei de
Tábua de Adição em Z 6 cancelamento
- -
a + b = a + e =} b = e.
EB o 1 2 3 4 5
De fato, se a+ b = a+ e, então
o o 1 2 3 4 5
1 1 2 3 4 5 o -a + (a+ b) = -a + (a+ e)
2 2 3 4 5 o 1 e, daí,
3 3 4 5 o 1 2 (=ci + a) +b= ( -a + a) + e.
4 4 5 o 1 2 3
Mas, como -a + a = -a + a = O e O é o elemento neutro da adição
5 5 o 1 2 3 4
em Zn, temos, a partir da igualdade acima, que b = e.
Para a multiplicação de Zn, a lei de cancelamento é um pouco mais
complicada. Temos, inicialmente, a definição a seguir.
164 Classes de Congruência 6.2 O conjunto quociente Zn 165

Definição 6.9. Um elemento a E Zn é uma unidade se existir b E Zn Proposição 6.10. Uma classe de congruência a E Zn é uma unidade
tal que a · b = I. em Zn se e só se mdc (a, n) = 1. Em particular, Zn possui exatamente
cp(n) unidades distintas.
Assim como com conjuntos numéricos ordinários, as classes ±1 são
unidades em Zn, pois ±1 · ±1 = I. Por outro lado, a novidade em Prova. Por definição, a E Zn é uma unidade se, e só se, existir b E Zn
relação a Z é que pode haver outras unidades; por exemplo, em Z 9 as tal que a · b = I, i.e., tal que ab = I ou, ainda,
classes 4 e 7 são unidades, uma vez que
ab = 1 (mod n).
4 · 7 = 4 · 7 = I = 7 . 4. Portanto, a é uma unidade em Zn se, e só se, a E Z for um inver-
tível módulo n. Mas, pela proposição 5.23, tal ocorre se, e só se,
Se a E Zn é uma unidade, então temos a seguinte lei de cancela- mdc (a, n) = 1.
mento em relação à multiplicação: Para o que falta, basta observar que, como Zn = {I, 2, ... , n}, o
conjunto das unidades de Zn coincide com o conjunto das classes a
a . e = a . d ::::} e = d. tais que 1 ~ a ~ n e mdc (a, n) = 1. Mas, uma vez que tal conjunto
tem cp(n) elementos, nada mais há a fazer. •
De fato, tomando b E Zn tal que a · b = I (e, portanto, b · a = I
também), segue de a· e= a· d que Chegamos ao caso de maior interesse.

b· (a·c) = b· (a· d). Corolário 6.11. Se p é primo, então todo elemento em Zp \ {O} é
uma unidade.
Então, a associatividade da multiplicação fornece
Prova. Pela proposição anterior, basta mostrar que, se a =J. Oem Zp,
então mdc (a,p) = 1. Mas, a =J. O é o mesmo que a f=. O(modp) ou,
(b · a) · e = (b . a) · d
ainda, p f a, e a prova do lema 1.40 garante que p f a se, e só se,
ou, ainda, I · e = I · d. Mas isso é o mesmo que e = d, conforme mdc (a,p) = 1. •
desejado. Em termos das operações aritméticas de adição, subtração, multi-
Em particular, se a E Zn é uma unidade, então o elemento b E Zn plicação e divisão, o corolário acima nos permite colocar Zp, p primo
cuja existência é garantida pela definição 6.9 é único, pois se em pé de igualdade com os conjuntos numéricos Q e IR (e, após o
capítulo 1 do volume 6, também com o conjunto CC dos números com-
a·b=I=a·c ' plexos). Mais precisamente, em Zn (para todo n > 1, não somente n
primo) podemos definir uma operação - , a qual chamaremos subtra-
então a lei do cancelamento para a multiplicação garante que b = e.
ção, análoga à subtração ordinária de números reais, pondo
Dizemos, portanto, que tal b E Zn é o inverso multiplicativo de a.
A proposição a seguir caracteriza todas as unidades de Zn. a-b=a-b
166 Classes de Congruência 6.2 O conjunto quociente Zn 167

(cf. problema 3). Por outro lado, restrinjamo-nos agora ao caso em 5. Fixados m, n inteiros, sendo n > 1, mostre que a operação de
que n = p, um número primo; se, para a E Zp \ {O}, convencionar- multiplicação por m em Zn, dada por m · a = m · a está bem
mos escrever a;- 1 para denotar o inverso multiplicativo de a (i.e., se definida e é tal que
denotarmos por a;- 1 a classe b E ZP cuja existência é garantida pela
definição 6.9), então m · (a+ b) = m · a+ m · b e ( m1 + m2) · a = m1 · a+ m2 · a,
para todos m, m 1, m 2 E '1l, e a, b E Zn.
ademais, se a· b = e, com a, b, e E Zp e b #- O, então é imediato verificar
6. Se n > 1 não é primo, mostre que Zn possui divisores de zero,
que
--1 i.e., que existem a, b E Zn, tais que a, b #- O, mas a· b = O.
a=c· b .
Mostre ainda que, se p é um número primo, então 'll,P não possui
Portanto, podemos definir em 'll,P uma operação de divisão de forma divisores de zero.
análoga à divisão usual entre números reais (resp. complexos, cf. ca-
pítulo 1 do volume 6), i.e., pondo, para b E Zp \ {O} e a E Zp, 7. Sejam dados a e n inteiros, sendo n > 1. Mostre que, em Zn, a
equação a· x = b tem solução para todo b E Zn se, e só se, a for
- --1
a7b=a·b . uma unidade em Zn. Nesse caso, mostre que a solução x E Zn é
única, sendo dada por x = a;- 1 . b.

1,
Problemas - Seção 6.2

1. Escreva as tábuas de adição e multiplicação de Z 5 e Z 7 .

2. * Prove que, em Zn, a multiplicação é distributiva em relação à


adição. Mais precisamente, mostre que

a . (b + e) = (a . b) + (a . e),
para todos a, b, e E Zn·
3. * Prove que a operação de subtração em Zn está bem definida.

4. Em 'll, 12 , obtenha todas as unidades que são iguais a seus inversos


multiplicativos.
168 Classes de Congruência

CAPÍTULO 7

Raízes Primitivas e Resíduos Quadráticos

Neste último capítulo, retomamos a análise da congruência

ak - 1 (mod n),

concentrando-nos em dois problemas distintos, descritos brevemente


a seguir. Por um lado, já sabemos pelo teorema de Euler que, se
mdc (a, n) = 1, então tal congruência sempre é satisfeita para k =
rp(n), onde rp denota a função de Euler; por outro lado, fixado a E Z
primo com n, nada sabemos ainda sobre se tal valor de k é o menor
possível, e um dos resultados centrais deste capítulo é a caracterização
dos números inteiros a relativamente primos com n, ditos as raízes
primitivas módulo n, tais que o valor mínimo de k é k = rp( n).
Um segundo problema que queremos considerar aqui impõe uma
importante mudança de ponto de vista em relação à congruência acima.
Em vez de fixarmos a base a e procurarmos os valores k E N para os

169
170 Raízes Primitivas e Resíduos Quadráticos 7.1 Ordem módulo n 171

quais a congruência tenha solução, fixamos k E N e procuramos os va- (b) ak _ 1 (modn) {:} ordn(a) 1 k. Em particular, ordn(a) 1 cp(n).
lores a E Z que resolvem a congruência em questão. De fato, dado o
Prova.
caráter elementar destas notas, restringimo-nos ao caso k = 2, quando
(a) Se existissem O :::; k < l < h tais que a 1 = ak (mod n), o item (f)
os a E Z que resolvem a congruência são os resíduos quadráticos mó-
da proposição 5.6 daria al-k _ 1 (mod n), com O < l - k < h. Mas
dulo n.
isso seria uma contradição à minimalidade de h.
Como subproduto da teoria desenvolvida, provamos mais um famo-
Para o que falta, basta ver que, se ordn (a) = cp( n), então o con-
so teorema de Fermat, que desta feita caracteriza os números naturais
junto {1, a, a 2 , •.. , a'P(n)-I} tem cp(n) inteiros primos com n e dois a
que podem ser escritos como a soma dos quadrados de dois inteiros.
dois incongruentes módulo n, logo é um SCI módulo n.

7.1 Ordem módulo n (b) Seja ordn(a) = h, de sorte que, em particular, ah - 1 (modn). Se
k = hl, então
Sejam a, n inteiros tais que n > 1 e mdc (a, n) = 1. Sabemos, pelo
teorema de Euler 5.19, que sempre existe um natural k tal que ak = ak = ahl = (ah)Z _ 11 = 1 (mod n).
1 (mod n), qual seja, k = cp(n). Contudo, não há nada que garanta ser Reciprocamente, seja k um natural tal que ak =
1 (mod n). Pelo
cp(n) o menor dentre tais naturais k; por exemplo, 23 = 1 (mod 7) mas algoritmo da divisão, existem inteiros q e r, com O :::; r < h, tais que
cp(7) = 6. Essas considerações motivam a definição a seguir. k = qh + r. Assim, temos

Definição 7 .1. Dados a e n inteiros primos entre si, com n > 1, a 1 _ ak = aqh+r = (ah)q · ar - lq ·ar_ ar (modn).
ordem de a, módulo n, denotada ordn (a), é o menor h E N para o
Ser > O, temos quer é um expoente positivo, menor que h e tal que
qual
ar _ 1 (mod n), contradizendo a minimalidade de h. Logo, r = O e
ah - 1 (mod n).
então h I k, i.e., ordn(a) 1 k.
Das considerações acima, é imediato que Por fim, a relação ordn(a) 1 cp(n) segue da primeira parte do item
(b), juntamente com o teorema de Euler 5.19. •
ordn(a) :::; cp(n),
Exemplo 7.3. Calcule as ordens de 2 módulo 17 e de 7 módulo 10.
nem sempre ocorrendo a igualdade. A proposição a seguir estabelece
Solução. Se h = ord 17 (2), então h I cp(l 7) = 16, de sorte que h = 1,
algumas propriedades elementares do número ordn (a).
2, 4, 8 ou 16. Evidentemente h =/=- 1, 2; também, 24 - -1 (mod 17), de
Proposição 7.2. Sejam a, n E Z, com n > 1 e mdc (a, n) = 1. modo que h =/=- 4. Por outro lado, 28 = (2 4 ) 2 =
(-1) 2 - 1 (mod 17) e,
daí, h = 8.
(a) Se ordn(a) = h, então os inteiros 1, a, a 2 , ••. , ah-I são dois a dois Se h = ord 10 (7), então h I cp(lü) = 4, de maneira que h = 1, 2
incongruentes, módulo n. Em particular, se ordn(a) = cp(n),
então o conjunto {1, a, a 2 , ... , a'P(n)-I} é um SCI módulo n.
ou 4. Evidentemente, h =/=- 1; mas, como 72 =
-1 (mod 10), também
temos h =/=- 2. Logo, h = 4. •
~
1

172 Raízes Primitivas e Resíduos Quadráticos 7.1 Ordem módulo n 173

Vejamos, agora, como a proposição 7.2 pode ser aplicada à resolu- Prova.
ção de problemas interessantes. (a) Segue de a - a+ n (mod n) que ak - (a+ n)k (mod n), para todo
k EN. Em particular, ak _ 1 (modn) se, e só se, (a+n)k =
1 (modn)
Exemplo 7.4. Se p é um primo ímpar, prove que todos os fatores e, daí, a e a+ n têm ordens iguais, módulo n.
primos de 2P - 1 são da forma 2kp + 1, para algum k EN.
(b) Como m I n, o item (g) da proposição 5.6 garante que, se ak _
Prova. Se q é um primo que divide 2P - 1, então q é ímpar e 2P == 1 (mod n), então ak · 1 (mod m). Em particular, como ak _ 1 (mod n)
1 (mod q). Segue do item (b) da proposição anterior que ordq (2) 1 p e, quando k = ordn (a), temos que
daí, ordq(2) = 1 ou p. Mas, se ordq(2) = 1, então q = 1, o que é um
absurdo; logo, ordq(2) = p. aordn(a) _ 1 (mod m).
Por outro lado, segue do pequeno teorema de Fermat que 2q-l ==
1 (mod q), de sorte que, novamente pelo item (b) da proposição ante- O item (b) da proposição 7.2 garante, agora, que ordm(a) 1 ordn(a).
rior, temos
(c) Seja d= mdc (h, k). Pelo item (b) da proposição 7.2, temos
p = ordq(2) 1 (q - 1).
(ak)j _ 1 (mod n) {:} akj - 1 (mod n) {:} h I kj
Mas como q é ímpar, deve existir k EN tal que q - 1 = 2kp. •
h k h
{:} d I d . j {:} d I j,
A proposição a seguir estabelece mais algumas propriedades úteis
da ordem módulo n de inteiros, as quais serão de grande utilidade no onde, na última equivalência, utilizamos o item (a) da proposição 1.21,
que segue. juntamente com o fato de mdc (~, ~) = 1. A partir daí, é imediato
que
Proposição 7.5. Sejam a e n inteiros primos entre si, com n > 1. k h h
ordn(a) = - = - - - -
d mdc (h, k)'
(a) ordn(a) = ordn(a + n). (d) Segue claramente de (c).

(b) Sem> 1 é um natural tal quem I n, então ordm(a) 1 ordn(a). (e) Pelo item (d), o número de expoentes 1 ::::; k ::::; h tais que ak tem
ordem h módulo n é igual ao número de tais expoentes relativamente
(c) Se ordn(a) = h e k EN, então ordn(ak) = mdc(h,k)'
primos com h, i.e., é igual a cp(h). •
(d) Se k EN, então ordn(ak) = ordn(a) {:} mdc (ordn(a), k) = 1.

(e) Se ordn (a) = h, então o conjunto {a, a 2 , ... , ah} tem exatamente
cp(h) elementos com ordem h módulo n.
Raízes Primitivas e Resíduos Quadráticos 7.2 Raízes primitivas 175
174

Problemas - Seção 7.1 7.2 Raízes primitivas


Dados a, n inteiros tais que n > 1 e mdc (a, n) = 1, nesta seção es-
1. Calcule ord 7 (2), ord 11 (2) e ord 15 (7). taremos particularmente interessados no caso em que ordn(a) = <p(n).
Tal caso é tão importante que o isolamos na definição a seguir.
2. Prove que, para todo inteiro positivo n, o número 23 n + 1 não é
múltiplo de 17. Definição 7.6. Sejam a, n inteiros tais que n > 1 e mdc (a, n) = 1.
Dizemos que a é uma raiz primitiva, módulo n, se ordn(a) = <p(n).
3. Sejam a e n inteiros primos entre si, com n > 2. Se existe um
natural k tal que ak - -l(mod n), prove que ordn(a) é par. Vejamos alguns exemplos.

4. (Putnam.) Ache todos os n EN tais que n 1 (2n - 1). Exemplos 7.7.

(a) Como 21 _ 2 (mod 3) e <p(3) = 2, segue que ord3 (2) = 2 = <p(3),


5. (Turquia.) Para cada n EN, prove que n! divide o número
i.e., 2 é raiz primitiva módulo 3. Cálculos análogos garantem
n-1 que 2 também é raiz primitiva módulo 5.
II (2n - 2J).
j=O (b) Módulo 7 temos 21 - 2, 22 =4 e 23 _ 1. Portanto, ord 7 (2) =
3 < 6 = <p(7), e segue que 2 não é raiz primitiva módulo 7.
6. Dado um natural n > 2, rotulamos os vértices de um 2n-ágono
O principal resultado desta seção é a caracterização dos módulos n
regular P como 1, 2, 3, ... , n, -n, -(n - 1), ... , -3, -2,
que possuem raízes primitivas. Mais precisamente, mostraremos que
-1, sucessivamente e no sentido horário. A seguir, marcamos os
um inteiro n > 1 possui uma raiz primitiva se, e só se, n = 2, 4, pk
vértices de P da seguinte maneira: no primeiro passo, marcamos
ou 2pk, onde p é um primo ímpar. Calculemos, inicialmente, quantas
o vértice 1; por outro lado, se ki foi o vértice marcado no i-ésimo
podem ser as raízes primitivas duas a duas incongruentes módulo n.
passo, então, no (i + 1)-ésimo passo, marcamos o vértice que
se encontra a Iki I vértices do vértice ki, no sentido horário se
Proposição 7.8. Se um inteiro n > 1 tem uma raiz primitiva, a
ki > O e no sentido anti-horário se ki < O. Este procedimento
digamos, então toda raiz primitiva módulo n é congruente a um dos
continua até marcarmos um vértice já marcado em um passo
elementos do conjunto
anterior. Seja f(n) o número de vértices não marcados ao final
deste processo. {a\ 1 :S k :S <p(n) e mdc (<p(n), k) = 1}.

(a) Se f(n) = O, prove que 2n + 1 é um primo ímpar. Em particular, n tem exatamente <p( <p( n)) raízes primitivas, duas a
(b) Calcule f (1997). duas incongruentes módulo n.
Raízes Primitivas e Resíduos Quadráticos 7.2 Raízes primitivas 177
176

Prova. A segunda parte segue da primeira, pela definição da função ordn(a) < (()(n).
(() de Euler: o número de expoentes 1 S k S (()( n) tais que k é primo
(i) n = bc, com b, e> 2 primos entre si: como b e e são primos entre si,
com (()( n) é exatamente <p( (()( n)).
Para a primeira parte, sendo a uma raiz primitiva módulo n temos temos <p(n) = (()(bc) = (()(b)(()(c). Por outro lado, o fato de que b, e> 2
que ordn(a) = (()(n) e A = {a, a 2 ... , acp(n)} é um SCI módulo n. garante (cf. problema 19, página 88) que (()(b) e (()(e) são números
Portanto, qualquer raiz primitiva módulo n é congruente, módulo n, pares. Se a é um inteiro qualquer primo com n, então a é primo com
a um dos elementos de A, de maneira que basta ver quais elementos b e com e e, pelo teorema de Euler,
de A têm ordem (módulo n) igual a (()(n). Mas, pelo item (d) da acp(n)/2 = (acp(b)t(c)/2 - lcp(c)/2 = 1 (modb)
proposição 7.5, para 1 S k S (()(n) temos
e
ordn(ak) = (()(n) {::} mdc (<p(n), k) = 1. acp(n)/2 = (acp(c) t(b)/2 . 1cp(b)/2 - 1 (mod e) .

• Portanto, o item (h) da proposição 5.6 garante que acp(n)/ 2


em particular,
1 (mod n);

Exemplo 7.9. O exemplo 7.7 garante que 2 é raiz primitiva módulo ordn(a) S (()~n) < (()(n)
5. Como (()(5) = 4, a proposição acima ensina que um conjunto de
raízes primitivas módulo 5 duas a duas incongruentes é e, daí, a não é raiz primitiva módulo n.

{2\ 1:::;: k:::;: 4 e mdc(4,k) = 1} = {2,2 3 }. (ii) n = 2\ com k > 2: seja a um inteiro ímpar (i.e., primo com 2).
Se mostrarmos que
Portanto, módulo 5 as raízes primitivas duas a duas incongruentes são
2 e 3 (uma vez que 8 3 (mod 5)).
teremos
Prosseguimos, agora, na direção da caracterização dos inteiros que ord 2k(a) :::;: 2k- 2 < 2k-l = (()(2k),
I:!:."'
,, possuem raízes primitivas. Comecemos com uma condição necessária. e a não será raiz primitiva módulo 2k. Para o que falta, façamos
indução sobre k: o caso inicial k = 3 segue da proposição 5.9, uma
Teorema 7.10. Se n > 1 é um inteiro que possui raízes primitivas,
então n = 2, 4, pk ou 2pk, onde p é um primo ímpar e k é um natural.
=
vez que a 2 1 (mod8) para a ímpar. Suponha que já provamos que,
para algum inteiro k :?: 3, existe q E N tal que a 2k- 2 = 2kq + l; então
Prova. Note, inicialmente, que os inteiros n > 1 que não são de uma
ª2k-l ( ª2k-2) 2 = (2kq + 1)2 = 22kq2 + 2k+lq + 1
das formas do enunciado são ou da forma n = bc, com b, e > 2 intei-
ros primos entre si, ou da forma n = 2k, com k > 2 inteiro. Basta, 2k+1(2k- 1q2 + q) +1- 1 (mod2k+ 1 ),
pois, mostrarmos que tais inteiros n não possuem raízes primitivas,
coisa que faremos mostrando que todo inteiro a primo com n satisfaz conforme desejado.

178 Raízes Primitivas e Resíduos Quadráticos 7.2 Raízes primitivas 179

Resta estabelecer a recíproca do teorema acima, qual seja, que os Suponha que, para um certo k 2': 1, tenhamos a'P(Pk) = bkpk + 1,
números 2, 4, pk e 2pk, com p primo ímpar e k E N, possuem raízes tal que p f bk. A fórmula do binômio de Newton nos dá, então,
primitivas. É claro que 1 é raiz primitiva módulo 2 e 3 é raiz primitiva
módulo 4. O restante desta seção é devotado à análise dos demais acp(pk+l) = ( a'P(Pk) r = (1 + bkpk)P
casos.
A proposição a seguir garante que basta nos preocuparmos com os 1 + bkpk+l + I: (~)
j=2 J
l{pik + ~pPk.
números da forma pk.
Pelo exemplo 1.41, para 1 ::; j ::; p - 1 existe ck E N tal que
Proposição 7 .11. Se p é um primo ímpar e a E íZ é uma raiz pri-
(~) = pck. Portanto, a última expressão acima fornece
mitiva módulo pk, então a ou a + pk também é raiz primitiva módulo
2pk. p-1
1 + bkpk+l +L ckf4zJk+ 1 + ~pPk
Prova. Seja h = ord 2pk(a). Se a for ímpar, então a é primo com 2pk.

t
j=2
Nesse caso, usando o item (b) da proposição 7.5, juntamente com o
fato de ser ordpk (a) = cp(pk), obtemos 1 + bkp" 1+ ( ckú(P(j-1 )k-- 1+ fJ;p(p-1 Jk-2) p'+' _
Denotando por t a expressão acima entre parênteses e observando
que t E Z, segue finalmente que
onde, na última igualdade, utilizamos o fato de p ser um primo ímpar;
logo, ord 2pk(a) = cp(2pk) e a é raiz primitiva módulo 2pk.
Se a for par, troque a por a + pk no início e argumente como
acima. • Mas, como p f bk, fazendo bk+l = bk+tp obtemos a'P(Pk+i) = bk+iPk+l +
1, tal que p f bk+l· •
Completaremos a prova da recíproca do teorema 7.10 (i.e., a análise
do caso pk), em duas etapas. Antes, contudo, precisamos do seguinte Podemos finalmente cumprir o primeiro dos dois passos necessários
à demonstração da existência de raízes primitivas módulo pk, com p
Lema 7.12. Seja pum primo ímpar. Se a é uma raiz primitiva módulo primo ímpar.
p2 , então, para k 2': 1 inteiro, temos a'P(Pk) = bkpk + 1, com bk E íZ tal
Teorema 7.13. Seja pum primo ímpar e a um inteiro primo com p.
que p f bk.
(a) Se a for uma raiz primitiva módulo p, então a ou a+ p é raiz
Prova. Façamos indução sobre k 2': 1. O pequeno teorema de Fermat
primitiva módulo p 2 .
nos dá aP- 1 = b1p + 1 para algum b1 E Z. Se p dividisse b1 , teríamos
=
aP- 1 1 (modp2 ) e, daí, ordp2(a) ::; p-1 < cp(p2 ), contrariando o fato (b) Se a for uma raiz primitiva módulo p e módulo p 2 , então a é raiz
de a ser uma raiz primitiva módulo p 2 . primitiva módulo pk, para todo inteiro k 2': 1.
180 Raízes Primitivas e Resíduos Quadráticos 7.2 Raízes primitivas 181

Prova. de sorte que


(a) Segue do item (a) da proposição 7.5 e de nossas hipóteses que
ordp(a+p) = ordp(a) = p-1; por outro lado, o item (b) da proposição
Para o que falta, como a é raiz primitiva módulo p 2 , o lema anterior
7.2 garante que
nos dá a'P(Pk) = bkpk + 1, com bk E Z tal que p f bk; em particular,

ao passo que o item (b) da proposição 7.5 garante que



O teorema 7.13 nos coloca numa posição bastante boa. De fato,
se mostrarmos que o primo ímpar p possui uma raiz primitiva, a di-
Portanto, ordp2 (a) = p - 1 ou p(p - 1), o mesmo sendo válido para
gamos, então a+ p também o será (pelo item (a) da proposição 7.5);
ordp2 (a + p), e basta mostrarmos que
portanto, pelo item (a) do teorema anterior, podemos supor que a é
raiz primitiva módulo p e módulo p 2 , após o quê o item (b) do mesmo
resultado garante que a é raiz primitiva módulo pk, para todo inteiro
Mas, se ap-l 1 (modp2 ), então, módulo p2 , temos k ~ 1. Vejamos um exemplo.

Exemplo 7.14. Prove que 2 é raiz primitiva módulos 3k e 5k, para


+L P ~ l)pÍap-I-j
p-1 (
(a+ p)p-1 = ap-I + (p - l)pap-2
todo k EN.
j=2 J
ap-I + (p - l)paP- 2 Prova. Segue do exemplo 7. 7 e do teorema anterior que basta mos-
t
1 - paP- 2 1, trarmos que 2 é raiz primitiva módulos 9 e 25. Verifiquemos que 2 é
raiz primitiva módulo 9, sendo o caso do módulo 25 análogo: como
uma vez que p f a. cp(9) = 6, segue que ord9 (2) l 6; mas, uma vez que nenhum dos núme-
ros 21, 22 ou 23 é múltiplo de 9, segue que ord9 (2) = 6 = cp(9). •
(b) Suponha que a é uma raiz primitiva módulo p e módulo p2 e
provemos, por indução sobre k, que a é uma raiz primitiva módulo Para concluirmos a recíproca do teorema 7.10, resta apenas mos-
pk, para todo k ~ 1. Os casos k = 1 e k = 2 são nossas hipóteses. trar que primos ímpares possuem raízes primitivas. Como não é possí-
Suponha, pois, que já provamos ser a uma raiz primitiva módulo pk, vel darmos uma prova desta afirmação com o material de que dispomos
para um certo k ~ 2. até o momento, adiaremos a apresentação de uma demonstração para
Os itens (b) das proposições 7.2 e 7.5 nos dão a seção 7.3 do volume 6 (cf. teorema 7.20 de lá).
Finalizamos esta seção com dois exemplos que mostram a força dos
resultados aqui desenvolvidos.
182 Raízes Primitivas e Resíduos Quadráticos 7.2 Raízes primitivas 183

Exemplo 7.15. Se pé primo e n EN, prove que k E N é o expoente de p na decomposição canônica de n em fatores
primos. Então mdc (p, q) = 1 e
1n 2n ( l)n - { O(modp), se (p- l)tn
+ + ···+ p - = -1 (mod p), se (p - 1) 1 n 2n + 1 2n + 1
--2- EN::::} 2k EN.
n p
Prova. Se (p - 1) 1 n, digamos n = (p - l)k, o pequeno teorema de
Fermat nos dá, para 1 :::; a :::; p - 1, que Pelo pequeno teorema de Fermat, temos 2P 2 (modp) e, a partir
daí, uma fácil indução garante que 2Pk _ 2 (modp). Assim, módulo p
an = a(p-l)k - 1k = 1 (modp) temos
k
Ü - 2n + 1 = 2P q + 1 - 2ª + 1,
e, daí,
de modo que 22ª _ 1 (modp). Sendo t = ordp(2), segue, daí, que
t divide 2q e (pelo pequeno teorema de Fermat) t 1 (p - 1); logo,
p-1 t I mdc (2q,p- l). Mas, como os fatores primos de q são maiores que
p, isto força que t = 2 e, por conseguinte, t = 2 e p = 3. Portanto,
Se (p-1) f n, seja a uma raiz primitiva módulo p. ·Como o conjunto
{ a, a 2, ... , aP- 1 } é um SCI, módulo p, concluímos que os números a, a 2,
(7.1)
... , aP-l são, módulo p, congruentes, em alguma ordem, aos números
1, 2, ... ,P - 1. Portanto (ainda módulo p), temos
o que acarreta sucessivamente 23kª -1 (mod 32k) e

li
1n + 2n + ... + (p - 1 r - an + ª2n + ... + a(p-l)n = apn - an
---
an -1

Agora, segue do pequeno teorema de Fermat que Como já mostramos (cf. exemplo 7.14) que 2 é raiz primitiva mó-
dulo 32 k, segue da congruência acima que

por outro lado, uma vez que ordp(a) = p - 1 e (p - 1) f n, o item (b)


da proposição 7.2 garante que p f (an - 1). Logo, p 1
~~-
ª::=f,
conforme

Exemplo 7.16 (IMO). Ache todos os n EN tais que n 2 divide 2n+ 1.


. e, assim, 3k-l I q. Mas, como p = 3 e mdc (3, q)
k = 1. Assim, n = 3q e, por (7.1),

8ª + 1 EN.
= 1, devemos ter

--
2-
q
Solução. É claro que um tal n deve ser ímpar e que n = 1 é um valor
possível. Seja, pois, n > 1 um natural satisfazendo as condições do Suponha q > 1, e seja w o menor fator primo de q, digamos q = w 1v,
enunciado e escreva n = pkq, onde pé o menor primo que divide n e onde v E N e l é o expoente de w na decomposição canônica de q em
184 Raízes Primitivas e Resíduos Quadráticos 7.2 Raízes primitivas 185

fatores primos. Novamente pelo pequeno teorema de Fermat, temos (a) Mostre que f(O), f(l) E {O, 1} e que f é constante caso
8w 1 8 (mod w) e, daí, f(O) = 1 ou f(l) = O.
l
O - 8q + 1 = 8w v + 1 - 8v + 1 (mod w), (7.2) (b) Suponha, doravante, que f(O) = O e f(l) = 1. Use o pe-
queno teorema de Fermat para concluir que f(a) E {-1, 1},
de modo que 82v - 1 (mod w). Sendo t = ordw(8), segue que t 1 2v e para todo a E N tal que p f a.
(pelo pequeno teorema de Fermat) t 1 ( w - 1). Mas, como os fatores
primos de v (se existirem) são maiores do que w, chegamos a t = 1 ou (c) Se existe uma raiz primitiva a módulo p tal que f(a) = 1,
então
2, de modo que w 1 (8 1 - 1) ou w 1 (8 2 - 1). Em qualquer caso, segue
de w > p = 3 que w = 7 e, daí, (7.2) fornece f(x) = { O, se p Ix .
1, se p fx
O 8v + 1 1v + 1 2 (mod 7),
(d) Se f(a) = -1 para toda raiz primitiva a módulo p, então
um absurdo. Portanto, q = 1 e n = 3kq = 3.
• f(x) = { O, se p I x .
xP-l. (-l)ordp(x)+l (modp), se p f X

Problemas - Seção 7.2 6. (Turquia.) Prove que as duas afirmativas a seguir sobre n E N
são equivalentes:
1. Mostre que 2 é raiz primitiva módulo 29.
(a) n é livre de quadrados e, se p é um divisor primo de n,
2. Se m, n E N são tais que m I n, e a E Z é uma raiz primitiva
então (p - 1) 1 (n - 1).
módulo n, prove que a também é raiz primitiva módulo m.
(b) Para todo a EN, temos que n 1 (an - a).
3. Prove a seguinte generalização do teorema de Wilson: se n é
um natural que possui raízes primitivas e 1 = a 1 < a 2 < · · · < 7. (Índia - adaptado.) Para n E N, seja sn = 1 + E~=l kn-l. O
a<p(n) = n - 1 são os inteiros de 1 a n e primos com n, então propósito deste problema é caracterizar todos os n EN tais que
n I sn. Para tanto, faça os seguintes itens:

(a) Sepéprimoeq EN, mostrequep I E;~~ 1 Ef,:l(pj+zr- 1 .


4. (Romênia.) Encontre todos os primos p e q distintos de 2 e de 3
e tais que 3pq 1 (a 3pq-l -1), para todo natural a primo com 3pq. (b) Conclua que, se n I sn, então n é livre de quadrados.
(c) Seja n = P1 .. ·Pt, com P1 < · · · < Pt números primos, e
5. (OBM - adaptado.) Sejam p > 5 um primo tal que P; 1também
é primo, e f : Z+ ---+ :IR uma função tal que f(xy) = f(x)f(y) e
f(x + p) = f(x), para todos x, y E Z+. .
1. M ost re que Sn - 1 + Qi """'Pi-1
= L..,t=l zn-1 ( mo d Pi ) .
186 Raízes Primitivas e Resíduos Quadráticos 7.3 Resíduos quadráticos 187

ii. Se a é uma raiz primitiva módulo Pi, mostre que tem número de soluções igual a um múltiplo de p, onde duas
ap;(n-1) _ an-1) soluções (a 1 , ... , a 5 ) e (b 1 , ... , b5 ) são consideradas distintas se
Sn - 1 + qi (
an-I _ l (modpi)· existir 1 ::::; i ::::; 5 tal que ai 1- bi (modp). Para tanto, faça os
seguintes itens:
iii. A partir de ii., conclua sucessivamente que, se n I sn,
então pi 1 (an- 1 -l), (pi-1) 1 (n-1) e (Pi-1) 1 (qi-1). (a) Se J(x 1 , ... ,x 5 ) = l-(xf+xi+· · ·+xg)P-I em é o número
iv. Se n I sn, use os resultados dos itens i. e iii. para de soluções da congruência do enunciado, então
concluir que Pi I (qi - 1). m - L f(x1, ... , x 5 ) (modp),
v. Prove, a partir dos dois itens anteriores, que n I sn se, x1, ... ,x5EA

e só se, Pi(Pi - 1) 1 (qi - 1) para 1 :Si ::::; t. onde A= {1,2, ... ,p-1}.
8. Sejam p um primo ímpar, k -=I p, 2p um natural tal que 1 ::::; k < (b) Módulo p, temos
2(p + 1) e n = 2pk + 1. m - p5- X 4a1 x4a5
t.,, 1 " · 5
0<1 + .. ·+a5=p-l x1, ... ,x5EA
(a) Se n é primo e a é uma raiz primitiva módulo n, prove.que
mdc (ak + 1, n) = 1. ~
L.J(L.J
~ x4°'1) ... ( L.J
~ x4°'5)
(b) Suponha que existe 2 ::::; a < n inteiro, tal que akP = 0<1 +·+a5=p-l xi EA
1
x5EA
5 ·

-1 (mod n) e mdc (ak + 1, n) = 1. (c) Se a 1 + · · · + a 5 = p- l, com ai 2:: O para 1::::; i::::; 5, então
i. Se d= ordn(a), mostre que d 1 (n-1) e d f 2k. Conclua existe 1 ::::; i ::::; 5 tal que ai= O ou (p - 1) f 4ai·
que p I d e, daí, que p 1 <p(2kp + 1). (d) Se (p - 1) f 4a, use uma raiz primitiva a módulo p para
ii. Use a fórmula para <p(2kp+ 1) para concluir que existe concluir que
l > 1 inteiro tal que lp + 1 é um divisor primo de n. p-l
a4pa -a4a
111. Use que n = 2kp+ 1 para mostrar que n = (lp+ l)(hp+ L x4°' = L a4ja =
a4a - 1
O(modp).
1) para algum inteiro h E {O, l}. Conclua, em seguida, xEA j=l

que não podemos ter h = 1 e, daí, que n é primo. (e) Conclua quem_ O(modp).
O próximo problema é devido ao matemático francês do século
XX Claude Chevalley, sendo conhecido como o teorema de 7.3 Resíduos quadráticos
Chevalley.
Estudamos, nesta seção, congruências algébricas da forma
9. Se p é um primo ímpar, queremos mostrar que, módulo p, a
congruência x2 a (mod n).
Precisamos, inicialmente, da definição a seguir.
188 Raízes Primitivas e Resíduos Quadráticos 7.3 Resíduos quadráticos 189

Definição 7.17. Se a, n E Z, com n > 1 e mdc (a, n) = 1, diremos (b) Como {1, 2, ... ,p-1} é um SCI módulo p, para contarmos quantos
que a é um resíduo quadrático módulo n se a congruência de seus elementos são resíduos quadráticos, módulo p, basta calcular-
mos quantos dentre os números 12 , 22 , ... , (p - 1)2 são dois a dois
x2 - a(modn) incongruentes, módulo p. Para tanto, veja que, se 1 ::::; i ::::; P; 1, então

possuir pelo menos uma solução inteira x. Caso contrário, a é dito um i2 - (p - i) 2 (modp);
não resíduo quadrático módulo n.
por outro lado, se 1 ::::; i < j ::::; P;1, então
Em relação à definição acima, nossa tarefa primordial será obter
i2 -=/=. j2 (modp),
condições necessárias e suficientes para que um inteiro a seja resíduo
quadrático módulo n. Analisemos o caso em que n é primo, deixando uma vez que j 2 - i 2 = (j - i) (j + i) e O < j - i < j + i < p.
o caso geral para os problemas propostos ao final desta seção. Logo, há precisamente P; 1 resíduos quadráticos módulo p e, daí,
(p - 1) - (P; 1) = P;1 não resíduos quadráticos módulo p. •
Proposição 7.18. Seja pum primo ímpar.
O resultado a seguir é devido a L. Euler, sendo conhecido na lite-
(a) Se a é um resíduo quadrático módulo p, então a congruência ratura como o critério de Euler para resíduos quadráticos.
x 2 - a (modp) possui exatamente duas soluções incongruentes
módulo p. Proposição 7.19 (Euler). Se pé um primo ímpar, então um inteiro
a é resíduo quadrático módulo p se, e só se,
(b) Dentre os números 1, 2, ... ,P - 1 há exatamente P;1 resíduos p-1

quadráticos e P; 1não resíduos quadráticos módulo p.


a-2 - 1 (modp).

Prova. Suponha primeiro que a é resíduo quadrático módulo p. En-


Prova.
tão, mdc (a,p) = 1 e existe x0 E Z tal que x~ _ a (modp). Em
(a) Se x 1 e x 2são inteiros tais que Xi a (modp) e x~ - a (modp),
particular, x 0 também é primo com p, e o pequeno teorema de Fermat
então Xi - X~ (modp), de sorte que P 1 (xi - x~). Mas, como Pé
fornece
primo, segue que p 1 (x 1 - x 2) ou p 1 (x1 + x2). Portanto, a con-
a.e.=! (x 02 )-2 = xg- _ 1 (modp).
p-l 1
2 -
gruência do enunciado possui no máximo duas raízes incongruentes,
quais sejam, x 1 e p - x 1 . Por outro lado, sendo a um resíduo qua- Reciprocamente, suponha que a9 1 (modp), de sorte que, em
drático módulo p, sabemos que tal congruência admite uma solução particular, mdc (a, p) = 1. Então, sendo a uma raiz primitiva módulo
x 0 E Z. Segue imediatamente que p - x 0 também é solução da refe- p, temos que {a, a 2, ... , aP-l} é um SCI módulo p e, daí, existe um
rida congruência, com p - x 0 -=/=. x 0 ( mod p), uma vez que p é ímpar e inteiro 1 ::::; k::::; p - 1 tal que ak - a (modp). Portanto,
mdc (xo,P) I mdc (a,p) = l.
a k( -p-1)
2 - a .e.=!
2 1 (modp),
190 Raízes Primitivas e Resíduos Quadráticos 7.3 Resíduos quadráticos 191

e segue de ordp(a) = p - 1 que (p - 1) 1 k (P; 1), de maneira que k é A conveniência notacional do símbolo de Legendre é esclarecida
par, digamos k = 2l. Fazendo x 0 = o/, temos pelas duas próximas proposições.
x~ = a 21 = ak - a (modp), Proposição 7.23. Se pé um primo ímpar e a, b E Z, então:
de sorte que a é um resíduo quadrático módulo p.
Corolário 7.20. Se p é um primo ímpar, então um inteiro a primo

com p é um não resíduo quadrático módulo p se e só se (b) Se p f a, então ( ~) = 1.
p-1
a-2 = -1 (modp). (c) (:) _ ap;i (modp).
Prova. Pelo pequeno teorema de Fermat, temos
(ap;i - l)(ap;i + 1) = ap-l - 1 - O(modp).
(d) (:) 0) = (~).
Prova. A prova dos itens (a) e (b) é imediata. Provemos, pois, o item
Mas, como p é primo, temos que (c): se p I a, então

(pª)
p-1 p-1
a-2 - 1 - O(modp) ou a-2 +1_ O(modp).
=O_ a-2
p-1
(modp);
Por outro lado, pelo critério de Euler, a é um não resíduo quadrá-
tico módulo p se, e só se, a primeira das duas congruências acima não senão, a proposição 7.19 e o corolário 7.20 garantem que, modulo p,
p-1
ocorre, i.e., se, e só se, a-2- + 1 - O(modp).

Observação 7.21. É possível provar (cf. problema 8.2.7 do volume


ª
p-1
2
-
{ 1 se a for resíduo quadrático módulo p
-1 se a for não resíduo quadrático módulo p =
(ª)p ·
6) que, para todo a E Z, existem infinitos primos p tais que a não é
resíduo quadrático módulo p. Por fim, quanto a (d), observe que, se p I ab, então p Ia ou p I b e,
daí,
A fim de facilitar as manipulações com resíduos e não resíduos qua-
dráticos, precisamos de uma notação conveniente, a qual é introduzida (~) (!) = o= (:) .
pela definição a seguir. Por outro lado, se p f ab, então p f a e p f b, e segue do item (c) que
Definição 7.22. Dados inteiros a e p, com p primo, definimos o sím-
bolo de Legendre (:) por: (ª)p (b)p = a-2
p-1
. bE=.!_
2 = (ab)-2 _
p-1 (ªb)
p (modp).

1, se a for resíduo quadrático modulo p


-1, se a for não resíduo quadrático modulo p
Portanto, p divide a diferença (:) O) - (~),
a qual assume um dos
valores inteiros de -2 a 2. Mas, como pé ímpar, concluímos que deve
O, se p Ia
ser (:) O) - (~) = O. •
192 Raízes Primitivas e Resíduos Quadráticos 7.3 Resíduos quadráticos
193
O exemplo a seguir mostra como utilizar as propriedades do sím- Prova. Para 1 < j < P- 1 se· p-1 < t p-1 . .
bolo de Legendre deduzidas na proposição acima. - - 2' Ja --2- - j ::::; - 2- o mte1ro tal que

Exemplo 7.24. Prove que não existem x e y inteiros tais que y 2 = ja - tj (modp).
x 3 +7. Provemos primeiro que
Solução. Suponha o contrário. Então x é ímpar, pois, do contrário,
teríamos y2 = x 3 + 7 - 3(mod4), uma contradição ao item (c) da 1 ::::; i < j ::::; p; 1 :::} /ti/ =1- /tjl·
proposição 5.9.
Por outro lado, Para tanto, basta ver que, como mdc (a, p) = 1, temos

(7.3) /ti/= /tj/ :::} /ia/ - /ja/ (modp):::} /i/ - /j/ (modp)
:::} i ± j - O(modp),
e, como (x - 1)2 + 3 _ 3(mod 4) (lembre-se de que x - 1 é par), existe
um primo p da forma 4k + 3 tal que p divide (x - 1) 2 + 3. o que é impossível.
Voltando a (7.3), concluímos que y 2 + 1 O (modp), i.e., que -1 Segue do que fizemos acima que os números /t 11, ... , /t E=! I 1ormam
e
é resíduo quadrático módulo p. Segue da definição do símbolo de em alguma ordem, uma permutação dos números 1 2 2 p-1 ( . '
h , , ... , veJa
Legendre e do item (c) da proposição anterior que, módulo p, que nen um deles é igual a O, já que mdc (a,p) = 1). P~rtanto
lembrando que m denota a quantidade de , d' . .'
m ices J para os quais
1= ( ~1) - (-1/;1 = (-1)2k+l = -1, tj < O, temos, módulo p, que

uma contradição.

Nosso próximo resultado, devido a K. F. Gauss e conhecido na


e, daí,
literatura como o lema de Gauss, fornece um procedimento bem E=!
mais simples que o critério de Euler para decidir se um certo inteiro é a 2 - (-l)m (modp).
resíduo ou não resíduo quadrático módulo p, sendo pum primo ímpar O resto segue do item (c) da proposição 7_23 _
dado.
,Como aplicação do lema de Gauss, mostramos a seguir que 2 é

Proposição 7.25 (Gauss). Se pé um primo ímpar e a é primo com res1duo quadrático módulo p se, e só se, p O, 1, 2 ou 7 (mod 8).
p, então ( ~) = ( -1 )m, onde m é o número de elementos do conjunto
Exemplo 7.26. Se pé um primo ímpar, então

{1 ::::; J.::::; -p-2 -;1Ja. - -1, -2, ... ou - (p- -2 -1) (modp) }
. (7.4)
(p2) =(-1)-4 =(-1)~. lp+lj 2 1
194 Raízes Primitivas e Resíduos Quadráticos 7.3 Resíduos quadráticos 195

Prova. Pelo lema de Gauss, temos(~)= (-l)m, onde Lema 7.27. Se pé um primo ímpar e a é um inteiro primo com p e
também ímpar, então (~) = (-l)M, onde
m=#{lsksP;\2k -1,-2, ... ou - ( p ; l ) (modp)}.

Se 1 S k S lP~ 1 J, então 2 S 2k S 2lP~ 1 J S p;I e, daí, M= t ljªJ ·


j=l p

2k-:/=. -1, -2, ... , - ( p-1)


- 2- (modp). Prova. Para 1 S j S P; 1 , seja O < rj < p o resto da divisão de ja
por p, de maneira que (cf. proposição 1. 7)
Se lp-IJ
4
+ 1 -< k -< p-I
2 '
então

p - 1 2:: 2k 2:: 2 ( lp ~ l J + 1) > 2 (p ~ l - 1) + 2 = p; 1; . lJªJ


Ja=p p+rj. (7.5)

em particular, 2k-:/=. 1, 2, ... , P; 1 (modp), ou, o que é o 1?-esmo, É imediato que

Jª _ -1, -2, ... , - (p-1)


. = - 2- - 2 -1
(modp) {:} p+ S rj < p.
2k -= -1, -2, ... ou - (p-1)
- 2- (modp).

Portanto, Portanto, nas notações do lema de Gauss, há exatamente m índices


1 S j S P; 1 tais que P! 1 ~ rj < p, e basta mostrarmos que a paridade
m_-
p- 1 - ( lp
- -
2
-- -
4
1 +1 J ) + 1_-p --2 -1 - lp--4-1J = lp
-+-
4
J
1
' do número de tais índices é, por outro lado, igual à de Ljc!
p-1
1
lJ;- .

onde, para a última igualdade, basta considerarmos separadamente os Para o que falta, denote por 8 1 , 8 2 , ... , 8m (após uma reenumera-
casos p = 4k + 1 e p = 4k + 3. ção, se necessário) os restos rj tais que P! 1 S rj < p, e por t 1 , ... ,tn
Quanto à segunda igualdade, é suficiente mostrarmos que (também após uma reenumeração, se necessário) os restos rj tais que

l lJ -1
1 S rj S P; 1 . Então m + n = P; 1 e (assim como na prova do lema de
+- Gauss), para 1 Si< j S P; 1 temos ri-=/=- rj,p- rj. Logo,
2
p- - p 8 (mod 2).
4

Para tanto, basta considerarmos separadamente os casos p = 8k + 1, { 1, 2, ... , p-1}


- 2- = {p - 81, P - 82, ... , p - 8m} U {t1, t2, ... , tn},
p = 8k + 3, p = 8k + 5 e p = 8k + 7. •
Prosseguindo em nosso estudo de resíduos quadráticos, vamos pro- uma união disjunta, e, daí,
var um dos mais famosos teoremas de Gauss, o qual estabelece uma
relação simples entre os símbolos de Legendre ( !)
e (~), onde p e q
são primos ímpares. Comecemos reformulando o lema de Gauss.
Raízes Primitivas e Resíduos Quadráticos 7.3 Resíduos quadráticos 197
196

Por outro lado, somando as igualdades (7.5) sobre 1 ::S j ::S P;1, É, então, suficiente mostrarmos que
obtemos
a (p -l)
2

8
=p t ljaj
j=l p
+ f
j=l
Sj + ttj,
j=l
(7.6)

Subtraindo membro a membro as duas relações acima, segue então para o quê utilizamos o seguinte argumento geométrico: como o se-
que gundo membro da igualdade acima é igual ao número de pontos de

(a - 1) (p2 8- 1) = p t ljªJ - mp +2 f si;


coordenadas inteiras no retângulo fechado

j=l P
por fim, lembrando que a e p são ímpares e analisando a igualdade
J=l
R = { (x, y) E Il~.2; 1 :'.S x ::S p; l e 1 ::S y ::S q; l},

acima módulo 2, obtemos basta contarmos o número de tais pontos de outra maneira, obtendo

p;l ljªJ
o primeiro membro de (7.6) como resultado.
O=L -p -m, Sem perda de generalidade, suponha que p > q, e considere a reta
j=l y !x (cf. figura 7.1). Para cada j ~ 1, o número l1J conta o .

conforme desejado. • y
O teorema a seguir é conhecido na literatura como a lei da re- q-l (
p-1 (p-l)q)
-2- ........--------"-! 2 ' 2p
ciprocidade quadrática de Gauss, e a prova que apresentamos é
devido ao matemático alemão do século XIX Ferdinand Eisenstein.

Teorema 7.28 (Gauss). Se p e q são primos ímpares e distintos, então

(~) (~) = (-l)(p;l)(~). !l


p

1 J p-1
-2-
Prova. Pelo lema anterior, temos
Figura 7.1: contando os pontos de n n 'li}.

número de inteiros positivos menores ou iguais que jq;


p
por outro lado,
onde
q;l
m=I:: -
ljpj en=I:-·
~ ljqj para 1 ::::; j ::::; P; 1 temos que i:
(j. Z e, daí, l1
J conta o número de
j=l p pontos de coordenadas inteiras situados sobre a reta x = j, abaixo da
j=l q
,!
198 Raízes Primitivas e Resíduos Quadráticos 7.3 Resíduos quadráticos
199

reta y = '1.x
p
e acima da reta y = O. Mas, como e, pela lei da reciprocidade quadrática,

q-1
2
(p3) .(P)3 (
= -1) (p-1)(3-1)
-2
p-1
-2 =(-1)-2.

todos os pontos que estamos contando de fato pertencem ao retângulo Assim,


n. Portanto, nas notações da figura 7.1, temos 3
(~ ) = (-l)Y (E)-\-1)Y =(E).
p-1
t. 1
J=
ljqj =
p
# de pontos de coordenadas inteiras na região Ide n. Por fim, p - -1 (mod3) implica, pelo item (a) da proposição 7.23
que (U = (-31) = -1; por sua vez, tal igualdade contradiz o fato d~
-3 ser um resíduo quadrático módulo p. •
Analogamente,
q-1

t
j=l
ljpj =
q
# de pontos de coordenadas inteiras na região II de R
Problemas - Seção 7.3
e nada mais há a fazer.
• 1. Se P é um primo ímpar, prove que -1 é resíduo quadrático mó-
Terminamos esta seção com uma aplicação da lei da reciprocidade dulo p se, e só se, p 1 (mod4).
quadrática à existência de certos tipos de números primos.
2. Prove que a equação x 2 = y 3 + k não admite soluções inteiras
Exemplo 7.29. Prove que há infinitos primos da forma 3k+ 1, k EN.
x, y para infinitos valores inteiros de k.
Prova. Por absurdo, suponhamos que houvesse somente uma quan-
tidade finita de primos da forma 3k + 1, digamos p 1,p2, ... ,Pn, e seja 3. Sejam a, b e e inteiros não todos nulos e n um natural. Se
x = (2p 1 ... Pn) 2 + 3. Sendo p um divisor primo de x, é claro que existem inteiros x e y, relativamente primos com n e tais que
p-=/=- 2, 3,pi, ... ,Pn, de modo que p - -1 (mod3). Ademais, como ax 2 + bxy + cy 2 = n, prove que b2 - 4ac é um resíduo quadrático
módulo n.
(2p1 .. ·Pn) 2 - -3 (modp),
4. Prove que não existem x, y E Z tais que x 2 + 3xy - 2y2 = 122.
temos que -3 é resíduo quadrático módulo p. Devemos, então, ter
5. Dados a, b E Z, mostre que:
(-;3 ) = 1. Mas
(a) 2b2 + 3 tem um divisor primo p, tal que p ±3 (mod 8).
(~3) (~1) (i), (~1) =(-1/;
1
(b) (2b2 + 3) f (a 2 - 2).
200 Raízes Primitivas e Resíduos Quadráticos 7.3 Resíduos quadráticos 201

6. Se p / 3 é um primo ímpar, prove que 11. (APMO.) Durante um recreio, o professor reuniu seus n alunos
no pátio da escola, formando com os mesmos um círculo. Em
se p _ ±1 (mod 12) seguida, escolheu um aluno, deu a ele um doce e, no sentido anti-
se p _ ±5 (mod 12) horário, saiu distribuindo doces aos alunos da seguinte maneira:
pulou um aluno e entregou um doce ao terceiro, pulou dois alunos
7. Sejam m e n naturais ímpares, com n > 1. e entregou um doce ao sexto aluno, pulou três alunos e entregou
um doce ao décimo e assim por diante, sempre pulando um aluno
(a) 2n - 1 tem um divisor primo p, tal que p - ±5 (mod 12). a mais que na vez anterior e entregando um doce ao próximo.
Para que valores de n cada aluno terá recebido ao menos um
(b) (2n - l)t (3m - 1).
doce após um número finito e suficientemente grande de voltas?
8. Se n > 1 é um natural tal que p 2n + 1 é primo, faça os 12. Dados um primo p e inteiros primos entre si a e n, com n > 1, di-
seguintes itens: zemos que a é um resíduo n-ésimo, módulo p, se a congruência
xn a (mod p) tiver solução. Generalize o critério de Euler para
(a) Mostre que 3 é um não resíduo quadrático, rríódulo p.
resíduos quadráticos, do seguinte modo: se d= mdc (n,p - 1),
(b) Conclua que 3 é raiz primitiva, módulo p. então
a é um resíduo n - ésimo módulo p {::} av·t - 1 (modp).
9. (a) Sejam a e k inteiros dados, sendo a ímpar. Prove que a
congruência x 2 a (mod 2k) tem solução para todo k > 2 Para o próximo problema, o leitor necessitará utilizar alguns
se, e só se, a congruência x 2 _ a (mod 8) tem solução. fatos básicos sobre as somas simétricas elementares das raízes
(b) Sejam p um primo ímpar e a um inteiro primo com p. Prove de um polinômio, para o quê sugerimos a leitura da seção 4.2 do
que a congruência x2 - a (modpk) tem solução para todo volume 6.
k 2': 1 se, e só se, a congruência x 2 _ a (modp) tem solução.
13. (Bulgária - adaptado.) O objetivo deste problema é mostrar que,
se p > 5 é primo e k > 1 é inteiro, então p 3 k). Para
10. * Sejam a e n naturais primos entre si, com n > 1, e n =
1 ( (;) -

2kp~ ••• p~t a decomposição canônica de n em fatores primos.


1
tanto, faça os seguintes itens:
Prove que a congruência x 2 - a (mod n) tem solução se, e só se, (a) Suponha, até o item (f), que k é ímpar. Conclua que
forem satisfeitas as seguintes condições:
(';) - k = (p ~ l)! (!(a) - f(-a)),
(i) a = 1 (mod2) se k = 1, a - 1 (mod4) se k = 2 ou a=
1 (mod 8) se k 2': 3. onde f(x) = TI;:i (x + (k-;_I)p + j), um polinômio de coe-
p·-1
(ii) a~ 1 (modpi), para 1 ~ i ~ t. ficientes inteiros, e a = (k-;_I)p.
202 Raízes Primitivas e Resíduos Quadráticos 7.4 Somas de quadrados 203

(b) Se f(x) = xP- 1+ap-2xP- 2+ap-3xP- 3+· · ·+a1x+ao, mostre (b) -1 é resíduo quadrático módulo p.
que f(a) - f(-a) - 2a1 a (modp3 ) e deduza, a partir daí,
(c) p pode ser escrito como soma de dois quadrados.
que basta mostrar que a 1 - O(modp2 ).
(c) Use as relações de Girard (cf. proposição 4.6 do volume Prova.
6) para mostrar que a condição a 1 O(modp2 ) equivale à (a)::::} (b): sendo p = 4k+ 1, segue do item (c) da proposição 7.23 que
E=!
congruência

i).
I::j~l ri - O(modp), onde ri = f1 1 :s;i::;Sl (a+
i=/cj,p-j (p-1)· _ p-1
(-1)-2 - (-1) 2k 1 (modp)

p-1
(d) Mostre que a igualdade (a+ j)(a+p-j)ri = IJi~i (a+i) e, daí, -1 é resíduo quadrático módulo p.
implica, módulo p, a congruência j2rj _ -(p-1)! (modp),
para 1 5:. j 5:. P; 1. (b) ::::} (c): sejam h E Z tal que h 2 + 1 - O(modp), e
(e) Conclua, a partir do item (d), que ri é um resíduo quadráti- A= {(x,y); x,y E Z, O 5:. x,y < Jp}.
co, módulo p e ri:/:- ri (modp), para todos, 1 5:_·i < j 5:_ ·P;1.
(f) Deduza que, módulo p, o conjunto {r1 , r 2 , •.. , Tp-1} forma Pelo princípio fundamental da contagem (cf. corolário 1.9 do volume
2
4), temos IAI = (lJpj + 1)2. Agora, como
uma permutação do conjunto {1 2, 22, ... , (P; 1)2} e conclua
o que se pede. (lJ.PJ + 1)2 > Jp2 = p
(g) Se k for par, siga os passos delineados nos itens de (a)
a (f), argumentando, de início, com o polinômio f(x) = e só há p possíveis restos numa divisão por p, o princípio da casa
dos pombos (cf. seção 4.1 do volume 4) garante a existência de pares
n~=~ (x + k: - J). ordenados distintos (x 1 , y1 ), (x 2 , y2 ) E A, tais que

7.4 Somas de quadrados hx1 + Y1 hx2 + Y2 (modp).


Como aplicação das ideias desenvolvidas até aqui, caracterizaremos Fazendo a= lx1 - x2I e b = IY1--'- Y2I, temos a e b não ambos nulos e,
nesta seção os naturais que podem ser escritos como soma de dois daí,
r:
'!
quadrados. Comecemos pelo caso dos números primos, com o seguinte
resultado de Fermat. ·

Teorema 7.30 (Fermat). As seguintes condições sobre um primo ím- Mas, como
par p são equivalentes: + b2
ª2 lx1 - x21 2 + IY1 - Y21 2 - (x1 - x2) 2 + (hx1 - hx2) 2
(a) p _ 1 (mod4). (h 2 + l)(x 1 - x2 ) 2 . O(modp),

i
204 Raízes Primitivas e Resíduos Quadráticos 7.4 Somas de quadrados 205

a única possibilidade é que seja a2 + b2 = p. Prova. No que segue, seja


_ 2ª a1 ªk b1 b1
n- Pi · · · Pk Q1 · · · ql
(c) =} (a): se p = a2 + b2 , com a, b E Z, então a é par e b é ímpar
a decomposição de n em fatores primos, com a, ai, bj ~ O, Pi
ou vice-versa (lembre-se de que p é ímpar). Supondo, sem perda de
1 (mod4) e qj 3 (mod4), para todos 1 Si S k e 1 S j S l.
generalidade, que a é par e b é ímpar, segue da proposição 5.9 que

p = a2 + b2 - O+ 1 _ 1 (mod4). (i) Se cada bj for par, então n pode ser escrito como soma de dois
quadrados: note inicialmente que, pelo teorema 7.30, cada Pi pode

• ser escrito como soma de dois quadrados; por outro lado, temos 2ª =
(2ª12 ) 2 + 02 se a for par e 2ª = (2(a-l)/ 2 ) 2 + (2(a-l)/2 ) 2 se a for ímpar;
Para prosseguir, precisamos de um resultado auxiliar, usualmente por fim, se bj = 2cj, com Cj E Z para 1 ::S j ::S l, então qJi = (q?)2+0 2 .
atribuído a Euler e conhecido como a identidade de Euler. Portanto, aplicando repetidamente o lema 7.31, concluímos que n pode
ser escrito como soma de dois quadrados.
i':;i Lema 7.31. Sem e n são naturais que podem ser escritos como somas
de dois quadrados, então mn também pode ser escrito como soma de (ii) Se n puder ser escrito como soma de dois quadrados, então cada bj
dois quadrados. é par: é suficiente provar que se n pode ser escrito como soma de dois
quadrados e bj ~ 1, então bj ~ 2 e ; também pode ser escrito como
Prova. Sem= a2 + b2 e n = c2 + d2 , então J

soma de dois quadrados. Para tanto, se n = c2 + d2 , com e, d E Z,


mn = (a 2 + b2 )(c2 + d2 ) então c2 + d2 _ O(modqj)· Se d ,1: O(modQj), então mdc (d, qj) = 1,
de sorte que d é invertível, módulo qj; sendo f seu inverso, ~ódulo qj,
= ((ac) 2 + (bd)2) + ((ad) 2 + (bc) 2 ) (7.7)
obtemos
= ( ac + bd)2 + (ad - bc)2. (cj)2 + 1 O(modqj),

• em contradição ao teorema 7.30, uma vez que qj _ 3 (mod4). Por-


tanto, d _ O(modqj) e, daí, e - O(modQj). Logo, n = c2 + d2 _
O teorema a seguir, também atribuído a Fermat, dá uma condição O(modq;), de forma que bj ~ 2 e
necessária e suficiente para um natural poder ser escrito como soma
de dois quadrados.

Teorema 7.32 (Fermat). Um natural n pode ser escrito como soma de


dois quadrados se, e só se, n = 1 ou n é tal que todo primo congruente Terminemos esta seção mostrando que a maneira de escrever um

a 3 módulo 4 e que comparece na fatoração canônica de n o faz com primo da forma 4k + 1 como soma de dois quadrados é essencialmente
i
i1,
expoente par. única.
;i
., , ,j'
1- _I
Raízes Primitivas e Resíduos Quadráticos 7.4 Somas de quadrados 207
206

· - 7 . 33 · Se p é um primo da forma 4k + 1, então existem Problemas - Seção 7.4


P ropos1çao
únicos x, y EN tais que x < Y e x 2 + Y2 = P·
1. O propósito deste problema é mostrar que nem todo natural
Prova. Já sabemos que existe ao menos um par de naturais x, Y tais pode ser escrito como soma de três quadrados. Para tanto, prove
que x 2 + y2 -_ p . SeJ·a , então , a , b um outro tal par e observe que a, o seguinte teorema de Euler: não existem inteiros k, l, x, y, z tais
b, x e y são todos primos com p e menores que Jp. Escolha inteiros que l ;::=: O e
1 < e z < p tais que xz y e ac - b (modp).
- Afirmamos que e= z ou e+ z = p. De fato, módulo p, temos
2. Se a, b, e E N são tais que a( a - 1) = b2 + c2 , mostre que a+ b é
x2 + y2 - x2 + (xz)2 = x2(z2 + 1), ímpar.
de maneira que z2 - -1 (modp). Analogamente, c2 -1 (modp), de
3. (BMO.) Encontre todos os pares de naturais distintos x e y, tais
modo que p divide z2 - c2 = (z - c)(z + e) e, daí, p divide z - e ou 2
+Y2 seJ· a um inteiro divisor de 1995.
que x x-y
z + e. Mas, como 1 :::; e, z < p, segue que -p < z - e <_ z + e < 2p,
acarretando em z - e = O ou z + e = P· 4. Dado n natural, prove que existem n naturais consecutivos, tais
Suponha, agora, que e = z. As escolhas de e e z garantem que que nenhum deles pode ser escrito como soma de dois quadrados.

bxz = acy _ ayz (modp) (7.8)


5. O propósito deste problema é provar um teorema, devido a Jo-
seph L. Lagrange, que afirma que todo natural pode ser escrito
e daí, bx _ ay (modp). Mas, uma vez que O< a, b, x, y < Jp, temos
1

como uma soma de quatro quadrados. Para tanto, observamos


o'< bx, ay < p e, por conseguinte, bx = ay. Assim, inicialmente que vale a seguinte generalização do lema 7.31: se

p = x2 + y2 = (ª:f + y2 (*)2 (a2 + b2) (*)2 p


= =
dois naturais m e n podem ser escritos como somas de qua-
tro quadrados, então mn também pode ser escrito como uma
soma de quatro quadrados. De fato, se m = a 2 + b2 + c2 + d2
e, daí, y = b e x = a.
Se z + e = p, então, argumentando como em (7.8), chegamos a e n = w 2 + x 2 + y 2 + z 2, o leitor pode verificar sem dificuldade
bx - -ay (modp) e, daí, a bx + ay = p. Então, (7.7) fornece que 1

p2 = (a2 + b2)(x2 + y2) = (bx + ay) 2 + (by - ax) 2 = p 2 + (by - ax)2, mn = (aw - bx - cy - dz) 2 + (ax + bw + cz - dy) 2
(7.9)
+ (ay - bz + cw + dx) 2 + (az + by - ex+ dw)2.
ou seja, by = ax. Novamente como acima, concluímos que x = b~
y = a. De posse de tal resultado, faça os seguintes itens:
1 Para uma prova natural da identidade em questão, referimos o leitor ao pro-

blema 1.1.9 da segunda edição do volume 6.


208 Raízes Primitivas e Resíduos Quadráticos

(a) Sejam p primo, S = {x 2 ; x E Zp} e S' = {-1- y; y E S}.


Prove que S n S'-=/:- 0.
(b) Conclua que, se pé primo, então existem x, y, m E Z tais
que 1 :'.S m < p e x 2 + y 2 + 1 = mp.
(c) Sejam p um primo e 1 < m < p tal que mp = xf + x~ +
x~ + x~, onde os x/s são inteiros. Se -~ < Yi < 1; é tal CAPÍTULO 8
que Xi Yi (mod m), prove que existe 1 :'.S r < m tal que
rp = Yi + Y~ + Yi + Yl ·
(d) Mostre que todo número natural pode ser escrito como uma
soma de quatro quadrados.
Sugestões e Soluções
6. O propósito deste problema é encontrar todos os n E N para os
quais existe m E N tal que 2n - 1 divide m 2 + 9. Para tanto,
faça os dois itens a seguir:

(a) Mostre, por contradição, que, se 2n - 1 divide m 2 + 9 para


algum m E N, então n é uma potência de 2.
(b) Se n = 2k mostre, por indução sobre k, que existe mk EN Seção 1.1
tal que 22k - 1 dividem~+ 9.
1. Se n = (akak-I ... a1ao)10 é a representação decimal do natural n,
- k .
entao n = I:j=O ªi l()J. Use, agora, o resultado do exemplo 1.4 e o
corolário 1. 9.

2. Escreva lOk = (11 - l)k e desenvolva o binômio do segundo membro.

3. Escreva n = I:J=o ajloi e use o resultado do problema anterior, jun-


tamente com o corolário 1.9.

4. Seja n = (abc)io a representação decimal de n; use o resultado do


problema anterior para concluir que a - b +e= O ou 11.

5. Se n tiver k algarismos, use as condições do enunciado para mostrar


que gk 2: 102 (k-l) - lOk - 22 e, daí, concluir que k = 1 ou 2.

6. Para a EN, comece observando que (a+ l)(a+2) ... (a+n) = (a!~)!.

209
Sugestões e Soluções 211
210

20. Use o resultado do exemplo 1.10.


7. Comece mostrando que um número natural é múltiplo de 10 se, e só
se, for simultaneamente múltiplo de 2 e de 5. Assim, como o número 21. Para o item (a), escreva n = 2rq, onde ré um inteiro não negativo e
em questão é claramente par, basta mostrarmos que é um múltiplo q um naturalímpar. Conclua, então, que q = l.
de 5; para tanto, use o resultado do exemplo 1.3.

8. Use o resultado do item (b) do exemplo 1.3.

9. No item (c), para mostrar que Se Zm, use o algoritmo da divisão e Seção 1.2
a minimalidade de m.
l. Use o algoritmo de Euclides.
10. Comece analisando a primeira parte do problema e, para tanto, es-
2. 2m = (n - l)(n + 1); calcule, agora, os possíveis valores de mdc (n -
creva 264 + 1 = (264 - 1) + 2 e fatore 264 - l.
1, n + 1) e, em seguida, utilize o item (b) do corolário 1.22.
12. Para os itens de (a) a (e), é suficiente aplicar judiciosamente os itens
3. Mostre que, se existir um natural que é termo de ambas as PA's, então
(a) e (b) do problema anterior. Quanto ao item (f), comece utilizando
existirá uma infinidade de naturais que são termos das mesmas; para
o resultado do item (b).
a caracterização da existência de um natural que seja termo comum
13. Comece analisando o caso em que x pertence a um intervalo do tipo das PA's, use a proposição 1.25.
(n, n +!),para algum n E Z.
4. Trabalhe novamente o problema 10, página 11.
14. Comece mostrando que, nas condições do enunciado, temos
1> ª 2 :;t -
+ ~+
1 + ab e, daí, que 1 +(a+ b- l)(a + b - ab) > a 2 b2 .
2 2
ª: 5. Comece mostrando que o mdc que se quer calcular divide (n + 1)! -
n! = n · n!.
Conclua, por fim, que a 2:: b =:} b = l.
6. Se d= mdc (a, b), escreva a= du e b = dv, de sorte que mdc (u, v) =
15. Desenvolvendo a igualdade do enunciado, obtemos (x + y)(x + z) =
l. Então ab (a 2 + b2 ) se, e só se, uv (u 2 + v2 ). Nesse caso, como
J J
2xz. Conclua que um dos membros dessa última igualdade é um
u,v uv, temos que u,v (u 2 +v 2 ) e, daí, u v2 e v u 2 . Mas, uma
J J J J

múltiplo de 4, enquanto o outro não o é.


vez que mdc (u, v) = 1 implica mdc (u, v2 ) = mdc (u 2 , v) = 1, tais
16. Aplique os itens (a) e (b) do corolário 1.8. relações de divisibilidade implicam u = v = l. Assim, a = b = d.
Alternativamente, escreva a 2 + b2 = abk, com k E N, e examine O
17. Para o item (b), use o resultado de (a); quanto a (a), escreva n = discriminante da equação de segundo grau x 2 - (bk )x + b2 = O, a qual
7q + r, com r = O, 1, 2, 3, 4, 5 ou 6, e calcule n 3 . tem a raiz natural a.
18. Certamente n > 1 e podemos supor que x::; y < z; se x = y, mostre 7. Analise primeiro o caso mdc(a,b) = l. Nesse caso, mostre que a
que n = 2; se x < y, temos 1 = nY-x(nz-y - 1). diagonal principal não pode tocar um quadradinho somente em um
19. Para k 2:: 5, mostre que o último algarismo de k! é O; use, em seguida, vértice e que ela intersecta a coluna de quadradinhos entre x = j e
x = j + 1 em exatamente l~(j + l)J + 1 - l~jJ quadradinhos.
o resultado do exemplo 1.10.
~-

212 Sugestões e Soluções 213

8. Sendo d o mdc procurado, segue do teorema das colunas do triângulo 14. Escreva aj + b = ml ª!;b J + Tj, com O~ Tj < m; em seguida, use a
de Pascal - cf. proposição 6.5 do volume 1 - que d divide a soma condição mdc (a, m) = 1 para concluir quero, r1, ... , Tm-1 são dois
(n+k+l)
k+l
dos números dados . portanto ' d divide (n+k+l)
k+l
- (n+k)
k
-- a dois distintos.
(~!~). Conclua, a partir daí e com o auxílio da relação de Stiefel (cf.
15. Escreva r = gu, n = gv, com mdc(u,v) = l; em seguida, faça
proposição 6.3 do volume 1), que d divide (~!D, (~!~), ... (~!~). Por { ~i} = ~i - l ~i J e use o resultado do problema anterior.
fim, argumente indutivamente.
16. Escreva mdc (m, n) = mx + ny, com x, y E Z.
9. Se d é o mdc dos números binomais dados, então d é uma potência
de 2, uma vez que d divide sua soma, a qual vale 22n- 1 . Escreva 17. Utilize o resultado do item (a) do problema 5.11 do volume 1.
n = 2kq, com q EN ímpar, e mostre que
18. Prove o item (a) por indução sobre n e o item (b) por indução sobre k.
Quanto ao item (c), para o subitem (i) use (b) para mostrar, também
( 2n ) _ 2k+l (2n - l)q (2n - 2).
2t - 1 - · (2t - 1)(2n - 2t + 1) 2t - 2 ' por indução, que Fn I Fnqi para o subitem (ii), use (i) e (a); para
(iii), faça uma prova por indução sobre q 2 O, utilizando (b) e (ii) no
conclua, a partir daí, que 2k+1 divide todos os binomiais dados.
passo de indução. Por fim, para o item (d), adapte o algoritmo de
10. Se d= mdc (n 2 + k, (n+ 1)2 +k), então d 1 [((n+ 1)2 + kJ- (n 2 + k)]; Euclides à situação em questão, com o auxílio do item (c).
logo, d 1 [(2n + 1) 2 - 4(n2 + k)], e é fácil concluir, a partir daí, que
19. Adapte, ao presente caso, os passos e sugestões do problema anterior.
d 1 ( 4k + 1), de sorte que d ~ 4k + 1. Exiba, agora, um valor de n
para o qual o mdc correspondente seja igual a 4k + 1. 20. Para o item (a), sejam u e v naturais tais que au - bv = 1. O fato de
ser n > ab nos dá nau - nbv = n > ab e, daí, nbu - :_v > l; portanto,
11. Comece escrevendo a = du e b = dv, onde d = mdc (a, b) e, por
conseguinte, mdc(u,v) = l; conclua, em seguida, que u,v I e e, daí,
existe um inteiro t tal que:_v
<t < 1u; faça x = nu-bt e y = at-nv.
Agora, obtenha (b) argumentando por contradição e reduza (c) a (a)
que uv I e.
e (d) a (b). Quanto a (e), faça S = ab - a - b e mostre que, para
12. Faça x 2 + y 2 - x = 2xym, com m EN; em seguida, considerando tal O ~ m ~ S, exatamente um dos números m e S - m pode ser escrito
igualdade como uma equação de segundo grau em x, mostre que o fato como pede o enunciado.
de x ser inteiro garante que o discriminante~= (2ym + 1) 2 - 4y2 =
21. (a) Sejam x, y, z E Z+ tais que 2abc- ab-bc- ca = xbc+ yac+ zab.
(2ym+l-2y)(2ym+1+2y) deve ser um quadrado perfeito. Por fim,
Então
mostre que mdc (2ym + 1 - 2y, 2ym + 1 + 2y) = 1 e use o resultado
2abc = (x + l)bc + (y + l)ac + (z + l)ab
do corolário 1.22.
e, daí, a 1 (x + l)bc. Como mdc (a, bc) = 1, segue que a 1 (x + 1) e,
13. Mostre inicialmente, com o auxílio do princípio da casa dos pombos portanto, x + 1 2 a. Analogamente, y + 1 2 b e z + 1 2 e. Mas aí,
- cf. seção 4.1 do volume 4 - que ao menos um dos cinco naturais
ímpares em nosso conjunto não é múltiplo de 3, 5 ou 7; em seguida, (x + l)bc + (y + l)ac + (z + l)ab 2 a· bc + b · ac +e· ab = 3abc,
prove que tal número é primo com os outros nove. o que é uma contradição. (b) Se n > 2abc - ab - bc - ca, então
mdc(y, z) = 1. n > a·bc-a-bc e o problema anterior garante (a e bc são primos entre
214 Sugestões e Soluções 215

si) a existência de inteiros x, t E Z+ tais que n = xbc+ta. Sem perda 2. Comece escrevend o n - új=l 1 - 2 °"'k
m _ °"'2k+l 1 1 _ °"'2k+l 1.
új=l 2j - új=k+I 1, em se-
de generalidade, podemos supor que O ::; x < a. De fato, se x ~ a guida, agrupe as parcelas dessa última soma de duas em duas, para
escreva x = aq + x', com O ::; x' < a, obtendo n = x' bc + (t + qbc )a. escrevê-la como uma soma de frações com numeradores todos iguais
(c) Sendo x::; a - 1, temos a 3k + 2 = p, e use o fato de p ser primo.

ta= n - xbc ~ (2abc - ab- bc - ca) - (a - l)bc = abc - ab - ac, 3. Podemos supor x f:- y. Se x < y, prove que x < p < y e, daí, que
mdc (p, x) = 1. Escreva 2xy = p( x + y) para concluir que p I y. Faça
donde t > bc - b - e. Mas, como b e e são primos entre si, novamente
y = pz para obter z = 2 xx__p, concluindo, daí, que (2x - p) 1 2x e,
pelo resultado do problema anterior existem y, z E Z+ tais que t =
então, que (2x - p) 1 p.
bz + cy. (d) Nas notações dos itens anteriores, temos
4. Comece considerando separadamente os casos n = 6k, 6k+ 1, ... , 6k+
n = xbc + ta = xbc + (bz + cy )a = xbc + yac + zab.
5. Para n = 6k, por exemplo, use o fato de que k, 2k + 1, 3k + 1 e
6k + 1 são dois a dois primos entre si para concluir que k = 1.
22. Adapte os passos descritos no problema anterior e faça indução sobre
n~ 2. 5. Comece utilizando a condição do enunciado para mostrar que n I an
23. Para a primeira parte do item (a), use indução. · Para o item (b), e, daí, que se pé um primo que não divide n, então p f ªn·
mostre que, se O < a1 < a2 < a3 < · · · < ªn-1 são os restos obtidos
6. Para o item (a), use o teorema fundamental da aritmética (para cuja
na execução do algoritmo de Euclides para o cálculo do mdc de a =
prova não utilizamos a infinitude dos primos). Para a primeira parte
an+I e b = an (sendo ªn-1 o primeiro resto, etc, a1 o último), então
do item (b) use, além do resultado do item (a), o princípio funda-
ªi ~ Fj-l, para 1::; j::; n. Por fim, mostre que (c) segue de (a) e de
mental da contagem. Por fim, mostre que o resultado de (b) gera a
(b).
desigualdade n ::; 2k y'n, a qual, por sua vez, gera uma contradição.
24. Sejam k o único inteiro positivo tal que 2k < n < 2k+1 e M
7. Em cada um dos casos em questão, imite a ideia da prova do exemplo
mmc (1, 2, ... , n). Mostre que 2k IM e que
1.39. Para tanto, observe que todo primo p f:- 3 é da forma 3k ± 1 e
n 1 1 n M todo primo p f:- 2, 3 é da forma 6k ± 1.
L-:=MI:-.,
J
j=l J j=l
8. Fatore 22n - 1 e use o resultado do problema 4, página 31.
com ~ ímpar se, e só se, j = 2k.
J 9. Use a identidade de Lagrange - cf. exemplo 6.12 do volume 1 ou
exemplo 2.8 do volume 4 - para escrever (:); em seguida, use o
resultado do exemplo 1.41. Alternativamente, escreva
Seção 1.3 = (p + 1) (p + 2) ... (2p - 1) _
( 2p) _ 2 l
p 1·2 ... (p-1)
1. Use o fato de que p+ q é par e, se p < q, então p < ~ < q, de sorte
que (p + q)/2. e mostre que p divide (p + l)(p + 2) ... (2p - 1) - 1 · 2 ... (p- 1).
r,

216 Sugestões e Soluções 217

10. Para o item (a), mostre que, se n não for uma potência de 2, então é prove esse resultado escrevendo, para n = 2k - 2,
possível fatorar un + 1; argumente analogamente quanto ao item (b). 2k-l_1 k-1

11. Considere separadamente os casos n par e n ímpar; no caso n = 4k,


I(n) L d(2j - 1) + L (d(2i) + d(2i. 3) + ... + d(2k _ 2i))
j=l i=l
calcule mdc (~ -1,n); no caso n = 4k + 2, calcule mdc (~ - 2,n); k-1
2
por fim, no caso n ímpar, calcule mdc (n, nÍl ). ( 2k- 1 - 1) + :E (1 + 3 + s + ... + (2k-i - 1))
. i=l
12. Fatore Cm+j - Ck e, em seguida, aplique o resultado do problema 6, k-1
+ :E 22k-2i-2
2
página 10. ( 2k-1 _ 1)
i=l
1 ... pkªk e' a f at oraçao ( 2k-l - 1)2 + 22k-2 - 1
13. Se n = p a1 - canomca d e n, escreva u = p (31
A '
1 ... pkf3k = (2k - 2)(2k - 1)
e v = PI1 ... P7/, com f3i, "li ~ O, e calcule quantos são os pares or- 3 3 .
denados (f3i, 'Yi) tais que max{f3i, 'Yi} = ªi· Aplique, em seguida, o
corolário 1.47. 18. Veja que 3 1 n. Escreva n = 2ª · 3f3p~1 • • • p~t, onde 3 < p 1 < · · · < Pt
são primos e a ~ O, {3 ~ 1, e note que a relação do enunciado se
14. Escreva ~ = a,
com mdc (e, d) = 1, e deduza que .b = xd eu= xc, reduz à equação 3(a + 1)({3 + l)(k1 + 1) ... (kt + 1) = 2ª3f3p~ 1 ••• p~t,
para algum x E N. Conclua, então, que x = 1 ou cx 2d = dxc; no Conclua, a partir daí, que {3 = 1 ou 2 e que n não possui outros
segundo caso, analise separadamente os subcasos e < 2d, e = 2d e fatores primos; em seguida, analise cada um de tais casos.
e> 2d.
19. Fixado um primo p qualquer, basta mostrar que ep(2m) + ep(2n) ~
15. Se n for ímpar, então não há soluções, uma vez que ambos os membros ep(m) + ep(n) + ep(m + n). Para tanto, use a fórmula de Legendre e
da igualdade do enunciado têm paridades distintas. Se n for par, o item (f) do problema 12, página 11.
então n = 5 + di + dt com d3 e d4 com paridades distintas. Se n for 20. Para o item (a), use a fórmula de Legendre para mostrar que e2(2n) ~
múltiplo de 4, mostre que n = p 2 + 21, onde pé o menor primo ímpar 2e2(n) + 1, para todo n E N - alternativamente, veja o problema
que divide n e, então, que p 1 21; conclua que não há soluções nesse 6.2.18 do volume 1. Para o item (b), refine os cálculos do item (a)
caso. Se n for da forma 4k + 2, mostre que n = 5(p2 + 1), onde pé o para mostrar que e2(2n) ~ 2e2(n) + 2 se, e só se, n não for uma
menor primo ímpar que divide n. Conclua, a partir daí, que p = 5 e, potência de 2.
então, que n = 130.
21. Escreva n = uo + u1 · 2 + u2 · 22 + · · · + Um2m, com Uj E {O, 1} para
16. Sendo n = d13q13 = d14q14 = d15q15, com q13, q14, q15 EN, a condição O::; j ::; m, de sorte que uo + u1 +···+Um= k. Calcule
1 + -q
(b) equivale a -q 1 + -q
1 = 1; comece, então, obtendo todos os
13 14 15
i
u < b < e naturais tais que + } + = 1. i l; J = Uj + Uj+l · 2 +···+Um · 2m-j
17. Examinando os casos 1 ::; n ::; 30, conjecture que os naturais n da e, em seguida, use a fórmula de Legendre para mostrar que e 2(n) =
forma 2k - 2 satisfazem a propriedade do enunciá.do. Em seguida, n-k.
218 Sugestões e Soluções 219

22. Sejam p primo e t E N tais que (a + kb) (b + ka) = pt. Supondo, seja p o maior fator primq de Yi e p = Pr na sequência dos primos;
sem perda de generalidade, que a ::; b, temos a+ kb 2".: b + ka > 1, defina Yi+I = Xti+i, com tj+l > r, tj. Prossiga dessa maneira até
de sorte que existem r, s E Z tais que 1 ::; r ::; s, r + s = t e que j = m, mostrando, em seguida, que o conjunto B assim obtido
a + kb = p8 e b + ka = pr. Conclua que (b + ka) 1 ( a + kb) e, também satisfaz a condição (c).
escrevendo ~t~! = k- (k:.;k~ª,que (b + ka) 1 (k 2 - 1). Considere,
agora, dois casos separadamente: (i) se p > 2, use o fato de que
b + ka é uma potência de p e mdc (k - 1, k + 1) ::; 2 para concluir
Seção 2.1
que b + ka divide k - 1 ou k + 1, de sorte que b + ka ::; k + 1 e,
então, b = a = 1. Conclua que, nesse caso, as soluções são os ternos 1. Se (xo, Yo, zo) é uma solução, então (xoan+I, yoan+I, zoan) também é
(a, b, k) = (1, 1,pr -1), para todos r 2".: 1 e p > 2 primo. (ii) Se p = 2, solução; agora, obtenha uma solução com x = y = k + 1.
comece observando que ao menos um dentre a e b é ímpar e, portanto,
que k também é ímpar. Agora, como (b+ ka) 1 (k 2 -1) e b+ ka = 2r, 2. Use a proposição 2.1 para escrever x + y, y + z ex+ z em termos de
parâmetros u, v e d, como prescrito por aquele resultado.
mostre que (b+ka) l 2(k+l) ou 2(k-1); em particular, b+ka::; 2k+2
e, daí, a = 1 ou 2. Se a = 2, chegue a uma contradi~ão; se a_= 1, 3. Imite a prova da proposição 2.1. Mais precisamente, se x, y, z > O,
temos que (b + k) 1 2(k + 1) ou k - 1, o que suscita a análise de dois · comece observando que z - x e z + x são ambos pares e, se d =
subcasos: (b+k) 1 (2k+2) ou (b+k) 1 (2k-2). No primeiro subcaso, mdc ( z2x, z!x), então a igualdade
mostre que b = k + 2 ou b = 1. Se b = 1, conclua que k = 2r - 1;
se b = k + 2, escrevas= 2u para obter (a, b, k) = (1, 2u + 1, 2u - 1),
para todo u EN. No segundo subcaso, mostre que b = k - 2 e, em
seguida, faça s = 2u para obter (a, b, k) = (1, 2u - 1, 2u + 1), para garante a existência deu, v E Z tais que z:;f = 2v 2 , z;;t = u 2 ; agora,
todo u EN. mostre que mdc (u, 2v) = 1.

23. Para o item (a), use o resultado do problema 4, página 31, juntamente 4. Use o método da descida de Fermat em cada um dos itens acima,
com o fato de que p(q 2k) = q, para todo k 2".: O. Para o item (b), nos moldes dos exemplos 2.3 e 2.4. Especificamente para o item (d),
fatore q2k - 1 e use a minimalidade de k para concluir que não pode comece mostrando que x e y deixam restos iguais quando divididos
ser p(q2k-1 - 1) > p(q2k-1). por 3; em seguida, calculando (3k + r )3, conclua que, se x e y não
forem múltiplos de 3, então x 3 + 5y3 não será múltiplo de 9.
24. Escolha um primo p maior que 1 + 2 + · · · + 1000 e mostre que o con-
junto A = {p, 2p, 3p, ... , 1000p} satisfaz as condições do enunciado. 5. Comece escrevendo x = u-v e y = u+v, com u, v E (Q) e, em seguida,
u =%e v = t com a,b,c E Z e mdc(b,c) = 1. Em seguida, para o
25. Se t é o produto dos primos menores ou iguais a m, defina Y1 = Xti, item (a) utilize o resultado do exemplo 2.3; para o item (b), adapte a
onde Pti é um primo maior que t. Sendo p o maior divisor primo de Yl demonstração da proposição 2.1 para resolver a equação 3a2+b 2 = c2.
e p = Pr na sequência dos primos, defina Y2 = Xt 2 , com t2 > r, t1, e se
convença de que mdc (Y1, Y2) = 1. Escolhidos elementos YI = Xt 1 < 6. Se z for ímpar, use o resultado do item (c) do corolário 1.8 para
. · · < Yi = Xti de A satisfazendo a condição do item (b), com j < m, mostrar que não há soluções. Se z for par, use o item (b) do mesmo
220 Sugestões e Soluções 221

resultado para concluir que w, x e y também são pares e, em seguida, 5. Se x 2 - dy 2 =me a 2 - db 2 = 1 com a b EN então
aplique o método da descida de Fermat. ' ' '
(a - bvd)(xo + Yovd) ·(a+ bvd)(xo + Yovd) = m
7. Multiplique a igualdade do enunciado por 4, complete quadrados e
reduza 2x + 3, 2y + 3 e 2z + 3 a um mesmo denominador comum para ou, ainda,
reduzir o problema em questão ao problema anterior.
[(axo + bdyo) - (ayo + bxo)vdJ[(axo + bdyo) + (ayo + bx0 )vd] = m,
8. Se a e b são os catetos e e é a hipotenusa de um desses triângulos, então
o teorema de Pitágoras nos dá a 2+b2 = c2; por outro lado, o resultado de sorte que x1 = axo + bdyo e YI = ayo + bxo também resolvem a
da proposição 3.37 do volume 2 fornece a relação a+b-c = 2r. Use as equação do enunciado. Observe, agora, que
fórmulas da proposição 2.1 na primeira igualdade, substituindo-as, em
x~ + dy~ = (a2 + db 2 )(x5 + dy5) + 4abxoyod > x5 + dy5,
seguida, na segunda igualdade para obter v( u - v) = r; a partir daí,
mostre que mdc (v, u - v) = 1 e conclua que há 2k escolhas possíveis de sorte que (xi, Y1) =/= (xo, Yo).
para v, após o quê u estará completamente determinado. Para ver
6. Completando quadrados, conclua que a equação dada é equivalente
que os triângulos assim obtidos são dois a dois não congruentes,_veja
à equação (2x + y) 2 - 5y2 = 4. Use, agora, o resultado do problema
que
anterior, observando que x = y = 1 é uma solução da equação origi-
r)2 r2
c2 = u 2 + v2 = ( v + - + v 2 = 2v2 + - 2 + 2r, nal.
V v
com 2v1 + 2Vl
2 r2
= 2v22 + ~
r2
V2
se, e so, se, v1 = v2. 7. Imitando o argumento do problema 5, mostre que, se u, v E N são
tais que u 2 - l:!i.v 2 = 1, então a := uxo - l:!i.vyo e /3 = uyo - vxo é
9. Comece fazendo A1A2 igual a um diâmetro do círculo; em seguida,
solução inteira de x 2 - l:!i.y 2 = 4an. Agora, observe que
use (2.1) para escolher pontos Â3, ... , An tais que A1Ai e A2Ai
sejam racionais, para 3 ::; i ::; n; por fim, use o teorema de Ptolo- ax 2 + bxy + cy 2 = n =} (2ax + by) 2 - l:!i.y 2 = 4an,
meu (teorema 4.17 do volume 2) para mostrar que AiAj também é
racional. de sorte que, para gerar uma solução inteira da equação ax 2 + bxy +
cy 2 = n, é suficiente mostrar ser possível resolver, em .Z, o sistema
linear
2ax+by=a.
Seção 2.2 {
y =/3
1. Substitua d = 4k + 3, m = 4l + 3 e use o corolário 1.8. Tal tarefa, por sua vez, equivale a mostrar que 2a 1 ( a - d/3), o que
pode ser feito escrevendo
3. Imite a discussão do exemplo 2.6, observando que x = y = 1 é uma
solução da equação. a - d/3 = (uxo - l:!i.vyo) - b(uyo - vxo)

4. Multiplique a igualdade n 2 +(n+ 1) 2 = m 2 por 2, complete quadrados


= u(xo - byo) + bv(xo - byo) + 4acvyo,
e use o resultado do problema anterior. expressão que é igual a uma soma de múltiplos de 2a.
222 Sugestões e Soluções 223

Capítulo 3 11. Conclua, a partir da proposição 3.4 e do problema 1, que ambos os


membros da igualdade acima podem ser vistos como funções aritmé-
2. Prove primeiro que ITo<dln d = ITo<dln l Em seguida, denote tal ticas multiplicativas, F e G digamos. Em seguida, se p é primo e
t
produto por P e use essa igualdade para mostrar que P 2 = nd(n). a E N, mostre que F(pª) = G(pª).
I'
i
12. Aplique contagem dupla - cf. seção 2.2 do volume 4 -, mostrando
3. Para O < d I n, temos d + ~ ~ 2..jn. Agora, some tais desigualdades
inicialmente que, para 1 :S j :S n, o primeiro membro conta f(j)
sobre todos os divisores positivos de n.
exatamente L!!<J J vezes.
4. Para a segunda equação, mostre que um par (x, y) é solução se e
2 ' 13. Comece aplicando o resultado do problema anterior; em seguida, apli-
só se, x = n - n~y, de modo que n + y deve ser um divisor de n2
que a proposição 3.4 para mostrar que F(m) = 1 sem for um qua-
maior que n; utilize agora o exemplo 1.46 e (3.3) para concluir que
drado perfeito e F(m) =Ocaso contrário.
que há exatamente !(d(n2) - 1) possibilidades para y e, portanto,
para (x, y). Quanto à primeira equação, imite a sugestão apresentada 14. Se f(j) = 1 para j E N e F(n) = LO<dln f(d), então F(n) = d(n),
para a segunda equação para concluir que há d(n 2 ) soluções. Por fim, para n E N. Agora, aplique o resultado do problema 12.
some os números de soluções das duas equações para obter a equação
3d(n2) = 157, a qual não possui soluções. 15. Use a fórmula de inversão de Mobius e o resultado do problema 1.

5. Para o item (a), use o resultado do lema 3.3. Para o item (b), se 17. Use o resultado da proposição 3.4.
mdc (m, n) = 1, use o resultado do item (a) para calcular ID1(mn)I-
18. Comece provando que, fixado um divisor positivo d de m, há exata-
ID3(mn)I em função de ID1(m)I - ID3(m)I e ID1(n)I - ID3(n)I. Por
mente '![; · <p( d) pares ordenados (d, n) como pede o enunciado. Em
fim, aplique indução.
seguida, o resultado do corolário 3.13.
6. Para o item (a), multiplique ambos os membros da igualdade desejada
20. Para (a), use o item (a) do problema 19. Para (b), escreva Sm(n) =
por n. Para o item (b), calcule diretamente a soma dos divisores
L7=1 (n-air e desenvolva o binômio (n-ai)m. Para a primeira parte
positivos de 2P- 1(2P - 1).
de (c), use (b); quanto à segunda parte, considere separadamente os
7. A segunda parte de (b) utiliza o item (b) do problema 10, página 48. casos n par e n ímpar e aplique a conclusão do item (a) no caso em
que n é par.
8. Mostre que s(ab) ~ s(a)b, para todos a, b EN. Para tanto, exiba, em
21. Para o item (a), escolha d de tal forma que mdc (m, n) = ~ e a tal
função dos divisores positivos de a, um conjunto de divisores positivos
que m = ~ · a; conclua, então, que mdc (a, d) = 1. Para o item
de ab cuja soma seja igual a s(a)b.
(b), use o resultado de (a) para mostrar que Lo dl (!!)k Sk(d) =
k k k . . <nd
9. Aplique a proposição 3.4. 1 + 2 + · · · + n . Para o item (c), aplique a fórmula de inversão de
Mobius ao resultado do item (b). Por fim, para os itens (d) e (e), use
10. Aplique a proposição 3.4. a fórmula do item (c).
224 Sugestões e Soluções 225

Seção 4.1 maior primo menor ou igual a p - 1, conclua que m 2: 2q. Por fim, use
a observação que antecede o enunciado deste problema para concluir
1. Use o teorema do número primo. que q :S 5 e, portanto, que p - 1 :S 6. As soluções são m = n = 2 ou
2. Use o lema 4.3 em conjunção com (4.4). m = 10, n = 7.

3. Use o lema 4.3 e o teorema 4.7. 6. Sejam n > 1 um natural satisfazendo as condições do enunciado e
p < ,/ri um número primo. Então p I n, pois, do contrário, teríamos
4. Para a primeira parte do item (a), use o fato de que, se n < p :S p 2 < n e mdc (n,p 2) = 1. Assim, n é divisível por P1, ... ,Pk, onde
2n, com p primo, então mdc (n!,p) = l; para a segunda parte, use Pl < · · · < Pk são os primos menores que ,ln. Agora, seja l EN tal
a fórmula do desenvolvimento binomial. Para o item (b) use (a), que 21 :S ,ln < 21+1 . Como, para todo inteiro k > 1, há pelo menos
juntamente com o fato de que o número de primos p tais que n < p :S dois primos entre 2k e 2k+1, temos que
2n é exatamente 1r(2n) - 1r(n). Para a segunda parte do item (d),
use o fato de que~ 2: x 213 ex+ 2 :S 5,f para x 2: 8. Para o item (d), Pl ... Pk 2: 2(22 ... 21-1 )2 = 212.
verifique os casos 2 :S n :S 8 diretamente; em seguida, prove os demais
casos por indução sobre n 2: 8. Por fim, para o item (e)i use os itens Assim,
212 :S P1 · · · Pk :S n < 221+2 ,
(c) e (d), juntamente com a desigualdade (óbvia) 1r( x) :S 2 l ~ J + 2.
de forma que l :S 2. Há, pois, três casos a considerar:

Seção 4.2 (i) l = O: nesse caso, temos 1 < n < 4, e é imediato verificar que
n = 2, 3 são soluções.
1. Pelo teorema de Chebyschev, há pelo menos um primo entre Pn e 2pn,

2. Se pé o maior primo que é menor ou igual a n, utilize o teorema de (ii) l = 1: temos que 4 :S n < 16 e 2 n, de forma que n E
1

Chebyschev para mostrar que 2p > n. { 4, 6, 8, 10, 12, 14}. Uma rápida inspeção garante que as soluções são
n = 4,6,8 ou 12.
3. Se n = 2k, escreva 1!2! ... n! = 2kk!(3!5! ... , (2k-1)!) 2 e, em seguida,
aplique o resultado do problema anterior. O caso n = 2k + 1 pode
(iii) l = 2: temos, nesse caso, que 16 :S n < 64 e 2, 3 1 n. Portanto,
ser tratado de modo análogo.
n E {18, 24, 30, 36, 42, 48, 54, 60}. Novamente uma inspeção simples
4. Se p e q denotam os maiores primos respectivamente menores ou iguais dessas possibilidades fornece as soluções: n = 18, 24 ou 30.
a m e a n, use o resultado do problema 2 para comparar as maiores
potências de p e de q em ambos os membros da equação dada.

5. Comece utilizando o resultado do problema 2 para mostrar que n = p, Seção 5.1


um número primo, para o quê siga uma argumentação análoga à
delineada na sugestão ao problema anterior. Em seguida, se q é o 1. Argumente por indução sobre m.
226 Sugestões e Soluções 227

3. Use congruências para calcular o resto da divisão do número dado 15. Use módulo 3 para mostrar que zé par, digamos z = 2t; em seguida,
por 4. Em seguida, aplique o resultado da proposição 5.9. fatore 52t - 4Y e argumente como no exemplo 5.11.

4. Para k E Z+, mostre que 74 k = 1 (mod 10), 74 k+l = 7 (mod 10), 16. Se y = 1, entãop = 2 ex= l; se y > 1, escrevapx = (y+l)(y 2-y+l)
74k+2 = -1 (mod 10) e 74 k+ 3 = 3 (mod 10); em seguida, calcule o e mostre que, se d= mdc (y + 1, y2 - y + 1), então d= 1 ou 3. Por
resto da divisão de 310 por 4. fim, considere separadamente os casos d = 1 e d = 3.
5. Se todos os divisores primos de n fossem congruentes a 1, módulo 17. Escreva a = 2ka e b = 21/3, com k, l ?: 1 e a, f3 ímpares. Mostre, a
4, use as propriedades elementares de congruências para concluir que partir daí, que kb = la e ab + /3ª = 2c-kb, e use módulo 4 para concluir
deveríamos ter n = 1 (mod 4). que a = /3 = 1 e .; = i·
Por fim, analisando a função f(x) = ,fx,
mostre que, se k, l ?: 4, então k = l.
6. A primeira igualdade nos dá 232 + 228 · 54 = O(mod641); a segunda,
228 . 54 = (-1) 4 (mod641). 18. Inicialmente, mostre que basta considerar o caso b = an. Para tal
7. Suponha que a representação decimal de n tenha k + 1 algarismos e caso, faça indução sobre n ?: O, utilizando congruência módulo a no
seja m o natural formado pelos k primeiros algarismos qe n. Mostre passo de indução.
que a condição do enunciado equivale à igualdade 6 · lOk + m = 19. Comece analisando a equação, módulo 3, para concluir que b é par.
4(10m + 6) ou, ainda, 3 · lOk = 13m + 12; a partir daí, conclua que Em seguida, faça b = 2c e conclua que 2c+l = 15ª + 1 ou 2c+1 =
basta encontrar os naturais k tais que lOk = 4(mod 13).
3ª + 5ª. Faça d = e + 1 e, no primeiro caso, use módulo 3 para
8. Comece observando que 2n + 3n = O(mod 7) se, e só se, 5n + 9n = concluir que d é par e, em seguida, que 2d/ 2 - 1 = 3ª e 2d/ 2 + 1 = 5ª.
O(mod 7), o que, por sua vez, equivale a (-lr + 2n = O(mod 7). Em No segundo caso, use módulo 4 para concluir que a é ímpar, de sorte
seguida, calcule as possíveis congruências de 2n, módulo 7. que, se e - 2 > O, temos

10. Use congruências para mostrar que os restos das divisões dos números 2c-2 = 3a-l - 3a-2. 5 + ... + 5a-l = a (mod2);
de Fibonacci por 5 formam uma sequência periódica.
conclua, pois, que e - 2 = O.
11. Use congruência módulo 3 para mostrar que p ou q deve ser múltiplo
de 3 e, daí, igual a 3; em seguida, supondo q = 3, escrevap2 +9p+9 = 20. Utilizando o fato de que 1992 = 24 · 83, mostre que o resto da divisão
n 2 e resolva tal igualdade para p. de 101992 por 1083 + 7 é 724 . Em seguida, se q E N é tal que 101992 =
(1083 + 7)q + 724 , use congruências módulo 10 para calcular o último
Aplique o item (e) da Proposição 5.9.
algarismo de q.
13. Use módulo 3 para concluir quem é par. Em seguida, fatore k 2 - 2m
e argumente como no exemplo 5.11.

14. Se n?: 5, conclua que mP = 3 (mod 10) e, daí, que p > 2. Em seguida, Seção 5.2
use módulo 3 para concluir que 3 1 m e, portanto, que 27 1 mP. Por
fim, use módulo 27 para concluir que não há soluções quando n?: 9. 1. Use o pequeno teorema de Fermat com p = 7.
228 Sugestões e Soluções 229

2. Considere, inicialmente, os casos p = 2, 3, 5. Se pi- 2, 3, 5, escreva a conclua, a partir daí, que devemos ter 5m-n 1 (mod 2k). Use,
diferença acima como então, o teorema de Euler.

(a· 108P - 108 ) + 2(a · 107P - 107 ) + · · · + 9(a - 1), 9. Com a ajuda do teorema de Euler, prove primeiro que, se l for ímpar,
então mdc (2'P(l)m - 3, l) = 1, para todo m EN; em seguida, escolha
onde a= 11 ... l; em seguida, aplique o pequeno teorema de Fermat
'-v-' kI, ... , kn indutivamente.
p
para concluir que a= 1 (modp) e para analisar o resto da divisão de 10. Use a fórmula de inversão de Mõbius para obter an = I:o<dln µ(d)2~.
cada parcela da soma acima por p. Em seguida, use o teorema de Euler para mostrar que, se p é primo e
3. Mostre que, módulo 2n+ 1, o j-ésimo termo da sequência vai para a
pª é a maior potência de p que divide n, então pª 1 ªn· Para tal fim,
escreva n = pªk, com mdc (k,p) = 1, e, a partir daí,
posição 2k j após k 2:: 1 operações; em seguida, use o pequeno teorema,
de Fermat. an = L µ(d)2p:k = L µ(d)2p:k + L µ(pd)2p:dk
O<dJpªk O<djk O<djk
4. Inicialmente, use módulo 11 para concluir que não há soluções tais
que x = O(mod 11); se x -;;j. O(mod 11), use o pequeno teorema de L µ(d)2I!:f - L µ(d)2Pª:lk
Fermat para concluir que 11 divide x 5 + 1 ou x 5 - 1 e, daí, que O<djk O<dlk
y 2 = -5 ou -3 (mod 11). Por fim, mostre que tais congruências não
têm soluções.
I: µ(d) { ( 2p°':lk) p - 2p°':lk}
O<djk

5. Podemos supor p > 2; nesse caso, se n = pq + r, o pequeno teorema


de Fermat garante que basta encontrarmos um inteiro O :'.S; r < p tal
= L µ(d)2Pa-I.~ ( 2po:-1~-l)k - 1)
O<djk
que 2q+r = r (modp) para infinitos naturais q. Escolha r = 1 e, a
partir daí, infinitos q's.
L µ(d)2Pa-1.~ ( 2rp(p°')·~ - 1) •
O<djk
6. Suponha que exista um inteiro n > 1 tal que n 1 (3n - 2n). Se p
11. Para o item (a), argumente por contradição. Mais precisamente, se
é o menor fator primo de n, com n = mp, use o pequeno teorema
a = (PI - 1) ... (Pk - 1), com k 2:: 1 mínimo como em (a), use que
· de Fermat para concluir que 3m = 2m (mod p). Em seguida, use a
mdc (2ª/ 2 - 1, 2ª/ 2 + 1) = 1 para concluir que existem l < k primos
minimalidade de p para concluir que mdc (m, p - 1) = 1. Por fim,
.
d ent re PI, ... ,Pk, d 1gamos PI, ... ,Pl, t ais
· que 2a/2 - 1 -_ PIa1 .. . p a1
1 ,

escreva mx = (p- l)y + 1, com x, y EN, e use novamente o pequeno
em seguida, se b = (pI -1) ... (pz-1), use o fato de (2b-1) 1 (2ª1 2 -1)
teorema de Fermat para chegar a uma contradição.
para contradizer a minimalidade de k.
7. Para o item (b), comece usando o fato de que mdc (p, q-1) = 1 para
12. Para o item (b), use o pequeno teorema de Fermat. Para (f), se q f x,
escrever px = (q - 1)y + 1, com x, y E N. Em seguida, use o pequeno
mostre que x(q-I)/ 2 = ±1 (mod q), e analogamente para y(q-I)/ 2 e
teorema de Fermat.
z(q-I)/ 2 ; em seguida, use a igualdade x(q-I)f 2 +y(q-I)f 2 +z(q-I)/2 = O,
8. Se k é o número de algarismos de 5n, mostre que a condição do juntamente com q > 5, para chegar a uma contradição. Por fim, para
enunciado equivale a 5m - 5n = O(mod lOk); prove que n 2:: k e (g) ii., observe que, se q I a e mdc (a, d) = 1, então q f d.
230 Sugestões e Soluções 231

13. Para o item (e), você precisará utilizar o resultado do exemplo 3.14. Para a recíproca, utilize novamente o teorema de Bézout, juntamente
Para o item (h), mostre primeiro que qn = 2n- 1q1 + (2n-l - 1), para com 8.1).
todo inteiro n 2: 1; em seguida, use o pequeno teorema de Fermat
3. Se x denota o número de soldados, mostre que x satisfaz um sistema
para mostrar que, se q1 for um primo ímpar, então é possível escolher
de congruências lineares com duas equações. Em seguida, utilize a
n 2: 2 tal que q1 1 qn.
prova do teorema 5.27 para mostrar que x = 7·12·12+5·1·13 (mod 12·
13) e, portanto, que x = 132q + 17, para algum q E Z. Por fim, utilize
a condição 600 < x < 700 para concluir que x = 677.
Seção 5.3 4. Para o item (a), comece observando que, se u, v for uma solução da
1. Provemos os dois itens simultaneamente. Se x é um inteiro tal que equação Diofantina linear em questão e x = m 1u + a 1 = m 2v + a 2,
ax = b (mod n), então temos ax = nq+b, para algum q E Z. Portanto, então x resolve (5.7) para k = 2.
b = xa + (-q)n, uma combinação linear de a e n, e segue do teorema
5. Use o teorema de Euler 5.19.
de Bézout que mdc (a, n) b. Suponha, pois, que mdc (a, n) = d e
1

que d I b. Então 6. Se p e q são primos distintos, segue do exemplo 1.28 que 2P - 1 e


2q - 1 são primos entre si. De posse dessa observação, mostre que
ax a = db
= b(modn) {::} dx n)
( modd .
um inteiro x satisfaz as condições do enunciado se, e só se, resolver
um sistema de congruências lineares apropriado. Por fim, aplique o
Mas, como mdc (~, ~) = 1, a existência de solução segue do corolário
teorema chinês dos restos.
5.25. Para o que falta, observe que
7. Adapte, ao presente caso, a ideia da prova do exemplo 5.28.
ax===axo(modn){=}x=xo (mod~) {=}x=xo+~t, ::ltEZ.
8. Para a implicação~ do item (a), seja ai E Z uma solução de f(x) =
Por outro lado, é imediato que O(modpfi). Se x for uma solução do sistema de congruências lineares

!
n n
xo + dt1 =xo + dt2 (mod n) {::} t1 =t2 (mod d), x = a1 (modpf 1)

de maneira que há tantas soluções para a equação original incongruen- x = a2 (modp~ 2)

tes, módulo n, quantos forem os valores de t incongruentes, módulo


X= ak (modprk)
d, i.e., a equação original tem exatamente d = mdc (a, n) soluções
incongruentes módulo n. - cuja existência é garantida pelo teorema chinês dos restos-, use o
item (c) da proposição 5.6 para concluir que f(x) = O(modpfi), para
2. Se a congruência em questão possui solução, utilize o teorema de Bé-
1 :S: i :S: k e, daí, que f(x) = O(mod m). Para o item (b), seja S(t)
zout, juntamente com o item (d) da proposição 5.6, para mostrar que
um conjunto de N(t) soluções duas a duas incongruentes, módulo t,
o mdc entre mdc (a1, a2, ... , ak) endivide b. Em seguida, observe
para a congruência f (x) = O(mod t). Use o resultado do item (a) para
que
garantir a existência de uma bijeção f: S(m)----+ S(pf 1 ) X ... X S(prk);
mdc ( mdc (a1, a2, ... , ak), n) = mdc (a1, a2, ... , ak, n). (8.1) em seguida, aplique o princípio fundamental da contagem.
232 Sugestões e Soluções 233

9. Se a é o primeiro termo e r a razão da PA, as hipóteses do problema temos e2(n) :S: n - 1 e, daí, 2e 2 (n) 1 S. Seja, agora, 2 < p :S: n primo
equivalem à existência de x, y E N, tais que x 2 = a (mod r) e y 3 = e l o expoente da maior potência de p que divide S. Use o fato de
a (mod r); da mesma forma a tese do problema equivale à existência 2n - 2t = O(modpÍ) se, e só se, ordpJ (2) 1 (n - t), juntamente com a
de z E N, tal que z 6 = a (mod r). Faça indução completa sobre ideia da prova da fórmula de Legendre, para concluir que
r, sendo o caso r = 1 trivial; para o passo de indução, considere
separadamente os casos r = pª, para algum primo p e algum a E N,
e r = st, com s, t > 1 primos entre si. No primeiro caso, se x e y são
como no início e y' é o inverso de y, módulo pª, mostre que z = xy'
satisfaz o problema. No segundo caso, sejam u e v termos das PA's 6. Mostre que ki+I = 2ki (mod2n + 1). Em seguida, para o item (a),
(a + ks) k?.O e (a + kt) k?.O, respectivamente, tais que u 6 = a (mod s) e conclua que J (n) = O se, e só se, o conjunto {1, 2, 22, 23 , ... } contiver
v6 = a (mod t); use o teorema chinês dos restos para encontrar z E N, um SCI módulo 2n + 1. Para o item (b), mostre que
tal que z 6 = a (mod st).
J (1997) = 2 · 1997 - ord2.1997+1 (2)
e calcule ord2.1997+1 (2) = 8 · 23 = 184 utilizando as igualdades 2 ·
Seção 7.1 1997 + 1 = 5 · 17 · 47, ord17(2) = 8 e ord47(2) = 23.

2. Por contradição, se 23 n =
-1 (mod 17), então 22·3n =
1 (mod 17); use
a congruência 2 = 1 (mod 17), para concluir que ord 17 (2) = 1 ou 2,
16
o que é uma contradição.

3. Sem= ordn(a), então m / 2k e, pelo teorema de Euler, m divide


cp(n). Portanto, m / mdc (2k, cp(n)) = 2d, onde d= mdc (k, cp(n)/2)
(recorde que n > 2 ::::} cp(n) par - cf. problema 19, página 88). Se
m / d, então m / k, o que contradiz a congruência do enunciado. Logo,
m = 2d', para algum divisor d' de d e, daí, m é par.

4. Se n / (2n - 1), então n é ímpar; se n > 1 e k = ordn(2), então


1 < k / n, cp( n); escolha, de início, o menor natural n > 1 tal que
n 1 (2n - 1) e chegue a uma contradição mostrando que k 1 (2k - 1),
com 1 < k < n.

5. Denote por S o produto do enunciado. Pela fórmula de Legendre


(1.9), a maior potência de 2 que divide n! tem expoente
+oo +oo
e2 (n) = L
j=l
l; J < L ;
j=l
= n,
1---
i
Sugestões e Soluções 235
234

Seção 7.2 6. Para (a) =} (b), escreva n = Pl .. ·Pk, com p 1 < · · · < Pk números
primos tais que (Pi - 1) 1 ( n - 1); em seguida, use o pequeno teorema
1. Mostre que 214 = -1 (mod 29) e, a partir daí, conclua que ord29(2) = de Fermat para concluir que an = a (modpi), para todo 1 ::::; i ::::; k.
28. Para (b) =} (a), tome um divisor primo p de n e faça o inteiro a
igual a uma RP módulo p 2 para obter uma contradição se p 2 1 n; em
2. Considere separadamente os casos n = 2, 4, pk e 2pk, com p primo
seguida, faça o inteiro a igual a uma raiz primitiva módulo p, para
ímpar e k EN. concluir que (p ~ 1) 1 (n - 1).
3. Se a é uma raiz primitiva módulo n, observe que os números ai são
7. Para o item (a), observe que
congruentes, em alguma ordem, aos números a, a 2 , ... , a'P(n), de sorte
pq-1 p-1 pq-1 p-1 p-1
que
a1a2 ... ª<p(n) =a
<p(n)(<p(n)+l)
2 (mod n).
I: I:(pj + zr- 1 = I: z:=zn-l =pq. z:= zn-l =o(modp).
j=O l=O j=O l=O l=O
<p(n)(<p(n)+l)
Se n = 2 ou 4, mostre diretamente que a 2 = -1 (mod n); Para o item (b), use (a) para concluir que, se n = p 2 q, com p primo e q
se n = pk ou 2pk, onde k E N e p é um primo ímpar, use o fato de
<p(n) natural, então Sn = 1 (modp). O item i. é um cálculo análogo àquele
que ordn(a) = r.p(n) para concluir que a-2- cuidàdo: = -1 (mod n) - indicado na sugestão de (a). O item ii. segue imediatamente de i.,
n I <p(n) )( <p(n) ) - lº
( a-2- + 1 a-2- - 1 nao necessariamente 1mp 1ca

iret ament e
• • ,
juntamente com o fato de que, módulo Pi, temos { a, a 2 , •.. , aPi-1} =
<p(n) <p(n)
que n 1 (a_2_ + 1) ou n 1 (a_2_ - 1). {1, 2, ... ,Pi - 1}. Para iii., se n I sn, então Sn = O(modPi)- Mas,
4. Se a3pq-l =1 (mod 3pq), então a 3pq-l =1 (mod p); em particular,
como Pi 1 (aPi(n-l) - an- 1), a fórmula em ii. garante que a única
maneira de Sn = O(modpi) é que Pi 1 (an-l - 1). Então, Pi - 1 =
sendo a uma RP módulo p, devemos ter (p - 1) 1 (3pq - 1) e, analoga-
ordPi(a) 1 (n - 1) e, daí, (Pi - 1) 1 (qi - l)(uma vez que n - 1 =
mente, (q - 1) 1 (3pq - 1). Conclua, a partir daí, que (p - 1) 1 (3q - 1)
(Pi - l)qi + (qi - 1)). Para iv., se n I sn, então Pi I Sn, e segue dei. e
e (q -1) 1 (3p-1), de modo que p = 11 e q = 17, ou vice-versa. Por
iii. (especificamente, de (Pi - 1) 1 ( n - 1)) que, módulo Pi,
fim, mostre que a 3 · 11 · 17 - 1 = 1 (mod 3 · 11 · 17), para todo natural a
primo com 3 · 11 · 17. Para mostrar que p = 11 e q = 17 se p::::; q, p;-1 p;-1
pode-se argumentar da seguinte maneira: como p::::; q, temos O= Sn = 1 + qi L zn-l = 1 + qi L 1 = 1 + qi(Pi - 1);
l=l l=l
3p - 1 3q - 1 2 2
,--<--=3+--<3+-, portanto, qi(Pi - 1) = -1 = Pi - 1 (modpi), de sorte que qi =
q-l - q - l q-l- 4
1 (modpi)· Por fim, em relação ao item v., é suficiente mostrar
de sorte que 3:__::-f = 1, 2 ou 3; analisando cada caso separadamente, que, se Pi(Pi - 1) 1 (qi - 1) para 1 ::::; i ::::; t, então Pi I sn, para
, 3q-1 9q-3 ,
concluímos que 3p - 1 = 2( q - 1) e, d a1, p-l = 2q_ 4 , um numero 1 ::::; i ::::; t; para tanto, basta utilizar a congruência do item i., junta-
maior que 4 e menor que 8; assim, ~~=i = 5, 6 ou 7, o que fornece mente com o fato (facilmente dedutível a partir de nossas hipóteses)
q = 17 como única possibilidade viável. de que (Pi - 1) 1 ( n - 1).

5. Para o item (d), considere separadamente os casos ordp(x) 1, 2, 8. Para o item (a), se mdc (ak + 1, n) > 1, então n 1 (ak + 1) e, daí,
P21 ou p - 1 - aqui usamos que (p - 1)/2 também é primo. a2k = 1 (mod n), o que contradiz o fato de ordn(a) = 2pk. Para
236
Sugestões e Soluções 237

i., segue de akp = -1 (modn) que a 2kp = 1 (modn), de sorte que


da reciprocidade quadrática e a proposição 7.23 para chegar a uma
d / 2kp = n - 1. Por outro lado, se d / 2k, então a2k = 1 (mod n)
·
assim, n t-
/ ( k ) k
1 (a + 1); mas, como mdc (ak + 1, n) = 1, segue daí
e, contradição.

5. Para o item (a), se todo divisor primo de 2b2 + 3 fosse congruente


que n / (a - 1) e, portanto, akP = 1 (mod n), um absurdo. Agora,
se~/ 2k~ e_d f 2k, é imediato que mdc (d,p) > 1, de sorte que p / d; a ±1, módulo 8, teríamos 2b2 + 3 = ±1 (mod 8); mostre que isso é
entao, a ultima parte de i. segue do teorema de Euler. Quanto a ii., impossível. Para o item (b), se (2b 2 + 3) / (a 2 - 2) e p é como em (a),
l'i é evidente que p f (2kp + l); portanto, a fórmula para <p(2kp + 1) então a 2 = 2 (modp), i.e., 2 seria um resíduo quadrático, módulo p;
garante que P / (q - 1), para algum divisor primo q de 2kp + l; logo, use o resultado do exemplo 7.26 para chegar a uma contradição.
basta fazer q - 1 = lp, observando que l > 1 pois, do contrário, tanto
6. Aplique o lema de Gauss, nos moldes do exemplo 7.26, ou a lei da
P quanto q = p + 1 seriam primos ímpares, o que é um absurdo.
reciprocidade quadrática.
Por fim,. para iii., se 2kp + 1 = (lp + l)u, então u = 1 (modp),
de maneira que u = hp + 1, para algum h > _ 1·, se h > _ 2 , ent-ao 7. Para o item (a), se todo divisor primo de 2n - 1 fosse congruente a
n = (lp+ l)(hp+ 1) 2: (2p+ 1) 2 > 2(2p+ l)p+ 1 2: 2kp+ 1 = n, um ±1, módulo 12, teríamos 2n - 1 = ±1 (mod 12); mostre que isso é
absurdo; logo, h = O ou 1 e, se h = 1, então (p + 1) / (2kp + 1), 0 que impossível. Para o item (b), se (2n - 1) f (3m - 1) e p é como em
é um absurdo (pois p + 1 é par e 2kp + 1 é ímpar. · (a), então 3m = 1 (modp) ou, ainda, 3m+l = 3 (modp); como m + 1
é par, seguiria que 3 seria um resíduo quadrático, módulo p; use o
9. O item (a) segue do pequeno teorema de Fermat, juntamente com O resultado do problema anterior para chegar a uma contradição.
fato de que xf_+xi+xt+x!+xg = O(modp) se, e só se, J(x 1 , ... , xs) =
1 (modp). O item (b) segue da fórmula do desenvolvimento multino- 8. Para o item (a), use a lei da reciprocidade quadrática. Para o item
mial (cf. problema 1.4.2 do volume 4) e do resultado do item (a). (b), use o resultado do corolário 7.20, juntamente com o fato de que
p - 1 é uma potência de 2.

9. Para o item (a) faça indução sobre k 2: 3, mostrando que, se x~ =


Seção 7.3 a (mod 2k), então x~+l = a (mod 2k+1 ), com Xk+1 = Xk ou xk + 2k-I _
Para o item (b) faça uma indução análoga; mais precisamente, se
1. Use o critério de Euler. x~ = pkq + a, faça Xk+l = Xk + pkt, com t E Z, e imponha a validade
da congruência x~+l = a (modp) para deduzir que t deve satisfazer a
2. Adapte a prova do exemplo 7.24 ao presente caso para obter infinitos congruência linear 2xkt = -q (modp); por fim, mostre que é sempre
valores de k com a propriedade desejada. possível escolher um tal t.

3. Comece multiplicando a igualdade do enunciado por 4a e completando 10. Se x 2 = a (mod n) para um certo inteiro x, então x 2 = a (mod 2k) e
quadrados. x 2 = a (mod p7;), para 1 :S i :S t; basta, pois, aplicar os resultados do
problema anterior, juntamente com o critério de Euler. Reciproca-
4. Mostre inicialmente que , se existirem tais x e y , enta-0 eles sao
- pnmos
·
mente, uma vez satisfeitas as condições (i) e (ii), aplique novamente
com 122. Use, então, o resultado do problema anterior para con-
os resultados do problema anterior, juntamente com o critério de Eu-
cluir que 17 é resíduo quadrático módulo 61. Por fim, aplique a lei
ler, para garantir a existência de inteiros xo, x 1 , ... , Xt tais que
238 Sugestões e Soluções 239

x5 =a (mod 2k) e x; =a (mod P7i), para 1 ::=:; i ::=:; t; por fim, aplique Seção 7.4
o teorema chinês dos restos para obter, a partir dos x/s, um inteiro
x que satisfaça todas essas congruências simultaneamente. 1. Com o auxílio da proposição 5.9 mostre que, se existissem inteiros k,
l, x, y e z como no enunciado, com l 2:: 1, então x, y e z seriam todos
11. Numere as crianças no sentido horário e de maneira contínua de
' 1, pares.
forma que o professor encontre a primeira criança nas posições 1, n+
2n + 1, 3n + 1, ... , e analogamente para as demais crianças. Mostre 2. Suponha, por contradição, que a+ b é par. Então, a+ c2 = a2 - b2 =
que o professor entregará doces às crianças nas posições k(\+ 1 ), para O(mod4), de sorte que a=
-c2 (mod4). Conclua, a partir daí, que
k E N, de sorte que basta encontrarmos os valores de n tais que a(a - 1) tem um fator primo congruente a 3 módulo 4 que aparece
,
a congruencia. x(x+l)
- 2- = a ( mo d n ) ten h a soluçao,
_ para todo inteiro
com expoente ímpar em sua decomposição canônica.
1 :S a :S n. Em seguida, observe que, se tal congruência tem solução,
então o mesmo sucede com a congruência (2x + 1) 2 = 8a+ 1 (mod n), 3. Observe que 1995 = 3 · 5 · 7 · 19, com 3, 7, 19 = 3 (mod4). Por
e use o problema 10 para mostrar que n tem de ser uma potência de outro lado, se d é um divisor positivo de 1995 (o caso d < O pode
2. ser tratado de forma totalmente análoga), analise separadamente as
quatro possibilidades a seguir: d= 1, d= 5, 1 < d 1 (3·7·19) e d= 5d',
12. Para ::::}, use o pequeno teorema de Fermat. Para ~, sejam a uma onde 1 < d' < (3 · 7 · 19), de acordo com os seguintes exemplos: (i) se
raiz primitiva, módulo p, e j E N tal que JJ = -1 ( mod Pd 1 ); em d= 5, então x 2 + y 2 = 5(x - y) se, e só se, (2x - 5) 2 + (2y + 5) 2 = 50,
seguida, tome k E N tal que ak = aJ (modp) e mostre que k Id e de forma que 2x - 5 = ±1, ±7; (ii) se 1 < d 1 (3 · 7 · 19), então,
xo = ak/d resolve a congruência xn = a(modp). como x 2 + y 2 = O(mod d), o argumento esboçado no passo (ii) da
demonstração do teorema 7.32 garante que d I x e d I y; pondo
13. Os itens de (a) a (d) utilizam somente manipulações algébricas sim- x = da e y = db, obtemos a2 + b2 = a - b, a qual equivale à primeira
ples e propriedades elementares de congruências. Para a primeira das quatro possibilidades acima.
parte do item (e), use o teorema de Wilson, juntamente com o cri-
tério de Euler; para a segunda parte, suponha, por contradição, que 4. O exemplo 1.39 garante a existência de infinitos primos da forma 4k+
ri= rj (modp) e use o fato de que p f (j2-i 2 ). Para a segunda parte 3; escolha, então, n primos q1, ... , qn, todos congruentes a 3 módulo 4,
do item (f), conclua que e aplique o teorema chinês dos restos, juntamente com o teorema 7.32,
p-1 p-1
ao sistema de congruências lineares x = -i .+ qi (modq;), para 1 <
-2- -2-
i ::=:; n.
L rj =Li= O(modp),
j=l j=l 5. Para o item (d), use a identidade (7.9).
onde, na última congruência, utilizamos o fato de que p 2:: 5. 6. Para o item (a), suponha que n tem um divisor ímpar d> 1. Então,
como (2d - 1) 1 (2n - 1), temos que (2d - 1) 1 (m 2 + 9), o qual
é uma soma de dois quadrados perfeitos. Contudo, como 2d - 1 é
da forma 4k - 1, ele deve ter algum divisor primo p dessa forma.
Segue, agora, da discussão da teoria desta seção que p I m e p 1 3,
"",@

i 240 Sugestões e Soluções


1
d! forma que P = 3. Logo, 3 1 (2d - 1), o que é um absurdo, já que
i 2 -1 = (-l)d-1 = -2(mod3). Para o item (b), comece observando
1
1 que o caso k = 1 é trivial. Agora, suponha que (22k-i -1) 1 (m2 +9)
b 2k-l k 2 k-1
1
eo serveque(2 +1) 1 ((3·2 2 - )2+9). Emseguida,escolha,com
1 o auxílio do teorema chinês dos restos, mk E N tal quer
- ( 2k-l k-2
mk=mk-l mod2 -1) e mk=3·22 (mod22k-i+l).

É imediato verificar que 22k-i - 1 e 22k-i + 1 dividem m2k + 9·, mas,


como tais fatores são primos entre si, concluímos que seu produto Referências
também divide m~ + 9.

[1] AIGNER, M. e ZIEGLER, G. (2010) Proofs from THE BOOK.


Springer-Verlag.

[2] ANDREWS, G. (1994). Number Theory. Dover.

[3] APOSTOL, T. (1976). Introduction to Analytic Number Theory.


Springer-Verlag.

[4] DE FIGUEIREDO, D. G. (1987). Análise de Fourier e Equações


Diferenciais Parciais. Instituto Nacional de Matemática Pura e
Aplicada.

[5] LANDAU, E. (2002). Teoria Elementar dos Números. Ciência


Moderna.

[6] LIMA, H. N. (2011). Limites e Funções Aritméticas. Preprint.

[7] ROBERTS, J. (1978). Elementary number theory: a problem ori-


ented approach. MIT Press.

241
242 Referências

[8] SCHEINERMAN, E. (2010). Matemática Discreta, uma Introdu-


ção. Cengage Learning.

[9] SINGH, S. (1998). O Último Teorema de Fermat. Record.

[10] STEIN, E. e SHAKARCHI, R. (2003). Fourier Analysis. An In-


troduction. Princeton University Press.
CAPÍTULO A
[11] VAINSENCHER, 1. (1996). Introdução às Curvas Algébricas Pla-
nas. Instituto Nacional de Matemática Pura e Aplicada.

Glossário

APMO: Asian-Pacific Mathematical Olympiad.

Áustria-Polônia: Olimpíada de Matemática Austro-Polonesa.

BMO: Balkan Mathematical Olympiad.

Baltic Way: Baltic Way Mathematical Contest.

Crux: Crux Mathematicorum, periódico de problemas da Sociedade


Canadense de Matemática.

IMO: International Mathematical Olympiad.

Israel-Hungria: Competição Binacional Israel-Hungria.

243
244 Glossário

Miklós-Schweitzer: The Miklós-Schweitzer Mathematics Competi-


tion (Hungria).

NMC: Nordic Mathematical Contest.

OBM: Olimpíada Brasileira de Matemática.


Índice Remissivo
OBMU: Olimpíada Brasileira de Matemática para Universitários.

OCM: Olimpíada Cearense de Matemática.

OCS: Olimpíada de Matemática do Cone Sul.


Algoritmo propriedades elementares da,
OIM: Olimpíada Ibero-americana de Matemática. da divisão, 5 120
de Euclides, 24 relação de, 153
OIMU: Olimpíada Ibero-americana de Matemática Universitária. Congruências
Bézout
lineares, 145
ORM: Olimpíada Rioplatense de Matemática. Étienne, 14
Cubo perfeito, 12
teorema de, 14
Putnarn: The William Lowell Mathematics Competition (Estados Cesàro, Ernesto, 105 Decomposição canônica de um in-
Unidos). Chebyshev teiro, 42
Pafnuty, 92 Descida de Fermat, 55
Torneio das Cidades: The Tournament of the Towns, olimpíada teorema de, 102 Diofanto de Alexandria, 22
intermunicipal mundial de Matemática. Chevalley Dirichlet
teorema de, 186 Gustav L., 39, 64
Chevalley, Claude, 186 lema de, 64
Classe Divisão
de congruência, 153 algoritmo da, 5
Congruência quociente de uma, 5
classe de, 153 resto de uma, 5
como relação de equivalência, Divisibilidade
120 por 11, critério de, 10

245
246 ÍNDICE REMISSIVO ÍNDICE REMISSIVO 247

por 9, critério de, 10, 122 160, 207 Hadamard de Gauss, 192
propriedades elementares da, Evento, 106 Jacques, 92 Liouville
3 teorema de, 92 Joseph, 86
relação de, 1 Fórmula teorema de, 86
de inversão de Mobius, 79 Identidade
Divisor
de Euler, 204 Mobius
comum, 13 de Legendre, 45
Inteiro fórmula de inversão de, 79
comum, máximo, 13 para cp(n), 82
divisível por outro, 2 função de, 77
de um inteiro, 2 Fermat
divisor de um, 2 Máximo divisor comum, 13
positivo, 2 descida de, 55
livre de quadrados, 47 Múltiplo
equação de, 56
múltiplo de um, 2 comum, mínimo, 28
Eisenstein grande teorema de, 56
mais próximo, 12 de um inteiro, 2
Ferdinand, 196 número de, 13
Inteiros Mínimo múltiplo comum, 28
Equação pequeno teorema de, 134
congruentes, 118 mmc, 28
de Fermat, 56 Pierre Simon de,. 52
primos entre si, 13
de Pítágoras, 51 teorema de, 202, 204
relativamente primos, 13 Número
de Pell, 63 Fração irredutível, 17
Inverso ímpar, 2
Diofantina, 22 Função
multiplicativo, 164 abundante, 86
diofantina linear, 23 cp de Euler, 81
Inverso módulo n, 145 composto, 35
Eratóstenes aritmética, 73
de Fermat, 13
crivo de, 37 aritmética multiplicativa, 73 Lagrange
par, 2
de Cirene, 38 de Euler, fórmula para a, 82 Joseph L., 207
perfeito, 85
teorema de, 36 de Euler, multiplicatividade da, teorema de, 207
primo, 35
Espaço amostral, 105 82 Lamé
primo, teorema do, 92
Euclides de Mobius, 77 teorema de, 35
algoritmo de, 24 número de divisores positivos, Lamé, Gabriel, 35 Ordem módulo n, 170
lema de, 36 74 Legendre
teorema de, 37 soma de divisores positivos, 76 Adrien-Marie, 45 Parte fracionária, 64
Euler fórmula de, 45 Parte inteira, 5
critério de, 189 Gauss símbolo de, 190 Pell
função cp de, 81 lei da reciprocidade quadrática Lema equação de, 63
identidade de, 204 de, 196 de Dirichlet, 64 John, 63
teorema de, 62, 82, 85, 96, 136, lema de, 192 de Euclides, 36 Pitágoras
248 ÍNDICE REMISSIVO

equação de, 51 de Eratóstenes, 36


de Euclides, 37
•!lseM
Pitágoras de Sarnas, 53
Primo, 35 de Euler, 62, 82, 85, 96, 136,
(continuação dos títulos publicados)
Probabilidade, 105 160, 207 Tópicos de Matemática Elementar - Volume 6 - Polinômios - A. Caminha
distribuição de, 105 de Fermat, 202, 204 Treze Viagens pelo Mundo da Matemática - C. Correia de Sa e J. Rocha (editores)
de Fermat, grande, 56 Como Resolver Problemas Matemáticos - T. Tao
Quadrado perfeito, 6 de Fermat, pequeno, 134 Geometria em Sala de Aula - A. C. P. Hellmeister (Comitê Editorial da RPM)
Quociente, 5 de Hadamard, 92 COLEÇÃO PROFMAT
de Lagrange, 207
Raiz Introdução à Álgebra Linear - A. Hefez e C.S. Fernandez
de Lamé, 35
de uma congruência, 145 Tópicos de Teoria dos Números - C. G. Moreira, F. E Brochero e N. C. Saldanha
de Liouville, 86
Raiz primitiva Polinômios e Equações Algébricas - A. Hefez e M.L. Villela
de Sophie Germain, 143 Tópicos de Historia de Matemática - T. Roque e J. Bosco Pitombeira
módulo n, 175
de Wilson, 146 Recursos Computacionais no Ensino de Matemática - V. Giraldo, P. Caetano e F.
Relação de Wilson, generalização do, Mattos
de congruência, 153 Temas e Problemas Elementares - E. L. Lima, P. C. Pinto Carvalho, E. Wagner e
184 A. C. Morgado
de divisibilidade, 1
do número primo, 92 Números e Funções Reais - E. L. Lima
Resíduo fundamental da aritmética, 41 Aritmética - Abramo Hefez
n-ésimo módulo p, 201 Geometria - A. Caminha
Terno Pitagórico, 53
quadrático módulo n, 188 Avaliação Educacional - M. Rabelo
Matemática Discreta - A. Morgado e P.C.P. Carvalho
Resto, 5 Unidade
em Zn, 164 COLEÇÃO INICIAÇÃO CIENTÍFICA
SCR, 154
Sistema Wiles, Andrew, 56 Números Irracionais e Transcendentes - D. G. de Figueiredo
completo de invertíveis, 158 Wilson Números Racionais e Irracionais - I. Niven
generalização do teorema de, Tópicos Especiais em Álgebra - J. F. S. Andrade
completo de restos, 154
Soma dos divisores positivos, 77 184 COLEÇÃO TEXTOS UNIVERSITÁRIOS
teorema de, 146
Taylor, Richard, 56 Introdução à Computação Algébrica com o Maple - L. N. de Andrade
Elementos de Aritmética - A. Hefez
Teorema
Métodos Matemáticos para a Engenharia - E. C. de Oliveira e M. Tygel
chinês dos restos, 148 Geometria Diferencial de Curvas e Superfícies - M. P. do Carmo
de Bézout, 14 Matemática Discreta - L. Lovász, J. Pelikán e K. Vesztergombi
de Chebyshev, 102 Álgebra Linear: Um segundo Curso - H. P. Bueno
de Chevalley, 186 Introdução às Funções de uma Variável Complexa - C. S. Fernandez e N. C.
Bernardes Jr.
.!SBM
(continuação dos títulos publicados)
Elementos de Topologia Geral - E. L. Lima
A Construção dos Números - J. Ferreira
Introdução à Geometria Projetiva - A. Barros e P. Andrade
Análise Vetorial Clássica - F. Acker
Funções, Limites e Continuidade - P. Ribenboim
Fundamentos de Análise Funcional - D. Pellegrino, E. Teixeira e G. Botelho
Teoria dos Números Transcendentes - D. Marquez
Introdução à Geometria Hiperbólica - O modelo de Poincaré - P. Andrade

COLEÇÃO MATEMÁTICA APLICADA

Introdução à Inferência Estatística - H. Bolfarine e M. Sandoval


Discretização de Equações Diferenciais Parciais - J. Cuminato.e M. Meneguette

COLEÇÃO OLIMPÍADAS DE MATEMÁTICA

Olimpíadas Brasileiras de Matemática, la a 8a - E. Mega, R. Watanabe


Olimpíadas Brasileiras de Matemática, 9a a 16a - C. Moreira, E. Motta,
E. Tengan, L. Amâncio, N. C. Saldanha e P. Rodrigues
21 Aulas de Matemática Olímpica - C. Y. Shine
Iniciação à Matemática: Um curso com problemas e soluções - K. I. M. Oliveira e A.
J. C. Fernández

COLEÇÃO FRONTEIRAS DA MATEMÁTICA

Fundamentos da Teoria Ergódica - M.Viana e K. Oliveira

Você também pode gostar